VOL. 45, NO. 1 • 2012

“Designer”-Surfactant-Enabled Cross-Couplings in M ICHI ICA A DR C L TA A

Water at Room Temperature

2 8 th 0

6

1

9 2 The Growing Impact of Niobium in Organic 1 4 A N 5 Y R Synthesis and Catalysis N I V E R S A For convenient General Reaction Equation trifluoromethylation N N Cu I CF3 of aryl iodides. CF 3

DMF, 25-50 oC, 18 h

Representative Reaction Scope

CF3 CF3 CF3

H Me Hartwig Trifluoromethylations MeO 96% O 89% O 93% Aldrich Chemistry would like to introduce Trifluoromethylator™, a highly efficient trifluoromethylation reagent from the CF3 CF3 CF3 laboratory of Professor John Hartwig and Catylix, Inc.

Trifluoromethylator™ (Phen)Cu-CF3 (Prod. No. L510009) is a EtO2C BnO AcHN convenient, thermally stable, single-component reagent for 94% 89% 85% the trifluoromethylation of aryl iodides. It provides: O CF CF CF • Tolerance of electrophilic functional groups, including 3 3 H 3 aldehydes, nitroarenes, ketones and esters O2N Ph OAc • High yields under mild conditions for electron-rich and quant. 83% 68% OMe electron-deficient iodoarenes, as well as sterically hindered iodoarenes O BOM Br CF3 CF3 Increased flexibility for carrying out ester hydrolysis in N • O your synthesis O N 89% 78% BOM

CF3 Cl N N F3C CF3 Cu N CF3 Cl N CO2Et L510009 89% 99% Boc 69% Solutions of Trifluoromethylator™ in toluene CF3 CF3 are safe to use at process scale. n-C6H11 99% OMOM OMOM To request custom and bulk quantities Reference: Morimoto, H.; Tsubogo, T.; Litvinas, N. D.; Hartwig, J. F. CF3 of Trifluoromethylator™, visit Angew. Chem. Int. Ed. 2011, 50, 3793. 97% Aldrich.com/catylix

Trifl uoromethylator is a trademark of Catylix, Inc. 78095

78095_ALD Trifluoromethylator Ad (Acta 45.1).indd 1 3/28/2012 11:41:17 AM 1 VOL. 45, NO. 1 • 2012

Aldrich Chemical Co., Inc. “PLEASE BOTHER US.” 6000 N. Teutonia Ave. Milwaukee, WI 53209, USA Dear Fellow Chemists,

To Place Orders We have had many suggestions for the DalPhos ligands shown below. These were recently developed by Professor Mark Stradiotto and his group Telephone 800-325-3010 (USA) at Dalhousie University. Both are air-stable, chelating N,P ligands that enable FAX 800-325-5052 (USA) the Pd-catalyzed cross-coupling of ammonia. Mor-DalPhos is also effective in or 414-438-2199 coupling hydrazine and acetone, and in the gold–catalyzed hydroamination. Mail P.O. Box 2060 The catalysts are used with low loadings and exhibit excellent functional Milwaukee, WI 53201, USA group tolerance and chemoselectivty. Customer & Technical Services (a) Lundgren, R. J.; Peters, B. D.; Alsabeh, P. G.; Stradiotto, M. Angew. Chem., Int. Ed. 2010, 49, 4071. (b) Lundgren, R. J.; Stradiotto, M. Angew. Chem., Int. Ed. 2010, 49, 8686. Customer Inquiries 800-325-3010 Technical Service 800-231-8327 N CH SAFC® 800-244-1173 N 3 P O Custom Synthesis 800-244-1173 P CH3 Flavors & Fragrances 800-227-4563 International 414-438-3850 24-Hour Emergency 414-438-3850 751618 751413 Website sigma-aldrich.com 751618 Di(1-adamantyl)-2-morpholinophenylphosphine (Mor-DalPhos) 250 mg Email [email protected] 1 g 751413 Di(1-adamantyl)-2-dimethylaminophenylphosphine (Me-DalPhos) 250 mg General Correspondence 1 g Editor: Sharbil J. Firsan, Ph.D. P.O. Box 2988, Milwaukee, WI 53201, USA We welcome fresh product ideas from you. Do you need a compound that isn’t listed in our [email protected] catalog? Ask Aldrich! For over 60 years, your research needs and suggestions have shaped the Aldrich product offering. To submit your suggestion visit Aldrich.com/pleasebotherus. Subscriptions Request your FREE subscription to the Aldrichimica Acta, through: John Radke Web: Aldrich.com/acta Director of Marketing, Chemistry Email: [email protected] TABLE OF CONTENTS “Designer”-Surfactant-Enabled Cross-Couplings in Water at Room Temperature ...... 3 Phone: 800-325-3010 (USA) Bruce H. Lipshutz* and Subir Ghorai, University of California, Santa Barbara, and Sigma-Aldrich Co., Mail:  Attn: Mailroom Sheboygan Falls, Wisconsin Aldrich Chemical Co., Inc. The Growing Impact of Niobium in Organic Synthesis and Catalysis...... 19 Sigma-Aldrich Corporation Valdemar Lacerda, Jr.,* Deborah Araujo dos Santos, Luiz Carlos da Silva-Filho, Sandro José Greco, and P.O. Box 2988 Reginaldo Bezerra dos Santos, Universidade Federal do Espírito Santo and Universidade Estadual Milwaukee, WI 53201-2988 Paulista (Brasil) ABOUT OUR COVER Venice: The Dogana and San Giorgio Maggiore (oil on canvas, 91.5 × 122.0 cm) was completed in 1834 by Joseph Mallord William Turner (London, 1775–1851). A prolific British artist and an influential member of the Romantic Movement of the late eighteenth and first half of the nineteenth century, he elevated landscape painting to a level not achieved before The entire Aldrichimica Acta archive is also available free of and introduced several innovations to the genre. His talent charge at Aldrich.com/acta. manifested itself very early in life, and he was much admired

Aldrichimica Acta (ISSN 0002-5100) is a publication of Aldrich. as an artist in life and death, with institutions, works, and artistic Detail from Venice: The Dogana and San Giorgio Maggiore. Aldrich is a member of the Sigma-Aldrich Group. prizes dedicated to preserving his legacy. Photograph © Board of Trustees, National Gallery of Art, Washington. This painting depicts a bustling maritime scene in the Grand Canal of Venice, which Turner had visited several ©2012 Sigma-Aldrich Co. LLC. All rights reserved. SIGMA, times. (Another Venetian cityscape, painted seven years earlier by another British Romantic Movement artist, R. SAFC, SIGMA-ALDRICH, ALDRICH, and SUPELCO are trademarks P. Bonington, has graced the cover of Aldrichimica Acta, Vol. 43, No. 1.) Turner intended this work as a celebration of Sigma-Aldrich Co. LLC, registered in the US and other of commerce, as symbolized by the statue of Fortune atop the Dogana (Customs building) in the foreground. countries. Add Aldrich is a trademark of Sigma-Aldrich Co. LLC. Aldrich brand products are sold through Sigma-Aldrich, Inc. While Turner was not concerned with a faithful depiction of the scene, as evidenced by the apparent widening Purchaser must determine the suitability of the product(s) of the canal and placement of San Giorgio’s church, his skill as a draftsman and his mastery of perspective for their particular use. Additional terms and conditions may drawing are evident in his precise, linear rendering of the buildings and sharp angles. His color combinations, apply. Please see product information on the Sigma-Aldrich freely handled layers of paint, and the sparkling water and light sky exemplify his lifelong preoccupation with website at www.sigmaaldrich.com and/or on the reverse side the effects and significance of light in an awe-inspiring natural world. of the invoice or packing slip. This painting is part of the Widener Collection at the National Gallery of Art, Washington, DC. TPGS-750-M Cat. No. 733857

O CH3 O O H3CO O H3C n O O CH3 Want to n = ~16 H3C H C CH3 3 run transition- 3 transition metal catalyst partner A TPGS-750-M metal-catalyzed + product C water, rt reactions in water at partner B

Heck Sonogashira room temperature? O O

MeO

97% 99%

Suzuki-Miyaura Negishi-like

CO Et OMe 2 4

87% (E/Z = 99/1) Congratulations to Professor Bruce Lipshutz, University of 88% California, Santa Barbara, winner of the 2011 Presidential Green Chemistry Challenge Award: Academic, for the development C-H activation Fujiwara-Moritani of TPGS-750-M, an amphiphile that enables transition-metal- OMe CO n-Bu catalyzed reactions to be run in water, at room temperature. 2 H H N Applicable to many transition-metal-catalyzed reactions N N • O O • Scalable OMe OMe • Highly water-soluble 68% 83% • No frothing at workup ring-closing metathesis cross-metathesis Easily separated from reaction product(s) and recycled • N O Ts OTBS • Amenable to high-throughput screening 88% 74%

aminations H N

For greener chemistry N in your lab, Add Aldrich. 98% Aldrich.com/tpgs750m Lipshutz, B. H. et al. J. Org. Chem. 2011, 76, 4379.

78375

78375_ALD Lipshutz TPGS-750-M Ad (Acta 45.1).indd 1 3/23/2012 11:16:59 AM 3 VOL. 45, NO. 1 • 2012 “Designer”-Surfactant-Enabled Cross-Couplings in Water at Room Temperature

Bruce H. Lipshutz*,a and Subir Ghoraib a Department of Chemistry & Biochemistry University of California Santa Barbara, CA 93106, USA Email: [email protected] b Catalysis R&D Sigma-Aldrich Co. 5485 County Road V Prof. Bruce H. Lipshutz Dr. Subir Ghorai Sheboygan Falls, WI 53085, USA

Keywords. green chemistry; micellar catalysis; designer surfactants; within their lipophilic cores (Figure 1). While a disaster of this cross-couplings; PTS; TPGS-750-M. magnitude needs the tincture of time to assess its full impact on the environment, from a purely chemical perspective, tremendous hope Abstract. New methodologies are discussed that allow for several has been placed on these surfactants in anticipation that they will commonly used transition-metal-catalyzed coupling reactions to prevent further damage to wildlife and shores. be conducted within aqueous micellar nanoparticles at ambient This micellar chemistry undertaken on a grand scale by the temperatures. petroleum industry highlights the potential for simple amphiphilic molecules to “solubilize” organic materials in a purely aqueous Outline medium. Many other industries use surfactants routinely; examples 1. Introduction include paint, cosmetics, cleaning, leather, carpet, asphalt, and pulp 2. Background & paper companies.3 Relatively small amounts have been used for

3. Ch emistry in PTS–H2O, an Update: a 1st-Generation Amphiphile decades in highly controlled environments, e.g., as “excipients” in for Transition-Metal-Catalyzed Cross-Couplings the pharmaceutical arena, to help increase dissolution of otherwise 3.1. Amination water-insoluble drugs in aqueous media. But where are the studies 3.2. Suzuki–Miyaura Coupling on their usage in synthesis? Why not apply the same concepts of 3.3. Silylation solubilization within micelles to reactants and catalysts that become, 3.4. C–H Activation albeit transiently, the occupants? Of course, some synthetic chemistry 3.5. Negishi-like Couplings on the Fly…in Water can be, and has been, done in micelles.4 Interestingly, this approach, 4. New Insights into Micellar Catalysis for Organic Synthesis oftentimes referred to as “micellar catalysis”, is technically a 4.1. Heck Coupling in PTS–3 M NaCl(aq) misnomer since the micelle is not participating as a catalyst in the 4.2. Olefin Metathesis at pH 2–3 reaction itself. Nonetheless, why should these, or thousands of other 5. De signing a Better Micelle: TPGS-750-M, a 2nd-Generation surfactants created by industry and designed for a narrow range of Amphiphile specialized applications, be the most appropriate for use in state- 5.1. CuH-Catalyzed Asymmetric Hydrosilylation of-the-art transition-metal-mediated cross-couplings? Since organic 5.2. Borylation of Aryl Halides reactions are oftentimes very sensitive to solvent effects,5 and since 6. Summary and Outlook the lipophilic portions of micelles are functioning as the reaction 7. Acknowledgements medium, which surfactant should be used in this capacity to best 8. References and Notes assist metal-catalyzed reactions in water at room temperature? No one knows. Perhaps it is time for synthetic chemists to start designing 1. Introduction surfactants for synthetic chemistry. Green chemistry recently took a front row seat on the world stage. The situation just a few years ago was not encouraging in that there Unfortunately, it was not associated with any special technological were virtually no general studies of the effect of varying micellar advance; rather, the 200-million-plus gallons (ca. 5 million barrels) of conditions on the most commonly used transition-metal-catalyzed oil that had leaked into the Gulf of Mexico focused attention on methods couplings. As green chemistry continues to expand, critical reviews for dispersing such a huge quantity of localized hydrocarbons.1 British have appeared highlighting micellar media in which the “hydrophobic Petroleum (BP) addressed this catastrophe by injecting dispersants at effect”—the tendency of nonpolar groups to cluster so as to shield the site, the key ingredient being a mix of surfactants. While this tactic themselves from contact with an aqueous environment6—assists with raised more than a few eyebrows with respect to additional pollutants organometallic processes in water, such as in oxidation and reduction having been introduced into the ecosphere, the presumption was reactions,7 as well as in selected C–C-bond forming reactions (e.g., that sodium dioctylsulfosuccinate (1), Span™ 80 (3), and a mixture hydroformylations). And so it was in recognition of the potential offered of TWEEN® 80 (4) and 85 (5)—present in dispersants COREXIT® by micellar catalysis, performed in water as the gross reaction medium EC9500 and COREXIT® EC95272—would help to “solubilize” the oil (and not the solvent), that we set out to develop “designer” surfactants by forming micelles, thereby relocating over time the oil deposited for use in transition-metal-catalyzed cross-coupling reactions. “Designer”-Surfactant-Enabled Cross-Couplings in Water at Room Temperature 4 Bruce H. Lipshutz* and Subir Ghorai

2. Background In our previous review in this journal,8 we introduced PTS (7; Figure O Et Me 9 n-Bu O HO O 2) as a useful (nonionic) surfactant; a nanomicelle-forming species in O n-Bu O n-Bu which a variety of Pd- and Ru-catalyzed couplings took place in water Et NaO3S O Me at room temperature. The idea behind the choice of PTS was simple: dioctyl sodium sulfosuccinate (DSS, 1) 2 in the Paul Anastas sense, it follows the “12 Principles of Green 10a 10b HO OH O O Chemistry”, as explained in Benign by Design. That is—in being HO OH z y composed of racemic vitamin E, sebacic acid, and PEG-600—neither H OH O OH O O O x PTS nor any of its three components is environmentally of concern. H O n-Oct 11 O n-Oct O Indeed, PTS is FDA GRAS affirmed for use in dietary supplements. 7 w 7 O By itself, PTS is a provitamin; a modified version of “ester-E.” ™ ® Span 80 (3)TWEEN 80 [w + x + y + z = 20] (4) By comparison, readily available alternative amphiphiles, such as ® ® 12 O O Triton X-100 and those in the Brij series, only on occasion lead O O n-Oct O O n-Oct to comparable results in cross-coupling reactions. Even the closely 7 z y 7 13 O OH structurally related TPGS (TPGS-1000, Figure 2) affords quite OH Me O O x O n-Oct different outcomes in cross-couplings under otherwise identical O w 7 conditions. This seems particularly odd, since both PTS and O TWEEN® 85 [w + x + y + z = 20] (5) 6 TPGS-1000 share the same micellar lipophilic interior in the form of α-tocopherol. So what’s responsible for the commonly observed greater rates of conversion in PTS? The answer seems to be that both Figure 1. Ingredients of COREXIT® EC9500 (a Mixture of 1–5) and COREXIT® the size and shape of their nanoparticles matter (Figure 3).8,14 For EC9527 (Includes 1–6). (Ref. 2b) PTS, both 8–10-nm spheres and larger worm- or rod-like particles are present (together averaging ca. 25 nm) according to cryo-TEM analysis (see Section 5). By contrast, TPGS in water forms very sharp 12–13-nm spherical micelles as indicated by microscopy and sebacic 14 acid Dynamic Light Scattering (DLS) measurements.

Me O O O 3. Chemistry in PTS–H2O, an Update: a 1st-Generation Me O H Me 4 14 Amphiphile for Transition-Metal-Catalyzed Cross- O Me O Me 3 Couplings Me PEG-600 It is not uncommon for synthetic chemistry to advance at a far α-tocopherol greater pace than does our understanding of why the chemistry goes PTS (7) as observed. This is certainly true here as well, involving non-ionic 15 Me O surfactants that self-aggregate to form nanoreactors, or what Fujita 16 O O and co-workers refer to as “functional molecular flasks.” Early work Me O H Me n O focused mainly on a series of Pd-catalyzed “name” reactions (e.g., Me O Me (n = ca. 23) 17 18 19 3 Heck, Suzuki–Miyaura, and copper-free Sonogashira couplings), Me as well as olefin cross-20 and ring-closing metathesis21 reactions, all of vitamin E TPGS-1000 ("TPGS") which could be carried out in water at room temperature (Figure 4). Figure 2. Polyoxyethanyl-α-tocopheryl Sebacate (PTS, 7) and TPGS-1000. Since these initial reports, considerable progress has been made (Ref. 8,13) on many related Pd- or Ru-catalyzed couplings. In most cases, the species involved are tolerant of the presence of water as the gross reaction medium.22 Although these couplings are likely occurring within the lipophilic portions of the micelles, some of a micelle’s

Suzuki–Miyaura amination C–H activation Negishi Sonogashira borylation Heck silylation asymmetric hydrosilylation

cross- increasing metathesis 50 nm A 50 nm B scope

ring-closing metathesis Figure 3. Cryo-TEM Data Comparing Nanoparticles of PTS (A) and TPGS- 1000 (B). (Ref. 8,14) Figure 4. PTS-Enabled Reactions in Water at Room Temperature. 5 VOL. 45, NO. 1 • 2012

33a occupants may be exposed at any time to the surrounding water, previously using a variety of protic or Lewis acids (e.g., SnCl2, 23 33b 33a 33b given its dynamic character. That is, there is constant exchange Et3B, RCO2H, and CO2 ), efforts toward the exclusion of water of monomeric units of surfactant between micellar arrays. This are common. Those reactions run in pure water typically require motion creates a mechanism by which educts, catalysts, and reaction harsh conditions.33c–e Under similar micellar conditions applied to product(s) can enter and exit a micelle. But the exchange phenomenon allylic ethers (above), couplings give highly favored linear rather than also leads to exposure to an aqueous environment through which they must traverse. This can have major (beneficial) consequences P(t-Bu) for the desired transformation; that is, the content of the water can be 3 t-Bu Cl t-Bu Cl Pd altered to great synthetic advantage. Foreshadowing associated with Me2N P Pd P NMe2 Pd Pd 8 t-Bu Cl t-Bu Cl these effects was noted in the previous Aldrichimica Acta review, P(t-Bu)3 where couplings run in seawater led, in some cases, to faster reactions 8 PdCl2(amphos)2 (9)[PdCl(allyl)]2 (10) than had been observed in pure (HPLC grade) water. Changes 24 in the ionic strength and pH of the aqueous medium have now P(t-Bu) 2 PPh2 been studied and, indeed, there are benefits to be had from such O PdCl Fe 2 Fe Ph P PPh perturbations resulting from the simple addition of selected salts to 2 2 P Pd PPh2 (t-Bu)2 water (vide infra). Cl Cl

PdCl2(DPEPhos) (11) PdCl2(dtbpf) (12) dppf (13) 3.1. Amination The use of PTS is an effective enabling technology for two types MeMe of amination, both taking place in water at room temperature. Ph Me O Unsymmetrical di- and triarylamines can be constructed using aryl Ph P(t-Bu)2 O PPh2 PPh2 bromides and aniline derivatives. In the presence of [PdCl(allyl)]2, PPh2 PPh2 ® ® Takasago’s ligand, cBRIDP (14, Figure 5), was the most cBRIDP (14) DPEPhos (15) XantPhos (16) effective among several catalysts (8–16) screened. Although these transformations appear to be general in that a variety of reaction Figure 5. Ligands and/or Catalysts Used in Coupling Reactions. (Ref. 25) partners can be used and yields tend to be good, perhaps the most intriguing aspect of this methodology is the influence of the base (eq R' 25 1). Thus, while KOH (1.5 equiv) is oftentimes sufficient, reaction Br R' N 10 (0.5 mol %), 14 (2 mol %) Ar times can vary and may take up to a full day to reach completion. That R + N R H Ar base (1.5 equiv) KOH functions well in this capacity is rather interesting, since the 1.2 equiv PTS–H2O (2 wt %) coupling is occurring within the lipophilic core of the PTS micelle, rt, 0.75–23 h where presumably polar species are not to be found. It is certainly Me Ph H H possible that KOH remains in the aqueous phase, and that as species N Me N Me N Ph exchange between nanoreactors they are exposed to the surrounding Me F Me water, and it is here when the proton may be abstracted from the O participating nitrogen.26 Likely to be more effective, rate-wise, at KOH, 99% KOH, 81% KOH, 83% finding a protonated amine intermediate would be a more lipophilic H H H Me N N MeO N CF Me N base, capable of penetrating the micelle into the hydrophobic pocket 3 25 O (eq 2). A switch, therefore, to KOt-Bu, made a favorable difference MeO N in this regard, notwithstanding the fact that this base in water is OMe mainly KOH. Hence, the same outcome could be achieved by simply KOt-Bu, 94% KOt-Bu, quant.KOt-Bu, 83% CF 3 O Me adding tert-butyl alcohol to the original mixture containing KOH. H H N OMe N Even better, was addition of the commercially available potassium O 27 trimethylsilanolate (KOSiMe3), which reduced reaction times by NC almost an order of magnitude. Still more dramatic was inclusion of KOH–TIPS-OH, 97% KOH–TIPS-OH, 97% Cl the far more lipophilic potassium triisopropylsilanolate [(KOSi(i-Pr)3, or KO-TIPS], readily formed in situ from KOH and TIPS-OH. Representative Amination in PTS–H2O. 2’-(Trifluoromethyl)phenethyl 3-(5˝-chloro- 2˝-methylphenyl)aminobenzoate {R = 3-[2-F3CC6H4(CH2)2OC(O)], R’ = H, Ar = 5-Cl-2- Allylic aminations in water at room temperature can also be 25 MeC6H3}. Inside a dry box, a 5-mL round-bottom flask equipped with a stir bar and fit- accomplished using allylic phenyl ethers as substrates.28 These are ted with a rubber septum under argon was sequentially charged with 10 (3.0 mg, 0.008 unconventional partners in Pd-catalyzed couplings, where activation mmol), 14 (8.9 mg, 0.025 mmol), and KOH (89 mg, 1.58 mmol). Outside the dry box, under a positive flow of argon, were sequentially added via syringe to the mixture of 10 of the hydroxyl group in an allylic alcohol typically takes place in the and 14: degassed water (0.8 mL) and degassed 10 wt % PTS solution (0.2 mL) (to give a form of an acetate, carbonate, sulfonate, or phosphate.29 Nonetheless, degassed 2 wt % PTS solution; 1.0 mL), then TIPS-OH (320 μL, 1.61 mmol), 5-chloro-o-to- luidine (150 μL, 1.25 mmol), and, lastly, 2’-(trifluoromethyl)phenethyl 3-bromobenzoate under the influence of the hydrophobic effect, and in the presence (380 mg, 1.02 mmol). The milky reaction mixture was stirred under argon at rt for 45 min, of excess K2CO3 and methyl formate, couplings result in amination after which complete consumption of the aryl bromide was observed by GC analysis. predominantly at the least hindered, terminal site and give high E:Z The reaction mixture was diluted with brine and extracted with EtOAc. The combined 28 30 organic extracts were dried over anhydrous Na2SO4, filtered, and concentrated by rotary ratios (eq 3). DPEPhos (15) was found to be the ligand of choice, evaporation to give the crude residue. Purification by silica gel chromatography (gra- while the more rigid analogue XantPhos (16)31 led to only traces of dient from hexanes to 2% EtOAc in hexanes) afforded the product (428 mg, 97%) as a allylic amines. viscous, beige-colored oil. Perhaps more intriguing is the same net amination, …but with eq 1 (Ref. 25) allylic alcohols (eq 4).32 While in situ activation has been achieved “Designer”-Surfactant-Enabled Cross-Couplings in Water at Room Temperature 6 Bruce H. Lipshutz* and Subir Ghorai

branched products, also strongly favoring E isomers. Here again,

K2CO3 is the preferred base, while HCO2Me in excess is required. Since none of the intermediate resulting from transesterification between 10 (0.5 mol %) Br 14 (2 mol %) N the alcohol and methyl formate is observed at any point during the base (1.5 equiv) reaction, the proposed mechanism involves Pd(0), the educt, and the + N PTS–H2O (2 wt %) formate–reasonable given the presumed high concentration of species H rt Me Me within the micelles. Base Time Conv. 3.2. Suzuki–Miyaura Coupling 18 Et3N 42 h 79% As previously described, biaryls can be constructed from precursor KOH 44 h 72% KOH–t-BuOH 6 h 100% aryl halides or pseudo-halides Ar–X (X = I, Br, Cl, OSO2R) and boronic KOSiMe3 3 h 100% acids in 1% PTS in water mainly at ambient temperatures. Subsequent KOH–TIPS-OHa 0.5 h 100% to this work, various heteroaromatic halides were also shown to be a TIPS-OH = (i-Pr) SiOH; in this 3 34 case, a quantitative yield was amenable. Bromide was found to be the preferred leaving group (eq obtained. 5)34 and, like chlorides, their reactions occasionally required mild 34 eq 2 (Ref. 25) heating to 40 °C (eq 6). The choice of catalyst also varied by leaving group: bromides were best accommodated by PdCl2(dtbpf), 12, while

chlorides seemed more responsive to PdCl2(amphos)2, 9. A direct comparison with a literature case35 showed the potential for this green chemistry to be highly competitive with traditional organic media (eq R' 34 R' 10 (0.5 mol %), 15 (1 mol %) 7). A review on the Suzuki–Miyaura cross-coupling as an entry to R OPh + HN R N R" R" biaryls under green conditions has recently appeared, focusing on K2CO3 (1.5 equiv) 36 1.2–2.5 equiv HCO2Me (4 equiv) water as the reaction medium. PTS–H2O (2 wt %) Carbon–carbon bond formation between usually unreactive, rt, 0.3–5.0 h acid- and base-stable allylic phenyl ethers and arylboronic acids is R R' R" Yield

Ph Me 1-NpCH2 98% n-Oct Me 1-NpCH2 90% a Me 1-NpCH 80% 2 Br B(OH)2 12 (2 mol %) Ph Bn 4-BrC6H4CH2 82% R + R' R Ph H 2-MeC6H4 86% PTS–H2O (2 wt %) H EtO2C(Bn)CH b 85% 1.5 or 2.0 equiv Et3N (3 equiv) a o RCH=CHCH 2 = cyclohexen-3-yl. 22–40 C, 4–20 h R' b R" = (E)-4-(t-BuO CCH=CH)C H CH 2 6 4 2. N N F

N

eq 3 (Ref. 28) Ph N OMe N 82% 95% 95% O S

N R'R"NH (1.2–2.5 equiv) S 10 (2.5 mol %) R' 13 (5 mol %) R1 OH R1 N R1 R3 N R" + K2CO3 (3 equiv) 97% 99% 91% R2 R3 R2 R3 2 HCO2Me (4 equiv) R'R"N R PTS–H O (2 wt %), rt LB 2 eq 5 (Ref. 34)

Alcohol Product Yield L:B E:Z

OH NBn Me Me 2 84% >25:1 16:1 7 7

PdCl2(amphos)2 OH Cl B(OH)2 (2 mol %) R' Me NBn 2 76% 24:1 11:1 R + R' Me PTS–H2O (2 wt %) R Et N (3.0 or 5.0 equiv) 1.5 or 2.0 equiv 3 OH Me N(Me)Ph 90% 63:1 ---- OMe Me Me Me OMe S Ph OH Ph NH(o-Tol) 80% 100:0 all E H Me N N

OH MeO2C Bn N O 80% 100:0 all E N F3C Ph N Ph Bn Et2N O Me o o Me 40 C, 24 h 40 C, 24 h rt, 19 h 87% 95% 99% eq 4 (Ref. 32) eq 6 (Ref. 34) 7 VOL. 45, NO. 1 • 2012 also possible under micellar conditions.37 As with allylic aminations (vide supra), there is a strong tendency to generate linear products where conjugation is maintained. Aliphatic ethers, on the other hand, 77% yield o S favor branched products presumably due to the known faster rate of Me K2CO3, n-BuOH, 100 C Me 38 Cl reductive elimination from a more hindered Pd(II) intermediate. In S Pd (2 mol %), X-Phos addition to examples 17–19 (eq 8),37 linchpin 20 bearing both acetate + (4 mol % ), 10 h Me (HO)2B Me o and phenyl ether moieties can be sequentially coupled, where, e.g., Et3N, PTS–H2O, 38 C amination is followed by Suzuki–Miyaura coupling, all in one pot, all 90% yield in water at room temperature (eq 9).34 eq 7 (Ref. 34) 3.3. Silylation Palladium catalysis in PTS–water can also be extended to the formation of allylic silanes, likewise employing allylic phenyl ethers 39 as substrates, in the presence of Et3N (eq 10). Such reactions do R' B(OH) not occur in organic solvents (e.g., MeOH) at room temperature. 2 11 (2–8 mol %) R' R OPh + R" R" Moreover, whereas silylations of the corresponding acetates require PTS–H2O (2 wt %) R Et N (3.0 equiv) heating in organic solvents (e.g., DMF) for activation of a disilane 1.5 or 2.0 equiv 3 rt, 6–10 h 40 (R3Si–SiR3), cross-couplings in PTS nanoparticles take place at EtO O OMe ambient temperatures in ≤20 hours at a global concentration of 0.16 MeO Bn EtO o-Tol M. The most effective catalyst source is PdCl2(DPEPhos), 11, as EtO2C N o-Tol o-Tol O Ph MeO monodentate ligands on the metal (e.g., Ph3P), or bidentate chelation of palladium by dtppf or dtbpf, afford only moderate levels of conversion 17, 98% 18, 90% 19, 90% (40–60%). Of the various products possible, linear over branched and E over Z isomers are both favored.39 eq 8 (Ref. 37) 3.4. C–H Activation With more than a handful of reviews on the topic of aromatic C–H activation reactions in just the past few years,41 there is now a plethora of methods for introducing C–C bonds mainly ortho- to heteroatom- 10 (0.5 mol %), 16 (1 mol %) directing groups….but not in water at room temperature. This can AcO OPh Bn2N OPh Bn NH (1 equiv) 42 2 be done, however, employing micellar catalysis. Thus, aryl ureas, 20 PTS–H2O (2 wt %), rt, 5 h especially those bearing electron-donating groups, can be cross- coupled with aryl iodides in an aqueous medium containing any one 2-MeC6H4B(OH)2 (2 equiv) 11 (8 mol %), Et3N (3 equiv) ® 43 Bn2N o-Tol of several surfactants; including PTS, TPGS, Triton X-100, Solutol, rt, 19 h Brij® 30, and Brij® 35; where yields in optimization studies varied 76% (2 steps) between 65 and 76%. The corresponding “on water” experiment led to only a 35% yield in the same time frame. Since the best results eq 9 (Ref. 37) were obtained in nanomicelles containing Brij® 35, several additional examples were studied in this medium, along with ingredients

Pd(OAc)2, AgOAc, and HBF4. Yields were in the 70–97% range (eq 11),42 and products derived from double arylation were rarely observed, presumably reflecting the mild conditions involved. 11 (6 mol %) Me3Si–SiMe3 (21) or Arylations with Pd(OAc)2 are typically run in highly acidic media; PhMe2Si–SiMe2Ph (22) e.g., HBF4, TFA, TsOH, or HOAc; which facilitate loss of HOAc from R OPh R SiMe2R' Et3N (4 equiv) 44 the catalyst and lead to electrophilic attack on an aromatic ring. PTS–H2O (2 wt %) Alternatively, it has been shown that Fujiwara–Moritani coupling rt, ≤20 h 45 reactions with acrylates can be carried out in PTS–water (eq 12) Ra R' Yield E:Z by using a cationic source of palladium, as in commercially available 46 Ph Me 91% 10:1 [Pd(MeCN)4](BF4)2, Both benzoquinone (1 equiv) and AgNO3 (2 2-MeOC6H4 Me 86% >25:1 a equiv) are required. Double functionalization ortho to the amide In all cases, linear:branched 3-MeOC6H4 Me 90% 9:1 b b directing group is not observed here as well. = 25:1. PdCl2(Ph3P)2 (1.5 n-C8H17 Me 85% 3:1 mol %) and PdCl2(DPEPhos) Ph Ph 91% >25:1 (1.5 mol %) were used. 2-MeOC6H4 Ph 87% >25:1 3.5. Negishi-like Couplings on the Fly…in Water 3-MeOC6H4 Ph 91% 24:1 According to reference works on the topic, organozinc halides are 4-MeOC6H4 Ph 94% >25:1 4-EtO2CC6H4 Ph 95% >25:1 not moisture-tolerant.47 Indeed, Knochel and co-workers have spent, 2-Np Ph 93% >25:1 literally, decades elucidating the extent to which RZnX reagents can be used in the presence of protons of varying acidity.48 Free alcohols, in general, require protection; the presence of even tert-butyl alcohol takes its toll on couplings run in the presence of this additive.48a Water eq 10 (Ref. 39) is nowhere to be found among the various listings; hence, the message “Designer”-Surfactant-Enabled Cross-Couplings in Water at Room Temperature 8 Bruce H. Lipshutz* and Subir Ghorai

is clear: whether it’s “on water”, “in water”, or “with water”,49 the only H Ar 47 H Pd(OAc)2 (10 mol %) H conditions applicable to zinc reagents involve “no water”. N O AgOAc (2 equiv) N O + Ar I Could the textbooks be “wrong”? Not likely. But in this obvious NMe2 NMe2 aqueous HBF4 (5 equiv) R ® R acid–base chemistry between RZnX and H2O two assumptions Brij 35–H2O (2 wt %) rt, 20–72 h are implied: (i) that RZnX and H2O find each other, and (ii) that a stoichiometric amount (or more) of RZnX is used; otherwise, with OMe OMe Me lesser amounts, the anticipated yield must suffer. If both of these conditions are removed; that is, if RZnX could be “insulated” from H H H any water present, and if the amount of RZnX at any given time is N O N O N O minimized and yet, over time, a stoichiometric level of RZnX is NMe NMe NMe 2 Me 2 2 formed in the pot, then the desired Pd-catalyzed cross-couplings of OMe Me i-Pr organzinc reagents, in water, might be possible. Well, they are, thanks 76% 81% 74% eq 11 (Ref. 42) to micellar catalysis. For both aryl bromides50 and alkenyl halides,51 couplings with primary and secondary alkyl halides can now be accomplished in 3 4 3 4 R R [Pd(MeCN)4](BF4)2 R R a remarkably straightforward fashion. The recipe calls for the two H (10 mol %) precursor halides, a specific (commercially available) palladium R2O H + R2O CO2R' AgNO (2 equiv) 3 CO R' catalyst, and importantly, TMEDA. These are combined in water BQ (1 equiv) 2 NH 2 equiv NH 1 PTS–H2O (2 wt %) 1 (ca. 0.3 M) containing 2 wt % PTS to which is added inexpensive R rt, 20 h R O O zinc metal in the form of powder or dust; and the whole mixture is 23 24 BQ = 1,4-benzoquinone 25 then stirred. After a reasonable period of time, which is substrate- dependent, the cross-coupled product is obtained in good yield. 1 2 3 4 25 R R R R R' Yield Representative examples involving aromatic and heteroaromatic 50a a i-Pr Me H H n-C12H25 80% bromides are illustrated in eq 13. b Me2N Me H H Me(CH2)5CHMe 76% This remarkable process can be envisioned as depending entirely c n-Pr Me H H n-BuCH(Et)CH2 80% on the precise timing of the various events that need to take place, d Me n-Pr H H n-BuCH(Et)CH2 96% e i-Pr Me Me H 3,7-Me2C8H15 81% just like the gears of a fine-tuned pocket watch (Figure 6). Within f i-Pr Me H H Cy(CH2)2 77% the nanoparticles of PTS are housed the various reaction components, g i-Pr Me H MeO 3,7-Me2C8H15 80% densely packed due to the hydrophobic effect; no organometallic (RZnX) is present. As nanoparticles collide with zinc metal on its Representative Fujiwara–Moritani coupling of 23g and 24g to give cinnamate 3 25g.45 Anilide 23g (56 mg, 0.25 mmol), acrylate ester 24g (106 mg, 0.5 mmol), 1,4-ben- surface, preferential insertion of Zn into the sp halide (R–X) takes

zoquinone (27 mg, 0.25 mmol), AgNO3 (85 mg, 0.5 mmol), and [Pd(MeCN)4](BF4)2 (11 place in a likely successive one-electron-transfer sequence. The mg, 0.025 mmol) were sequentially added in air to a reaction tube equipped with a stir resulting water-sensitive RZnX is insulated from the surrounding bar and a septum. An aqueous solution containing PTS (1.0 mL, 2 wt %) was added by syringe and the resulting mixture vigorously stirred for 20 h. The contents of the flask water. RZnX is also stabilized by the TMEDA present, which may 2 were then quenched with aqueous NaHCO3 and extracted with EtOAc. The ethyl acetate assist in shuttling it into the micelle where both the Pd catalyst and sp , extracts were combined and filtered through a plug of silica gel and anhydrous MgSO4 halide await, in relatively high concentrations. If the rate of formation and then concentrated by rotary evaporation. The residue was purified by flash chro- matography, eluting with hexane–EtOAc to afford analytically pure 25g (86 mg, 80%). of RZnX is too fast for subsequent passage into the hydrophobic + HRMS (ESI) m/z calcd for C25H39NO5Na (M + Na ) 456.2726, found 456.2721 (Δ = 1.1 ppm). micellar core, the reagent escapes and is rapidly quenched by water, eq 12 (Ref. 45) as expected. Thus, while electron transfer en route to RZnX can be controlled for both 1° and 2° alkyl halides, reactions involving 3° precursors are far too rapid; thus, only quenched material (R–H) 50 PdCl2(amphos)2 (2 mol %) results. Zn, TMEDA Ar–Br + R–I Ar R The corresponding couplings with alkenyl iodides and bromides, PTS–H2O (2 wt %) rt, 24–48 h rather than aryl bromides, also give the anticipated products with retention of stereochemical integrity (eq 14).51 With both types of O n-C7H15 O 4 Me educts (aryl and alkenyl), the choice of catalyst is absolutely crucial: n-C H Me N 10 21 52 EtO2C PdCl2(amphos)2 (9) is the only species screened to date that affords Boc Cl Me OMOM high levels of conversion and, thus, good isolated yields. Even the 65% 82% 74% parent species; i.e., the bis(des-dimethylamino) analogue, was not nearly as effective. (Note that the nomenclature for “amphos” as used Ph OTBS Cl n-C8H17 for this ligand52 is seemingly inconsistent with prior literature.53) N N MeO N TIPS Boc 86% 93% 85% 4. New Insights into Micellar Catalysis for Organic Synthesis CO Et CO Et n-C H Ph 2 2 7 15 The observation that several of these Pd-catalyzed couplings not only O can be run in seawater,54 but actually take place in this medium at a O S OHC 62% 74% 80% greater rate than in HPLC grade water, has led to some very exciting developments of potential practical value. The presence of salts in eq 13 (Ref. 50a) the water can influence the chemistry in two significant ways:55 (1) the size and shape of the nanoparticles in which these couplings take 9 VOL. 45, NO. 1 • 2012 place are altered, and (2) the pH of the aqueous medium which can impact the nature of the catalysts involved. With NaCl, a “salting out” 56 R1 R1 effect exists, which removes water from the PEGylated portions of PdCl2(amphos)2 (2 mol %) X + R2–I R2 the particles, tending to increase micellar size and/or modify particle R Zn, TMEDA R shape as the PEG expands into the water. This effect is due to the anion, X = I, Br PTS–H2O (2 wt %) rt, 12–48 h and several salts examined lead to similar particle changes, although the corresponding coupling chemistry in the presence of each salt has Entry SM R R1 R2 X Yield E:Z yet to be examined. Particle shape can also be dramatically altered. 1 Z n-Hex H TMSCH I 95% 1:99 These effects can be shown by cryo-TEM analysis, e.g., on PTS with 2 2 Z Cl(CH2)4 H BnCH2 I 85% 3:97 54 and without NaCl (Figure 7). The “salting in” effect, which tends 3 E TMS H EtO2C(CH2)3 Br 83% 96:4 to decrease micellar size, is observed, e.g., with iodides NaI and KI.57 4 Ph Me Cy Br 66% ---- Changes in pH resulting from addition of small amounts of a salt 5 E BnO(CH2)3 H n-Hex I 85% >99:1 6 E Ph H n-Hex Br 92% 91:9 a such as KHSO4 can impact couplings due to the dynamic nature of 7 Z n-Hex H EtO2C(CH2)3 Br 74% 4:96 micelles; i.e., they are constantly in flux.23 Their amphiphiles traverse a EtO C(CH ) Br used. a sea of surrounding water for the exchange phenomenon to occur. 2 2 3 For catalysts that contain phosphines, e.g., ruthenium carbenes, as used routinely in olefin metathesis,58 their phosphine ligands can be Representative Pd-catalyzed, Zn-mediated coupling in PTS–H2O. Preparation of protonated upon exposure to aqueous acid,59 and hence, may arrive Ethyl (Z)-5-dodecenoate (entry 7).51 In a 5-mL, round-bottom flask under argon con- (amphos) (9; 7 mg, 0.01 mmol) was at the micelle as a coordinatively unsaturated species (and thus, quite taining zinc powder (260 mg, 4 mmol) and PdCl2 2 added a solution of 2 wt % of PTS (2 mL). N,N,N′,N′-Tetramethylethylenediamine (TMEDA) “hot”). These phenomena are illustrated below by Heck couplings and (232 mg, 2 mmol), was added at rt followed by addition of (Z)-1-bromooctene (181 mg, olefin metathesis reactions. 1 mmol; Z:E = 99:1) and ethyl 4-bromobutanoate (390 mg, 2 mmol). The flask was stirred vigorously at rt for 48 h. The product was extracted with EtOAc. Silica gel (1 g) was added to the combined organic phase and the solvents were removed under reduced pressure.

4.1. Heck Coupling in PTS–3 M NaCl (aq) The resulting dry, crude product mixed with SiO2 was introduced on top of a silica gel chromatography column which was eluted with 5% EtOAc–petroleum ether, affording Heck reactions in micellar PTS–H2O are especially responsive to 54 the analytically pure product (167 mg, 74%; Z:E = 96:4); HRMS (ESI) m/z calcd for C14H26O2 changes in the ionic strength of the medium. This effect is not to (M+) 226.1933, found 226.1934 (Δ = 0.4 ppm). be expected in related reactions that, albeit conducted in water, rely on alternative phenomena. For example, supported catalysts such as eq 14 (Ref. 51)

50 nm A 100 nm B

50 nm C

Figure 7. Cryo-TEM Image of (A) Aqueous PTS (50-nm Scale), (B) Aqueous Figure 6. Pictorial Account of Couplings of RZnX in Water: Timing Is Key. PTS in the Presence of 3 M NaCl (100-nm Scale), and (C) TPGS-750-M. (Ref. 54) “Designer”-Surfactant-Enabled Cross-Couplings in Water at Room Temperature 10 Bruce H. Lipshutz* and Subir Ghorai

Pd(OAc)2 mounted on a reverse-phase N,N-diethylaminopropylated

(NDEAP) silica gel impregnated with ionic liquid [bmim]PF6 have 60 NDEAP–Pd been used (Scheme 1). Other advances include applications of new O I (5 mol %) CO2Cy 61 + Pd-complexed cationic bipyridyl ligands (L-1), and new P–N ligands OCy 62 MeO (n-Bu)3N (2 equiv) MeO in the form of diazadiphosphetidines (L-2). In each case involving o H2O, 100 C, 24 h 94% an acrylate partner, while water is the reaction solvent, heating is PdCl (NH ) –L-1 needed to improve conversion. O 2 3 2 I (0.01 mol %) CO2Et + Two examples of reactions subject to the influence of salts under OEt NC (n-Bu)3N (2 equiv) NC micellar conditions are worth noting. In the first, cinnamate 26 is o H2O, 140 C, 12 h 80% formed in PTS–water to a limited extent after 14 hours (Scheme 2, top 54 PdCl2 (2.5 mol %) reaction). However, in the presence of NaCl (3 M) and at an identical O Br L-2 (40 mol %) CO2(n-Bu) + global concentration (0.50 M), the reaction reaches full conversion O(n-Bu) o Ac H2O, 100 C, 6 h Ac in the same time period, with an associated high yield. In the second 70% + example, the formation of 27 requires mild heating to 50 °C to reach Me3N PhHN NEt2 Ph completion after 8 hours; in PTS–3 M aqueous NaCl, however, the OH Br– P N Ph Si Si N P reaction was complete at room temperature in 3 hours (Scheme 2, O O OMe Ph N 54 O O Ph bottom). N P N + N silica bed P N NMe3 Ph NHPh Br– 4.2. Olefin Metathesis at pH 2–3 NDEAP L-1 L-2 In 2006, Hong and Grubbs reported on ring-opening polymerization (ROMP) reactions in aqueous acid, and showed that the 2nd-generation Grubbs catalyst has its phosphine sequestered via protonation.59 Scheme 1. Recent Examples of the Heck Coupling in Water. (Ref. 60–62) The same protonation is presumably responsible for accelerating cross-metathesis under micellar conditions in water, most visibly in reactions involving tough type-2 olefins such as methyl vinyl ketone (MVK) and acrylonitrile (eq 15).54 For related ring-closing reactions,

e.g., forming a 7-membered ring, the net effect of added KHSO4 was O equivalent to that seen using 3 M NaCl. H C=CHCO t-Bu (2 equiv) MeO Br 2 2 MeO 8 (2 mol %), Et3N (3 equiv) Ot-Bu

PTS–H2O (5 wt %), rt, 14 h 5. Designing a Better Micelle: TPGS-750-M, a 26 2nd-Generation Amphiphile 18% (98% w. 3 M NaCl) Notwithstanding the progress made utilizing the first-generation O designer surfactant PTS, the many lessons learned insofar as altering I 2-ethylhexyl acrylate (2 equiv) n-Bu the nature of the surfactant to maximize reaction rates and reagent 8 (2 mol %), Et3N (3 equiv) O RO Et or catalyst stability encouraged the design of a second-generation PTS–H2O (5 wt %) RO 27 63 Me Me amphiphile, TPGS-750-M. Experience had shown that micelle size 94% (50 oC, 8 h) R = and shape matter; that improvements in rate in going from TPGS- Me 96% (w. 3 M NaCl; rt, 3 h) 2 1000 to PTS, where particle size increases from 13 to (on average) 25 nm,54 seems to be an important clue. Since differences between the two were best visualized by cryo-TEM (see Figure 3), showing the Scheme 2. Two Examples of the Influence of Added Salt on the Outcome of longer rods or worms present only with PTS, TPGS-750-M (Figure the Heck Coupling in PTS–H2O. (Ref. 54) 7C) was engineered to have a higher percentage of larger particles, preferably in the 50–100-nm range. A shorter PEG chain requiring less volume in its coiled state, and hence micelles able to accommodate more molecules per particle, should expand the particle’s radius. A structural comparison between TPGS, PTS, and TPGS-750-M is Grubbs-2 (2 mol %) R R illustrated in Figure 8. + R' R' PTS–H2O, rt, 4 h From the standpoint of synthesis, a switch to four-carbon succinic 13 Me acid as found in TPGS-1000, rather than the ten-carbon sebacic acid CN Ts N as in PTS, makes a huge difference in overall efficiency of the two- O OTBS OTBS step sequence (Scheme 3). That is, opening of succinic anhydride by PTS (12 h): 74% PTS: 20% PTS–3 M NaCl: 90% vitamin E allows for a virtually quantitative esterification involving PTS–KHSO : 91% PTS–KHSO : 30% PTS–KHSO : 92% 4 4 4 the most expensive component in the process. By contrast, PTS relies Mes N N Mes on sebacoyl chloride, a diacid chloride that reacts at both termini Grubbs-2 = Cl with α-tocopherol, thereby affording mixtures of the mono- and the Ru Cl Ph diester.8 Conversion of monoester 28 to TPGS-750-M via traditional PCy3 esterification can be smoothly done in toluene in the presence of catalytic acid (TsOH), another close to quantitative event.63 Here, eq 15 (Ref. 54) use of a mono-methylated polyethylene glycol, or MPEG, rather than PEG (a diol) is the key to avoiding reactions at both ends. This is 11 VOL. 45, NO. 1 • 2012 yet another problem in the route to PTS, which uses PEG-600 (and not an MPEG analogue). The ‘net-net’ of these changes is that the targeted amphiphile can be prepared in an overall yield that exceeds succinic acid 95%, while the efficiency of making PTS is ca. 45%. Me O The greatly improved cost implies that couplings enabled by O DL- -tocopherol α Et3N, PhMe Me OH TPGS-750-M–H2O need only be as good as those done in PTS. Of + Me o O course, faster reactions might also be anticipated due to larger particle succinic anhydride 60 C, 5 h Me O Me 3 sizes. By virtue of increases in binding constants of the substrates Me and catalysts within the micellar environment (i.e., longer time spent racemic vitamin E within a micelle), reactions should reach higher states of conversion 28, 99% more rapidly.64 What has been found in this regard is that most types of cross-couplings in TPGS-750-M do, in fact, lead to isolated yields TPGS-750-M PEG-750-M, TsOH (cat) 98% PhMe, reflux, 5 h that are equal to, or better than, those seen in aqueous PTS. Some direct comparisons are illustrated in Scheme 4, with each pair of reactions being run at the identical concentration and time frame. Likewise gold-catalyzed cycloisomerizations of allenols take place Scheme 3. Straightforward, High-Yield Synthesis of TPGS-750-M. (Ref. 63) readily in both aqueous PTS and TPGS-750-M, where the presence of NaCl was found to shorten reaction times (eq 16).65 In a number of cases, yields were better with the second-generation surfactant, although some substrate dependence was noted.

5.1. CuH-Catalyzed Asymmetric Hydrosilylation Heck Suzuki–Miyaura Sonogashira

The in situ generation and use of copper hydride, derived from I i-Pr B(OH)2 precursors such as CuCl–NaOt-Bu, CuF , or Cu(OAc) , routinely OMe Br 2 2 + i-Pr Br + + 66 takes place in organic media: toluene, THF, etc. Although the Me Me reagent is stable to water and, in fact, was used long ago by Stryker i-Pr OMe to enhance the rate of quenching of copper(I) intermediates (e.g., 67 Me i-Pr enolates resulting from 1,4 addition of hydride from (Ph3P)CuH, its use in a strictly aqueous medium is not among the available protocols. i-Pr OMe 2-Np Me It might even be argued that water as “solvent” presents a likely major OMe i-Pr PTS: 92% 76% 84% TPGS-750-M: 95% 88% 99%

succinic Amination Negishi-like acid Me TPGS (TPGS-1000) Me Br NH2 Br Me O ++Me Br CO2Et O O Me 4 Me O H Me n O Me O Me (n = ca. 23) Me 3 H Me Me N PEG-1000 Me CO2Et 4 nonracemic vitamin E Me PTS: 81% 74% (E:Z = 96:4) sebacic TPGS-750-M: 93% 87% (E:Z = 99:1) PTS (PTS-600) acid (7) Me O O O Me O H Me 4 O 14 Me O Me Scheme 4. Cross-Couplings in PTS–H2O and in TPGS-750-M–H2O. (Ref. 63) 3 Me PEG-600

racemic vitamin E

succinic acid TPGS-750-M Me Me Me Me O Me O AuBr3 (1 mol %) Me O • OTBS Me OTBS Me O Me Me surfactant–H O O OH 2 H O 17 (2 wt %) Me O Me H 3 NaCl (3 M), rt PTS: 86% (30 min) Me MPEG-750 TPGS-750-M: 92% (10 min)

racemic vitamin E

Figure 8. Structures of TPGS, PTS, and the Newly Engineered TPGS-750-M. (Ref. 63) eq 16 (Ref. 65) “Designer”-Surfactant-Enabled Cross-Couplings in Water at Room Temperature 12 Bruce H. Lipshutz* and Subir Ghorai

limitation, since the literature on these conjugate reductions clearly suggests that lower reaction temperatures tend to maximize ee’s.68

t-Bu Me Thus, while eliminating organic solvents from these reactions would O OMe impart an element of “greenness”, the tradeoff in substrate insolubility and potentially lower ee’s might not favor a “green” approach. On the O 2 P t-Bu MeO P Me 2 other hand, such reactions being run at high concentrations within P t-Bu MeO P Me O micelles at room temperature may not follow the same rules, in which O OMe 2 case the use of organic solvent and consumption of energy for cooling t-Bu 2 Me would both be averted. (R)-DTBM-SEGPHOS® (29) (R)-3,5-Xyl-MeO-BIPHEP (30) It came with some surprise, therefore, that treatment of isophorone Takasago (Ref. 69) Roche (Ref. 73) with CuH ligated by Takasago’s (R)-DTBM-SEGPHOS® (29)69 Me Me (Figure 9) in 2 wt % PTS–H O at room temperature afforded the H H 2 product of 1,4 addition in high yield and in 99% ee (eq 17).70 This P(t-Bu)2 PCy2 Fe PPh2 Fe PPh2 result is comparable to the best that has been seen to date, albeit done in toluene at low temperatures.71 Unfortunately, this conversion

(R,S)-PPF-P(tBu)2 (31) JosiPhos (32) required far too much PhSiH3 (12 equiv, or 36 equiv of hydride), Solvias (Ref. 74) Solvias (Ref. 74) as this reagent is competitively decomposed in aqueous solution. Eventually, the best conditions identified focused on the use of polymethylhydrosiloxane (PMHS)72 as hydride donor (⅓ equiv, 9 Figure 9. Nonracemic Ligands Used in CuH-Catalyzed Asymmetric equiv of hydride) in 2 wt % TPGS-750-M, giving the same product Hydrosilylation. in 94% ee. By way of comparison, the identical reaction run in water only gave 18% conversion after the same 18-h reaction time. Other ligands, including Roche’s 3,5-Xyl-MeO-BIPHEP (30)73 and Solvias’s JosiPhos ligands (31 and 32)74 are also effective, depending 70 O O upon the nature of the substrate. Particularly interesting is enone Cu(OAc)2•H2O (3 mol %) ligand (3 mol %), silane 34, where in combination with ligand 31 and at –78 °C in toluene, 71 Me 2 wt % surfactant Me * an 87% ee was reported. The identical reaction run in water at Me Me Me H2O, rt, 18 h Me room temperature gave a comparable yield; however, the ee was isophorone 33 higher: 93% (eq 18).70a Likewise, butyrolactone 35 reacted at room

Ligand Silane Hydride Surfactant Conv. (Yield) ee temperature and led to a similar yield and ee relative to that seen earlier, where the reaction was run at 0 °C. That ee’s can be obtained 29 PhSiH3 36 equiv PTS 100% (89%) 99% 30 PMHS 9 equiv TPGS-750-M 100% (88%) 94% that are on par with, or even exceed, those normally realized at 30 PMHS 9 equiv none 18% nd low temperatures may be a consequence of restricted reagent and/ or substrate movement within tightly packed micellar arrays, where energetic differences between diastereomeric transition states are eq 17 (Ref. 70) accentuated, and less favorable orientations are minimized. The nature of the micelle interior also plays a crucial role, as documented by these asymmetric hydrosilylations run under otherwise identical conditions using alternative amphiphiles (Brij® 30, Cremophor®, SDS, and Solutol®)–water solutions (eq 19).70 Clearly, the newly designed Cu(OAc)2•H2O (3 mol %) TPGS-750-M affords the most effective medium for this type of EWG PMHS (0.31 equiv) EWG ∗∗ transformation. That high ee’s can be realized at room temperature conditions (a) or (b) R R' R R' within a micellar array without normal recourse to low temperatures

Conditions: (a) ligand (3 mol %), TPGS-750-M–H2O (2 wt %), is particularly striking. rt, 18 h; (b) ligand, PhMe. O O Me O 5.2. Borylation of Aryl Halides O OEt n-Bu Ph Aryl boronates, in particular those derived from pinacol using Ph Ph Me bis(pinacolato)diboron (B2pin2), figure prominently as coupling 34 35 partners in Suzuki–Miyaura reactions.75 Aryl bromides are common (a) 31; 80%, 93% ee (a) 30; 89%, 98% ee (a) 30; 94%, 95% ee (b) 31; –78 oC, 87% ee (b) 30; 0 oC, 99% ee (b) 30; rt, 91% ee educts, although borylations in organic media (e.g., dioxane, DMSO, or THF) at room temperature are very rare.76 Couplings “on water” do O O CN lead to product, but to a limited and unpredictable extent. However, OEt OEt N Ph in water containing 2–3 wt % TPGS-750-M, aryl pinacolatoboranes Me Ph (36) could be smoothly prepared usually within three hours at a global S Cl 77 concentration of 0.25 M (eq 20). A Pd(0) catalyst, Pd(Pt-Bu3)2 (8, 3 (a) 32; 87%, 86% ee (a) 30; 87%, 88% ee (a) 30; 87%, 94% ee mol %), afforded the highest yields over several alternatives examined

(e.g., Pd(OAc)2–XPhos, PdCl2(dppf), Pd 2(dba)3–2PCy3, etc.). Given that

palladium is already present in the form of Pd(Pt-Bu3)2, as well as its eq 18 (Ref. 70a) known use in Suzuki–Miyaura couplings,78 introduction of a second aryl bromide into the reaction mixture ultimately leads to biaryl 37 13 VOL. 45, NO. 1 • 2012

(eq 21). Compound 37 is formally the net cross-coupling product of 79 two aryl bromides, achieved in one pot in water at room temperature. O O Cu(OAc)2•H2O (3 mol %) 30 (3 mol %), PMHS 6. Summary and Outlook Me surfactant–H O, 18 h, rt Me * Me 2 Me Surfactants have an especially rich history of service to many areas Me Me of organic chemistry.3 But most amphiphiles listed in catalogs isophorone 33 perused by organic chemists today are from decades ago; they are Surfactant Conv. ee rarely “green”, nor are they matched in any way to the nanoscale ® environments that maximize the quality of transition-metal-catalyzed Brij 35 20% 92% ® reactions. However, starting to appear are newly engineered Brij 30 50% 81% Cremophor® EL 100% 86% “designer” surfactants that better accommodate reaction partners, SDS 6% 95% additives, and catalysts characteristic of modern organic synthesis. Solutol® HS15 21% 92% In the interest of sustainability, the goal in designing new surfactants TPGS-750-M 100% 94% for synthesis is to develop reproducible and scalable processes that minimize involvement of organic solvents, at least from the eq 19 (Ref. 70) standpoint of the reaction medium. Historically, however, “surfactant science & technology as pursued in academia has not overlapped well with the mindset of industrial chemists in this area”.80 That is, while 8 (3 mol %) academicians tend to strive for purified, homogeneous materials that Br KOAc (3 equiv) are readily subject to analytical techniques (e.g., surfactants such R + B2Pin2 R O Me ® TPGS-750-M–H2O B as Triton X-100 and SDS), industrial researchers oftentimes are 1.1 equiv (2 wt %), room temp O Me faced with a “Make it work and don’t worry about why!” approach. Me Me As author Drew Meyers unabashedly continues in his monograph,80 Ar–B(Pin), 36 “The sad fact of life is that real surfactant systems are almost always composed of mixed chemical isomers, contaminants, and added MeS H2N Me Me materials that can alter the effects of a given surfactant on a system…” B(Pin) B(Pin) B(Pin) This is precisely the current state of affairs surrounding PTS, since, in Me fact, it is a mix of many components: PEG-600 contains a broad range 3 h, 80% 24 h, 85%9 h, 93% of polyethylene units, and the many byproducts from its preparation, O OTBS Ac as discussed earlier, are likely to be present in varying amounts. Its EtO chromatographic purification using peak-shaving techniques led to B(Pin) B(Pin) the unexpected observation that, remarkably, PTS of >95% purity is 6 h, 77% 9 h, 82% actually insoluble in water!81 The switch to TPGS-750-M, therefore, provides an amphiphile eq 20 (Ref. 77) that avoids many of these contaminants by virtue of its design. While remaining benign, it offers the community a better economic profile, along with enhanced reaction rates for the cross-couplings taking place within its nanomicelles. As with PTS, it also usually leads OMe Me Me to a very attractive impurity profile, given the room temperature 1. 8 (3 mol %), B2Pin2, KOAc, 3 h Br TPGS-750-M–H O (2 wt %) conditions for the vast majority of reactions in this aqueous medium. 2 2. 4-MeOC H Br, KOH, 3 h For the intended industrial uses, time in the kettle (i.e., throughput) 6 4 (all in one pot) 37, 80% is another virtue, since there is no time (or energy) investment due to heating and/or cooling of these coupling reactions. While this eq 21 (Ref. 77) second-generation surfactant has yet to be fully evaluated, it would be naïve to assume that future generations of designed nanomicelles that offer even better matches between reaction ingredients and micellar interiors will not be forthcoming. β-sitosterol Part of the insight yet to be gained will come from just hard work; making and testing surfactants that address variables that Me Me succinic may provide additional clues as to how to improve metal-catalyzed H Me Me acid couplings. For example, the surfactant “Nok” (Figure 10) is currently Me H Me O being studied as an analogue of TPGS-750-M; it relies on a “healthy” H H O Me phytosterol, β-sitosterol, in keeping with the theme that any newly O O n O created surfactant should pose no potential environmental insult (n = ca. 13) regardless of scale of usage. The point to be tested, however, is MPEG-550 whether in providing a hydrophobic interior as solvent composed of a "Nok" (SPGS-550-M) cyclic hydrocarbon, as opposed to the linear hydrocarbon found in the vitamin E portion of TPGS-750-M, along with the anticipated control of size in the 50–100-nm range, there are further improvements in Figure 10. Structure of “Nok” (SPGS-550-M) Currently under Study. any or all of the cross-coupling chemistry of interest. Moreover, let’s “Designer”-Surfactant-Enabled Cross-Couplings in Water at Room Temperature 14 Bruce H. Lipshutz* and Subir Ghorai

appreciate that micellar catalysis, by virtue of synthetic design and Surfactant Science Series 125; Taylor & Francis: Boca Raton, FL, manipulation, de facto offers a virtually unlimited array of potential 2005. (b) Applied Surfactants: Principles and Applications; Tadros, reaction media tailored to best match a given transformation, each to T. F., Ed.; Wiley-VCH: Weinheim, Germany, 2005. be used catalytically as a nanoreactor.82 By contrast, consider just how (4) En gberts, J. B. F. N. Organic Chemistry in Water: Green and Fast. few the choices of traditional organic solvents there really are. In Methods and Reagents for Green Chemistry: An Introduction; New discoveries of potential major consequence in synthesis that Tundo, P., Perosa, A., Zecchini, F., Eds.; Wiley: Hoboken, NJ, 2007; take advantage of the hydrophobic effect await us; two of these have Chapter 7, pp 159–170. been discussed herein: organozinc-mediated cross-couplings in water (5) Solvents and Solvent Effects in Organic Chemistry; Reichardt, C., (see Section 3.5),50,51 and unexpectedly high ee’s from asymmetric Welton, T., Eds.; Wiley-VCH: Weinheim, Germany, 2011. hydrosilylations with CuH at room temperature (see Section 5.1).70 (6) h ttp://www.everythingbio.com/glos/definition.php?word= More are on the way; e.g., homogeneous catalytic organocopper hydrophobic+effect (accessed Jan 2012). chemistry that forms C–C bonds in water at room temperature.83 (7) (a ) Scarso, A.; Strukul, G. Adv. Synth. Catal. 2005, 347, 1227. (b) This latter discovery is seemingly difficult to accept, given the many Selke, R.; Holz, J.; Riepe, A.; Bömer, A. Chem.—Eur. J. 1998, 4, reviews on copper chemistry written by one of the authors. This is 769. (c) Scarso, A. Chimica e l’Industria 2009, 91, 142. especially true in the Schlosser Manuals,84 advising readers to be (8) Lipshutz, B. H.; Ghorai, S. Aldrichimica Acta 2008, 41, 59. extremely cautious about the sensitivity of carbon-based copper (9) (a) Borowy-Borowski, H.; Sikorska-Walker, M.; Walker, P. R. Water- reagents to moisture! One interpretation of these data could be that Soluble Compositions of Bioactive Lipophilic Compounds. U.S. the “rules” for doing chemistry at high concentrations in nanoparticles Patent 6,045,826, Apr 4, 2000. (b) Borowy-Borowski, H.; Sikorska- may well be different from those accumulated over the past 40–50 Walker, M.; Walker, P. R. Water-Soluble Compositions of Bioactive years of modern organic synthesis. This notion was foreshadowed Lipophilic Compounds. U.S. Patent 6,191,172, Feb 20, 2001. (c) by Lindström and Andersson in their Angewandte Chemie article Borowy-Borowski, H.; Sikorska-Walker, M.; Walker, P. R. Water- “Hydrophobicity Directed Organic Synthesis”,85 in which these Soluble Compositions of Bioactive Lipophilic Compounds. U.S. authors suggest that, “The design of stereoselective reactions based on Patent 6,632,443, Oct 14, 2003. hydrophobic interactions is an area of great potential that is still largely (10) ( a) Green Chemical Syntheses and Processes; Anastas, P. T., Heine, unexplored.” After all, Nature does not do chemistry by matching L. G., Williamson T. C., Eds.; ACS Symposium Series 767; American substrates and catalysts to organic solvents as we know them. Nature’s Chemical Society: Washington, DC, 2000. (b) Benign by Design: macroscopic medium is water, which is used either as a legitimate Alternative Synthetic Design for Pollution Prevention; Anastas, solvent, or to force molecular organization to create hydrophobic P. T., Farris, C. A., Eds.; ACS Symposium Series 557; American pockets. Isn’t that precisely what we call “micellar catalysis”? Chemical Society: Washington, DC, 1994. (11) U. S. Food and Drug Administration. Notice No. GRN 000202. 7. Acknowledgements http://www.accessdata.fda.gov/scripts/fcn/fcnNavigation.cfm?rpt=g The chemistry discussed herein has been the culmination of tremendous rasListing&displayAll=false&page=5 (accessed Jan 2012). efforts by the PI’s graduate students (Ben Taft, Alex Abela, David (12) For more information on Brij® 30 (CAS Registry Number® 9002-92- Chung, Ralph Moser, Karl Voigtritter, Shenlin Huang, and Zarko 0), see Satkowski, W. B.; Huang, S. K.; Liss, R. L. Polyoxyethylene Boskovic), postdoctoral students (Subir Ghorai, Takashi Nishikata, Alcohols. In Nonionic Surfactants; Schick, M. J., Ed.; Surfactant Christophe Duplais, and Arkady Krasovskiy), and group associates Science Series, Vol. 23; Dekker: New York, 1967; pp 86–141. (Wendy Leong, Isabelle Thome, Julian Graff, Paul Konopelski, and (13) Ca wley, J. D.; Stern, M. H. Water-Soluble Tocopherol Derivatives. Valeria Krasovskaya). These co-workers deserve the credit for the U.S. Patent 2,680,749, June 8, 1954. progress we have made in the past few years since our 2008 review (14) Bo rkovec, M. Measuring Particle Size by Light Scattering. In in a previous Aldrichimica Acta issue.8 We are also most appreciative Handbook of Applied Surface and Colloid Chemistry; Holmberg, K., of the support extended by the NIH (GM 86485), and the especially Ed.; Wiley: Chichester, U.K., 2002; Vol. 2, pp 357–370. generous gifts of both ligands and catalysts used in our studies by Dr. (15) Vriezema, D. M.; Aragonès, M. C.; Elemans, J. A. A. W.; Cornelissen, Tom Colacot (Johnson Matthey); Dr. Richard Pedersen (Materia); Ms. J. J. L. M.; Rowan, A. E.; Nolte, R. J. M. Chem. Rev. 2005, 105, 1445. Astrid Metzger (Umicore AG & Co.); Drs. Takao Saito and Hideo (16) Yoshizawa, M.; Klosterman, J. K.; Fujita, M. Angew. Chem., Int. Ed. Shimizu, and Mr. Izuru Nagasaki (Takasago); Dr. Benoit Pugin 2009, 48, 3418. (Solvias); and Drs. Rudolf Schmid and Michelangelo Scalone (Roche). (17) Lipshutz, B. H.; Taft, B. R. Org. Lett. 2008, 10, 1329. The author also warmly acknowledges the intellectual contributions (18) Lip shutz, B. H.; Petersen, T. B.; Abela, A. R. Org. Lett. 2008, 10, of Dr. Volker Berl (Mycell Technologies) toward the creation of 1333. TPGS-750-M, and Prof. Norbert Krause, along with several past and (19) Lipshutz, B. H.; Chung, D. W.; Rich, B. Org. Lett. 2008, 10, 3793. present group members for their comments on this manuscript prior (20) Li pshutz, B. H.; Aguinaldo, G. T.; Ghorai, S.; Voigtritter, K. Org. to publication. Lett. 2008, 10, 1325. (21) Lipshutz, B. H.; Ghorai, S.; Aguinaldo, G. T. Adv. Synth. Catal. 8. References and Notes 2008, 350, 953. (1) h ttp://en.wikipedia.org/wiki/Deepwater_Horizon_oil_spill (22) Jessop, P. G. Green Chem. 2011, 13, 1391. (accessed Jan 2012). (23) Surfactants and Polymers in Drug Delivery; Malmsten, M., Ed.; (2) (a) http://bpoilspillcrisisinthegulf.webs.com/corexit.htm (accessed Drugs and the Pharmaceutical Sciences Series, Vol. 122; Marcel Jan 2012). (b) http://www.nalco.com/news-and-events/4297.htm Dekker: New York, 2002; p 27. (accessed Jan 2012). (24) Bha ratiya, B.; Ghosh, G.; Bahadur, P.; Mata, J. J. Disp. Sci. Tech. (3) (a) Dynamics of Surfactant Self-Assemblies: Micelles, 2008, 29, 696. Microemulsions, Vesicles, and Lyotropic Phases; Zana, R., Ed.; (25) Lipshutz, B. H.; Chung, D. W.; Rich, B. Adv. Synth. Catal. 2009, 351, 1717. 15 VOL. 45, NO. 1 • 2012

(26) (a) Hartwig, J. F. In Handbook of Organopalladium Chemistry for (45) Nishikata, T.; Lipshutz, B. H. Org. Lett. 2010, 12, 1972. Organic Synthesis; Negishi, E., Ed.; Wiley: New York, 2002; Vol. 1, (46) Tetrakis(acetonitrile)palladium(II) tetrafluoroborate: ® pp 1051–1096. (b) Louie, J.; Hartwig, J. F. Tetrahedron Lett. 1995, [Pd(MeCN)4](BF4)2, CAS Registry Number 21797-13-7. 36, 3609. (47) (a ) Negishi, E. In Handbook of Organopalladium Chemistry for (27) Potassium trimethylsilanolate: CAS Registry Number® 10519-96-7. Organic Synthesis; Negishi, E., Ed.; Wiley-Interscience: New York, (28) Ni shikata, T.; Lipshutz, B. H. Chem. Commun. 2009, 6472. 2002; Vol. 1, p 243. (b) Negishi, E.; Gagneur, S. In Handbook of (29) (a ) Lu, Z.; Ma, S. Angew. Chem., Int. Ed. 2008, 47, 258 and references Organopalladium Chemistry for Organic Synthesis; Negishi, E., therein. (b) Weissman, S. A.; Zewge, D. Tetrahedron 2005, 61, 7833. (c) Ed.; Wiley-Interscience: New York, 2002; Vol. 1, p 597. Wuts, P. G. M.; Greene, T. W. Greene’s Protective Groups in Organic (48) (a) Knoess, H. P.; Furlong, M. T.; Rozema, M. J.; Knochel, P. J. Org. Synthesis, 4th ed.; Wiley-Interscience: New York, 2007; p 16. (d) Chem. 1991, 56, 5974. (b) Manolikakes, G.; Dong, Z.; Mayr, H.; Li, Takahashi, K.; Miyake, A.; Hata, G. Bull. Chem. Soc. Jpn. 1972, 45, 230. J.; Knochel, P. Chem.—Eur. J. 2009, 15, 1324. (c) Manolikakes, G.; (30) (a) DPEPhos: CAS Registry Number® 166330-10-5. (b) Kranenburg, Schade, M. A.; Munoz Hernandez, C.; Mayr, H.; Knochel, P. Org. M.; van der Burgt, Y. E. M.; Kamer, P. C. J.; van Leeuwen, P. W. N. Lett. 2008, 10, 2765. (d) Manolikakes, G.; Munoz Hernandez, C.; M.; Goubitz, K.; Fraanje, J. Organometallics 1995, 14, 3081. Schade, M. A.; Metzger, A.; Knochel, P. J. Org. Chem. 2008, 73, (31) (a) XantPhos: CAS Registry Number® 161265-03-8. (b) Hillebrand, 8422. S.; Bruckmann, J.; Krüger, C.; Haenel, M. W. Tetrahedron Lett. (49) (a) Blackmond, D. G.; Armstrong, A.; Coombe, V.; Wells, A. Angew. 1995, 36, 75. Chem., Int. Ed. 2007, 46, 3798. (b) Narayan, S.; Muldoon, J.; Finn, (32) Nishikata, T.; Lipshutz, B. H. Org. Lett. 2009, 11, 2377. M. G.; Fokin, V. V.; Kolb, H. C.; Sharpless, K. B. Angew. Chem., Int. (33) (a) Muzart, J. Eur. J. Org. Chem. 2007, 3077 and references therein. Ed. 2005, 44, 3275. (c) Hayashi, Y. Angew. Chem., Int. Ed. 2006, (b) Tamaru, Y. Eur. J. Org. Chem. 2005, 2647. (c) Yang, S.-C.; Hsu, 45, 8103. (d) Breslow, R. Acc. Chem. Res. 1991, 24, 159. (e) Pihko, Y.-C.; Gan, K.-H. Tetrahedron 2006, 62, 3949. (d) Yokoyama, Y.; P. M.; Laurikainen, K. M.; Usano, A.; Nyberg, A. I.; Kaavi, J. A. Hikawa, H.; Mitsuhashi, M.; Uyama, A.; Hiroki, Y.; Murakami, Tetrahedron 2006, 62, 317. (f) Nyberg, A. I.; Usano, A.; Pihko, P. Y. Eur. J. Org. Chem. 2004, 1244. (e) Yokoyama, Y.; Takagi, N.; M. Synlett 2004, 1891. Hikawa, H.; Kaneko, S.; Tsubaki, N.; Okuno, H. Adv. Synth. Catal. (50) (a ) Krasovskiy, A.; Duplais, C.; Lipshutz, B. H. J. Am. Chem. 2007, 349, 662. Soc. 2009, 131, 15592. (b) Krasovskiy, A.; Thomé, I.; Graff, J.; (34) Lipshutz, B. H.; Abela, A. R. Org. Lett. 2008, 10, 5329. Krasovskaya, V.; Konopelski, P.; Duplais, C.; Lipshutz, B. H. (35) Billingsley, K.; Buchwald, S. L. J. Am. Chem. Soc. 2007, 129, 3358. Tetrahedron Lett. 2011, 52, 2203. (c) Duplais, C.; Krasovskiy, A.; (36) Polshettiwar, V.; Decottignies, A.; Len, C.; Fihri, A. ChemSusChem Lipshutz, B. H. Organometallics 2011, 30, 6090. 2010, 3, 502. (51) Krasovskiy, A.; Duplais, C.; Lipshutz, B. H. Org. Lett. 2010, 12, 4742. ® (37) Nishikata, T.; Lipshutz, B. H. J. Am. Chem. Soc. 2009, 131, 12103. (52) PdCl 2(amphos)2: CAS Registry Number 887919-35-9. Amphos = (38) (a ) Yamamoto, Y.; Takada, S.; Miyaura, N.; Iyama, T.; Tachikawa, 4-dimethylaminophenyldi(tert-butyl)phosphine. (a) Guram, A. S.; H. Organometallics 2009, 28, 152. (b) Yamamoto, Y.; Takada, S.; King, A. O.; Allen, J. G.; Wang, X.; Schenkel, L. B.; Chan, J.; Bunel, Miyaura, N. Chem. Lett. 2006, 35, 1368. E. E.; Faul, M. M.; Larsen, R. D.; Martinelli, M. J.; Reider, P. J. Org. (39) Moser, R.; Nishikata, T.; Lipshutz, B. H. Org. Lett. 2010, 12, 28. Lett. 2006, 8, 1787. (b) Guram, A. S.; Wang, X.; Bunel, E. E.; Faul, (40) (a ) Tsuji, Y.; Funato, M.; Ozawa, M.; Ogiyama, H.; Kajita, S.; M. M.; Larsen R. D.; Martinelli, M. J. J. Org. Chem. 2007, 72, 5104. Kawamura, T. J. Org. Chem. 1996, 61, 5779. (b) Tsuji, Y.; Kajita, S.; (53) De Vasher, R. B.; Moore, L. R.; Shaughnessy, K. H. J. Org. Chem. Isobe, S.; Funato, M. J. Org. Chem. 1993, 58, 3607. 2004, 69, 7919. (41) (a) Colby, D. A.; Bergman, R. G.; Ellman, J. A. Chem. Rev. 2010, 110, (54) Lip shutz, B. H.; Ghorai, S.; Leong, W. W. Y.; Taft, B. R.; Krogstad, 624. (b) Ackermann, L.; Vicente, R.; Kapdi, A. R. Angew. Chem., D. V. J. Org. Chem. 2011, 76, 5061. Int. Ed. 2009, 48, 9792. (c) Daugulis, O.; Do, H.-Q.; Shabashov, D. (55) S chott, H. J. Colloid Interface Sci. 1995, 173, 265. Acc. Chem. Res. 2009, 42, 1074. (d) Chen, X.; Engle, K. M.; Wang, (56) Anton, N.; Saulnier, P.; Béduneau, A.; Benoit, J.-P. J. Phys. Chem. B D.-H.; Yu, J.-Q. Angew. Chem., Int. Ed. 2009, 48, 5094. (e) Li, C.-J. 2007, 111, 3651 and references therein. Acc. Chem. Res. 2009, 42, 335. (f) Kakiuchi, F.; Kochi, T. Synthesis (57) Nishikido, N.; Matuura, R. Bull. Chem. Soc. Jpn. 1977, 50, 1690. 2008, 3013. (58) (a) Grubbs, R. H.; Chang, S. Tetrahedron 1998, 54, 4413. (b) Fürstner, (42) Nishikata, T.; Abela, A. R.; Lipshutz, B. H. Angew. Chem., Int. Ed. A. Angew. Chem., Int. Ed. 2000, 39, 3012. (c) Trnka, T. M.; Grubbs, 2010, 49, 781. R. H. Acc. Chem. Res. 2001, 34, 18. (d) Schrock, R. R.; Hoveyda, (43) Fo r applications and properties of BASF®’s Solutol® HS 15 (CAS A. H. Angew. Chem., Int. Ed. 2003, 42, 4592. (e) Connon, S. J.; Registry Number® 70142-34-6), see http://worldaccount.basf. Blechert, S. Angew. Chem., Int. Ed. 2003, 42, 1900. (f) Handbook of com/wa/NAFTA/Catalog/Pharma/info/BASF/exact/solutol_hs_15 Metathesis; Grubbs, R. H., Ed.; Wiley-VCH: Weinheim, Germany, (accessed Jan 2012). 2003 (3-volume set). (g) Deiters, A.; Martin, S. F. Chem. Rev. 2004, (44) (a ) Join, B.; Yamamoto, T.; Itami, K. Angew. Chem., Int. Ed. 2009, 104, 2199. (h) Nicolaou, K. C.; Bulger, P. G.; Sarlah, D. Angew. 48, 3644. (b) Campeau, L.-C.; Stuart, D. R.; Leclerc, J.-P.; Bertrand- Chem., Int. Ed. 2005, 44, 4490. (i) Gradillas, A.; Pérez-Castells, J. Laperle, M.; Villemure, E.; Sun, H.-Y.; Lasserre, S.; Guimond, N.; Angew. Chem., Int. Ed. 2006, 45, 6086. (j) Schrodi, Y.; Pederson, R. Lecavallier, M.; Fagnou, K. J. Am. Chem. Soc. 2009, 131, 3291. (c) L. Aldrichimica Acta 2007, 40, 45. Yang, F.; Wu, Y.; Li, Y.; Wang, B.; Zhang, J. Tetrahedron 2009, (59) Ho ng, S. H.; Grubbs, R. H. J. Am. Chem. Soc. 2006, 128, 3508. 65, 914. (d) Scarborough, C. C.; McDonald, R. I.; Hartmann, C.; (60) Ha giwara, H.; Sugawara, Y.; Hoshi, T.; Suzuki, T. Chem. Commun. Sazama, G. T.; Bergant, A.; Stahl, S. S. J. Org. Chem. 2009, 74, 2005, 2942. 2613. (e) Kim, J.; Jo, M.; So, W.; No, Z. Tetrahedron Lett. 2009, 50, (61) Huang, S.-H.; Chen, J.-R.; Tsai, F.-Y. Molecules 2010, 15, 315. 1229. (f) Gorelsky, S. I.; Lapointe, D.; Fagnou, K. J. Am. Chem. Soc. (62) Ir anpoor, N.; Firouzabadi, H.; Tarassoli, A.; Fereidoonnezhad, M. 2008, 130, 10848. Tetrahedron 2010, 66, 2415. “Designer”-Surfactant-Enabled Cross-Couplings in Water at Room Temperature 16 Bruce H. Lipshutz* and Subir Ghorai

(63) Lipshutz, B. H.; Ghorai, S.; Abela, A. R.; Moser, R.; Nishikata, T.; (83) Lipshutz, B. H.; Huang, S.; Leong, W. W. Y.; Isley, N. A., submitted Duplais, C.; Krasovskiy, A.; Gaston, R. D.; Gadwood, R. C. J. Org. for publication, 2012. Chem. 2011, 76, 4379. (84) Lip shutz, B. H. Organocopper Chemistry. In Organometallics in (64) Mancin, F.; Scrimin, P.; Tecilla, P.; Tonellato, U. Coord. Chem. Rev. Synthesis: a Manual, 2nd ed.; Schlosser, M., Ed.; Wiley: Chichester, 2009, 253, 2150. U.K., 2002, pp 665–816. (65) Minkler, S. R. K.; Lipshutz, B. H; Krause, N. Angew. Chem., Int. Ed. (85) Lin dström, U. M.; Andersson, F. Angew. Chem., Int. Ed. 2006, 45, 2011, 50, 7820. 548. (66) (a ) Lipshutz, B. H. Synlett 2009, 509. (b) Deutsch, C.; Krause, N.; Lipshutz, B. H. Chem. Rev. 2008, 108, 2916. Trademarks. BRIDP® and SEGPHOS® (Takasago International Corp.); (67) (a) Mahoney, W. S.; Brestensky, D. M.; Stryker, J. M. J. Am. Chem. Brij® and TWEEN® (Uniqema Americas LLC); CAS Registry Number® Soc. 1988, 110, 291. (b) Mahoney, W. S.; Stryker, J. M. J. Am. Chem. (The American Chemical Society); COREXIT® (NALCO Co.); BASF®, Soc. 1989, 111, 8818. Cremophor®, and Solutol® (BASF SE); Span™ (Croda International PLC); (68) (a ) Czekelius, C.; Carreira, E. M. Angew. Chem., Int. Ed. 2003, 42, Triton® (Union Carbide Corp.). 4793. (b) Lee, D.; Kim, D.; Yun, J. Angew. Chem., Int. Ed. 2006, 45, 2785. About the Authors (69) Saito, T.; Yokozawa, T.; Ishizaki, T.; Moroi, T.; Sayo, N.; Miura, T.; Bruce Lipshutz has been on the faculty at UC Santa Barbara for the Kumobayashi, H. Adv. Synth. Catal. 2001, 343, 264. past 33 years. His training initially as an undergraduate with Howard (70) Hu ang, S.; Voigtritter, K. R.; Unger, J. B.; Lipshutz, B. H. Synlett Alper (SUNY at Binghamton), then Harry Wasserman (Yale), and 2010, 2041. finally as a postdoctoral student with E. J. Corey (Harvard) set the stage (71) Lip shutz, B. H.; Servesko, J. M. Angew. Chem., Int. Ed. 2003, 42, for his interest in organic synthesis and, in particular, organometallics. 4789. From his early contributions in the form of reagents such as SEM-Cl (72) Lawrence, N. J.; Drew, M. D.; Bushell, S. M. J. Chem. Soc., Perkin and higher order cyanocuprates to heterogeneous catalysts in the form Trans. 1 1999, 3381. of nickel- and copper-in-charcoal, the focus has been on providing (73) (a) Schmid, R.; Broger, E. A.; Cereghetti, M.; Crameri, Y.; Foricher, technologies that are broadly applicable to synthetic problems. More J.; Lalonde, M.; Mueller, R. K.; Scalone, M.; Schoettel, G.; Zutter, recently, he and his co-workers have turned their attention in large U. Pure Appl. Chem. 1996, 68, 131. (b) Schmid, R.; Foricher, J.; measure to “green” chemistry, in appreciation of the major problems Cereghetti, M.; Schönholzer, P. Helv. Chim. Acta 1991, 74, 370. now facing society from the standpoint of sustainability, and, more (74) (a ) Togni, A.; Breutel, C.; Schnyder, A.; Spindler, F.; Landert, H.; specifically, issues associated with the reduction of organic waste, Tijani, A. J. Am. Chem. Soc. 1994, 116, 4062. (b) Blaser, H.-U.; much of which is solvent-related. Hence, the Lipshutz group has Brieden, W.; Pugin, B.; Spindler, F.; Studer, M.; Togni, A. Top. introduced the concept of “designer” surfactants, utilizing micellar Catal. 2002, 19, 3. catalysis as an environmentally innocuous means of carrying out (75) (a) Nicolaou, K. C.; Bulger, P. G.; Sarlah, D. Angew. Chem., Int. Ed. important transition-metal-catalyzed cross-coupling reactions, as 2005, 44, 4442. (b) Miyaura, N.; Suzuki, A. Chem. Rev. 1995, 95, well as several other reaction types (e.g., organocatalysis), in water 2457. at room temperature. (76) Fo r a copper-catalyzed borylation of aryl bromides at ambient Subir Ghorai was born in 1977 in Panskura, West Bengal, India. temperature, see Kleeberg, C.; Dang, L.; Lin, Z.; Marder, T. B. After receiving his B.S. and M.S. degrees in chemistry from Jadavpur Angew. Chem., Int. Ed. 2009, 48, 5350. University, India, he joined the Indian Institute of Chemical Biology (77) Lipshutz, B. H.; Moser, R.; Voigtritter, K. R. Isr. J. Chem. 2010, 50, 691. (IICB), Jadavpur, in 2000 as a CSIR research fellow. He received his (78) Lou, S.; Fu, G. C. Adv. Synth. Catal. 2010, 352, 2081. Ph.D. degree in 2005 from IICB, working under the supervision of (79) Alt ernatively, direct oxidation to the corresponding phenol should Dr. Anup Bhattacharjya on the synthesis of chiral dendrimers and be possible; see Inamoto, K.; Nozawa, K.; Yonemoto, M.; Kondo, Y. heterocycles from carbohydrate precursors. From 2005 to 2006, he Chem. Commun. 2011, 11775. worked on isonitrile chemistry as a postdoctoral fellow with Professor (80) Myers, D. Surfactant Science and Technology, 3rd ed.; Wiley- Michael C. Pirrung at the University of California, Riverside. He Interscience: Hoboken, NJ, 2006. then moved to UC Santa Barbara as a postdoctoral fellow working (81) Lipshutz, B. H. University of California, Santa Barbara, CA. with Professor Bruce H. Lipshutz, where he helped initiate “green” Unpublished work, 2007. chemistry involving transition-metal-catalyzed reactions in aqueous (82) Th e potential for “designer” surfactants to provide an infinite media. Recently, he has taken a position at Sigma-Aldrich in number of organic solvents was first recognized (2010) by Dr. Zarko Sheboygan, Wisconsin, as an R&D scientist in the Catalysis and Boskovic at UCSB. Organometallics group. Looking for Common Applications: • Buchwald-Hartwig amination and C-O coupling • Suzuki, Negishi, Stille, Hiyama, Sonogashira cross-couplings highly efficient • -Arylation reaction cross-coupling New Applications for Buchwald Ligands N-Arylation of secondary amides1 n-Bu X O (allylPdCl)2 N O ligands? + n-Bu N CH 3 JackiePhos t-Bu H CH Cat. No. 731013 t-Bu 3 X = Cl, OTf, ONf 87%

Trifluoromethylation of aryl chlorides2

Cl CF3 Pd2(dba)3, TESCF3 NC BrettPhos NC Cat. No. 718742 78%

Conversion of aryl and vinyl triflates to bromides and chlorides3 OTf X Aldrich Chemistry is excited to expand our ever growing list of Pd 2(dba)3, KX R R Buchwald ligands. These new air- and moisture-stable ligands t-BuBrettPhos promote cross-coupling reactions more efficiently and exhibit Cat. No. 730998 70-95% improved reactivity compared to other catalytic systems. X = Br, Cl Conversion of aryl triflates to aryl fluorides4

Features: OTf [(cinnamyl)PdCl]2, CsF F R R White crystalline solids Wide functional group t-BuBrettPhos • • Cat. No. 730998 tolerance 57-84% • Air- and moisture-stable Excellent selectivity 5 • Thermally stable • O-Arylation of ethyl acetohydroximate and conversion X OH O N (allylPdCl)2 N • Highly efficient R + R EtO Me t-BuBrettPhos Me OEt CF CF Cat. No. 730998 3 3 X = Br, Cl, I 70-95% Cy2P i t i H3CO Pr H3CO Bu 2P Pr F C P CF 3 i 3 6 iPr iPr CH3O Pr Conversion of aryl chlorides and sulfonates to nitroaromatics X NO i i 2 Pr i Pr Pd 2(dba)3 CH3O Pr + NaNO CH3O R 2 R iPr t-BuBrettPhos New New CH3O New Cat. No. 730998 X = Cl, OTf, ONf 74-98% BrettPhos t-BuBrettPhos JackiePhos 718742 730998 731013 References: t (1) Hicks, J. D. et al. J. Am. Chem. Soc. 2009, 131, 16720. (2) Cho, E. J. et al. Science 2010, 328, 1679. Cy2P Cy2P i Cy2P i Bu 2P OCH3 O Pr Pr iPr (3) Shen, X. et al. J. Am. Chem. Soc. 2010, 132, 14076. (4) Watson, D. A. et al. Science 2009, 325, 1661. (5) Maimone, T. J.; Buchwald, S. L. J. Am. Chem. Soc. 2010, 132, 9990. (6) Fors, B. P.; iPr i Pr Buchwald, S. L. J. Am. Chem. Soc. 2009, 131, 12898. i i H3CO PrO Pr iPr

SPhos RuPhos XPhos t-BuXPhos 638072 663131 638064 638080 To find out more about For inquiries about bulk quantities, contact [email protected]. these ligands, visit Aldrich.com/buchwald

78098

78098_Ligands Ad (Acta 45.1).indd 1 3/28/2012 12:07:08 PM When you need Niobium Compounds for organic synthesis and catalysis. Professor Lacerda’s review focuses on applications of niobium compounds in catalysis of: • Aldol Reactions • Diels–Alder Reactions • Friedel–Crafts Reactions Epoxide Ring-Openings Niobium compounds have a variety of applications, including • the following: • Multicomponent Reactions • Catalysis • Protection and Deprotection Reactions • Nuclear technology • Demethylation Reactions • Superconductive magnets Electronics industry • A representative example employing NbCl5 as a Lewis acid in a

Diels–Alder reaction is shown below. The use of NbCl5 ensures formation of the endo stereoisomer. In contrast, the reaction Aldrich’s Niobium Compounds Portfolio does not take place in the absence of Lewis acids, and gives mixtures of endo and exo products, when the more commonly used AlCl is utilized. NbCl5 Nb(OCH2CH3)5 Nb2O5 3 O 215791 339202 203920 O H

0.5 eq. NbCl5 + NbO2 LiNbO3 -78 °C, EtOAc, 90 min H 383163 254290 326364 74% (endo)

OCH3 NbCl3 H CO NaNbO 3 3 Constantino, M. G.; Lacerda, V., Jr.; da Silva, G. V. J. Molecules 2002, 7, 456. 326356 400653

CH3

To order any of these Cl Nb Cl Nb Cl Cl niobium products, visit Aldrich.com/niobium CH3 260924 553956

78140

78140_Niobium.indd 1 3/28/2012 11:39:40 AM 19 VOL. 45, NO. 1 • 2012 The Growing Impact of Niobium in Organic Synthesis and Catalysis

Valdemar Lacerda, Jr.,*,a Deborah Araujo dos Santos,a Luiz Carlos da Silva-Filho,b Sandro José Greco,a and Reginaldo Bezerra dos Santosa a Departamento de Química Centro de Ciências Exatas Universidade Federal do Espírito Santo Campus Goiabeiras Prof. Dr. Valdemar Ms. Deborah Araujo dos Prof. Luiz Carlos da Silva- Lacerda, Jr. Santos Filho Avenida Fernando Ferrari, 514 Goiabeiras, Vitória, ES 29075-910, Brasil Email: [email protected] b Departamento de Química Faculdade de Ciências Universidade Estadual Paulista Campus Bauru Avenida Eng. Luiz Edmundo Carrijo Coube, 14-01 Vargem Limpa, Bauru, SP 17033-360, Brasil Prof. Sandro José Greco Prof. Reginaldo Bezerra dos Santos

Keywords. catalysis; organic synthesis; niobium chlorides; niobium niobium is highly oxophilic.1,2 Niobium can easily accommodate oxides; selectivity; versatility; efficiency. a number of ligands presenting different coordination numbers.3,4 For this reason, its organometallic chemistry is very rich,5 and a Abstract. The growing interest in, and applications of, niobium large number of niobium complexes have been reported.5,6 Niobium compounds in organic synthesis and catalysis are surveyed, with a possesses different oxidation states, which range from +5 to –3. Its focus on their efficiency and versatility in several classical and broad chemistry, however, is dominated by the higher oxidation states, organic reaction types. It is our hope that this review will spur further especially the +5 one.4 Niobium was originally called columbium (Cb) investigations of this lesser studied, but equally important, member of by Hatchett in 1802,7 was renamed niobium by Rose in 1844,8 and group 5 transition metals. then niobe. Finally, over a century later, IUPAC officially adopted the name niobium in 1949–1950.9 Niobium does not occur in nature in its Outline free metal form,1c but as a mixture of metal oxides such as columbites

1. Introduction [(Fe,Mn)(Nb,Ta)2O6] and pyrochlore [(Na,Ca)2Nb2O6(OH,F)]. The

2. Catalysis by Niobium Pentachloride (NbCl5) most commercially important ore deposits are in Brazil, Canada, 2.1. Aldol and Related Reactions Nigeria, and Zaire. About 86% of world reserves are in Brazil, which 2.2. Diels–Alder Reactions accounts for roughly 60% of total niobium production.1b,1c 2.3. Friedel–Crafts Reactions Many researchers have focused on the solid-state chemistry of 2.4. Epoxide Ring-Opening Reactions niobium and its compounds, in order to produce catalysts and other 2.5. Multicomponent Reactions materials for industrial applications.1b,10 Because of its high resistance 2.6. Protection and Deprotection Reactions to corrosion and high electrical conductivity, niobium is ideal for 2.7. Demethylation Reactions chemical and metallurgical applications such as: (i) heterogeneous 3. Other Niobium-Based Catalysts catalysis (catalyst components and co-catalysts), (ii) space and 3.1. Solid-Phase Niobium(V) aeronautical industries (Nb–Al–Ti alloys), (iii) superconductivity

3.2. NbCl3 and Niobium(III) complexes (magnets based on Nb–Sn alloys), and (iv) electronics industry

3.3. HNbMoO6•nH2O (capacitors, ceramics, bone implants, and internal suturing—since it 3.4. Niobium Alkoxides is completely inert to bodily fluids). Nevertheless, around 85 to 90% 4. Conclusion of the niobium produced worldwide is used in the steel industry as 5. Acknowledgments iron–niobium alloys (ferroniobium), which can contain from 40 to 6. References 70% niobium.11 The most commonly used niobium compound is undoubtedly

1. Introduction commercially available niobium pentachloride (NbCl5), which is Being in the same periodic table group as tantalum and vanadium highly Lewis acidic and, for this reason, has received increasing

(which is known for its many applications in organic synthesis), attention in recent years. NbCl5 can be prepared in several ways, The Growing Impact of Niobium in Organic Synthesis and Catalysis 20 Valdemar Lacerda, Jr.,* Deborah Araujo dos Santos, Luiz Carlos da Silva-Filho, Sandro José Greco, and Reginaldo Bezerra dos Santos

but the easiest one is the direct chlorination of metallic niobium at the same reactions employing other Lewis acids. The main purpose 300–350 ºC.1c Niobium pentachloride is a yellow solid that is quickly of this article is to highlight the most recent (over the last five years)

hydrolyzed and transformed into HCl and NbOCl3 or Nb2O5•nH2O applications of niobium pentachloride and other niobium compounds (niobic acid). It dissolves into nonaqueous solvents such as alcohols in organic synthesis and catalysis, including our own recent results.

and acetonitrile, and forms stable 1:1 complexes with a number of In this context, NbCl5 offers several benefits such as ease of handling donor ligands, including ethers, thioethers, tertiary amines, nitriles, and, generally, low catalyst loadings. 12 etc. NbCl5 exists as dimeric units in the solid state, in which the 4 metal is surrounded by a distorted octahedron of chlorine atoms. This 2. Catalysis by Niobium Pentachloride (NbCl5) dimer can be seen as two octahedra sharing one edge (Figure 1).4 It is 2.1. Aldol and Related Reactions strongly electrophilic and hence is able to catalyze a variety of organic Niobium pentachloride effectively catalyzes the Knoevenagel reactions, which is the subject of this review. condensation of aromatic and aliphatic aldehydes with active 15 The applications of NbCl5 in organic synthesis and the prospects for methylene compounds (eq 1). This has proven to be an efficient this promising reagent have been reviewed.13,14 Interestingly, several method for preparing α,β-unsaturated carbonyl compounds in

reactions mediated by NbCl5 had a different outcome as compared to good yields, high selectivity, shorter reaction times, and under mild reaction conditions in the presence or absence of solvent. The reaction proceeds presumably through activation of the aldehyde by complexation with Nb(V), followed by nucleophilic addition of the Cl Cl active methylene compound. Cl Cl Cl Another type of Aldol reaction, described by Yadav et al. involves Nb Nb the NbCl -catalyzed preparation of β-keto esters through insertion of Cl Cl Cl 5 ethyl diazoacetate (EDA) into the C–H bond of various aldehydes (eq Cl Cl 2).16 These reactions readily occur at room temperature in good yields and high selectivity (no glycidic esters, diethyl maleate, or diethyl fumarate side products are observed), and are believed to take place Figure 1. Dimeric Structure of Niobium Pentachloride in the Solid State. as shown in equation 2. Of a number of Lewis acid catalysts (InCl , (Ref. 4) 3 CeCl3•7H2O, TaCl5, and GdCl3) also tested in this reaction, NbCl5 was found to be the most effective. No reaction was observed between the aldehyde and EDA in the absence of the Lewis acid.

O O O NbCl 2 5 2 2.2. Diels–Alder Reactions + 1 R 1 R RHR neat or MeCN R In the Diels–Alder reaction between dienes and dienophiles, some rt or reflux 0.75–4.0 h R of the usually sluggish dienophiles (e.g., 2-cycloenones 1–4) can undergo the cycloaddition at 3 different temperatures (–78 ºC, rt, or NbCl5 (mol %) R R1 R2 Yield reflux) with unusually high stereoselectivity in the presence of NbCl5 17 as a Lewis acid (eq 3). NbCl5 induces a decrease in the energy of 10 Ph EtO CN 90% the LUMO of the carbonyl substrate through complexation with the 10 3-Py EtO CN 84% carbonyl oxygen, thus reducing the electron density of the double 10 2-Fur EtO CN 92%

10 n-C7H15 EtO CN 88% bond. 20 Ph EtO CO2Et 86% A related study has described the efficient synthesis of biologically 20 2-O2NC6H4 Me Ac 90% important, fused pyrano[3,2-c]quinoline derivatives by the aza- 20 n-C H Me Ac 70% 7 15 Diels–Alder reaction of Schiff bases (5) and 3,4-dihydro-2H-pyran 20 4-ClC6H4 Me CO2Me 80% 18 (6) facilitated by niobium(V) chloride (eq 4). In this context, NbCl5 eq 1 (Ref. 15) is effective, promoting short reaction times and generally improved yields and diastereoselectivities, especially when lower molar

concentrations of NbCl5 are employed. A versatile intermediate in the synthesis of eremophilanes

O O OO and bakkanes has been prepared by a highly regioselective and NbCl5 (5 mol %) + N2 stereoselective one-step synthesis that relies on an NbCl5-catalyzed RH OEt R OEt CH2Cl2, rt, 3–5 h Diels–Alder reaction (Scheme 1).19 The cycloaddition does not take 75–92% place in the absence of Lewis acids. NbCl5 plays a critical role, since it increases the reactivity to the point that the reaction is carried out at ≤ –50 ºC, thus reducing diene polymerization and improving NbV NbV O O O– O selectivity. The importance of this intermediate was demonstrated in

RH– OEt R OEt the total synthesis of (±)-bakkenolide A. + H + N2 N2 2.3. Friedel–Crafts Reactions R = alkyl, aryl, phenethyl, heterocyclic Inter- and intramolecular Friedel–Crafts acylations are also highly 20 efficiently catalyzed by NbCl5. In the intermolecular variant, the eq 2 (Ref. 16) introduction of additives, such as AgClO4, significantly enhances the catalytic action of the niobium complex. Arai et al. have reported that 21 VOL. 45, NO. 1 • 2012

NbCl5 smoothly catalyzes the acylation of aromatic compounds with

Ac2O and Bz2O to form the corresponding ketones in excellent yields (eq 5).20a An intramolecular variant was described by Constantino and O O O co-workers, who converted 3-arylpropanoic acids (11) into 1-indanones R1 H H Ph R2 (6) R1 R1 N H 2 H 12 in good yields and under mild conditions in the presence of 1.5– R + R2 20b NbCl (12.5, 25 N N 2.5 equivalents of NbCl (eq 6). The authors demonstrated through 5 3 5 H H 5 R R or 50 mol %) 4 R5 R3 R5 R3 NMR experiments that NbCl5 initially performs the conversion of the R MeCN, rt 4 4 3-arylpropionic acids into acyl chloride and anhydride derivatives. 5 1–190 min R R 1 2 3 4 5 endo (7) These intermediates are then converted into 1-indanones through a R , R , R , R , R = exo (8) H, Me, MeO, OCH2O, Cl, NO2 67–92%, endo:exo = 0:100 to 50:50 Friedel–Crafts acylation reaction.

2.4. Epoxide Ring-Opening Reactions The opening of epoxide rings is one of the most studied applications eq 4 (Ref. 18) of NbCl5 as catalyst. What this growing number of studies has shown is that small changes in reaction conditions can result in significantly different products being formed. One study reported that a number of cyclohexene oxide derivatives reacted rapidly with O Me 0.13–0.5 equivalents of NbCl , resulting in good conversions, but led Me NbCl5 Me 5 (25 mol %) H to mixtures of chlorohydrins (products containing solvent residues) Me + CH Cl 21 CHO OBn 2 2 and rearrangement products. –50 oC, 72 h H 4 equiv BnO In contrast, Oh and Knabe reported that addition of metallic zinc to 45% H ‡ the epoxide and NbCl5 promoted deoxygenation to the corresponding Me E alkenes under mild conditions and in good yields and relatively H Me 22 H short reaction times (eq 7). Cl4Nb O β-Amino alcohols are versatile intermediates in the synthesis H H of, among others, a wide variety of biologically active compounds, BnO synthetic amino acids, β-blockers, oxazolines, and chiral auxiliaries. 6 steps The NbCl5-catalyzed ring-opening of epoxides with aromatic amines leads to the formation of β-amino alcohols in excellent yields and Me O Me regioselectivity under mild reaction conditions (eq 8).23 The reaction (±)-7-epi-bakkenolide A + O is noteworthy since it does not require anhydrous solvents or stringent H reaction conditions and does not proceed in the absence of NbCl5. (±)-bakkenolide A

Scheme 1. The Key, NbCl -Catalyzed Step in the Synthesis of Eremophilane O NbCl O R1 O R1 5 R1 5 (50 mol %) and Bakkane Systems. (Ref. 19) n + n + n Et2O R2 2 2 R3 o R H R H –78 C, rt, R3 R3 1–4 or reflux endo exo

1; R1 = R2 = R3 = H, n = 1; 2; R1 = R2 = R3 = H, n = 2 O 3; R1 = Me, R2 = R3 = H, n = 2; 4; R1 = H, R2 = R3 = Me, n = 2 NbCl5 (1 mol %), AgClO4 (3 mol %) Ar H Ar R' (R'CO)2O (1.5 or 2.0 equiv), MeNO2 No. Temp Time Yield Endo:Exo 9 80 oC, 2–30 h 10

1 –78 oC 3 h 61% 89:11 OMe OMe OMe OMe 1 rt 0.42 h 58% 78:22 OMe 1 reflux 0.08 h 65% 74:24 2 –78 oCa 3 h 72% 100:0 OMe 2 rt 0.75 h 58% 80:20 2 reflux 0.25 h 62% 78:22 O Ph O Me O Me O Me o 3 –78 C 8 h 32% 48:52 100% 58% 79%99% 3 rt 24 h 43% 42:58 3 reflux 12 h 65% 30:70 Me O 4 –78 oC 8 h 40% 100:0 Me Ph Ph 4 rt 24 h 34% 100:0 S O 4 reflux 24 h 48% 100:0 Me Me O O N a With AlCl3 (25 mol %) and 6 equiv of O Ph Ts o cyclopentadiene at 40 C, 7 h in PhMe: 80%, 95% 86% 52% (at rt)92% endo:exo = 89:11. With SnCl4 (1.0 equiv) and 50 equiv of cyclopentadiene at –20 oC, 14 h: 93%, endo:exo = 92:8. eq 3 (Ref. 17) eq 5 (Ref. 20a) The Growing Impact of Niobium in Organic Synthesis and Catalysis 22 Valdemar Lacerda, Jr.,* Deborah Araujo dos Santos, Luiz Carlos da Silva-Filho, Sandro José Greco, and Reginaldo Bezerra dos Santos

2.5. Multicomponent Reactions

O A multicomponent reaction (MCR) is defined as a process in which R1 R1 three or more reactants are combined in one pot to form a structurally OH NbCl5 (1.5–2.5 equiv) complex product that incorporates structural elements from each 2 anhyd CH2Cl2, rt, 2 h 2 R R reactant. Lu and co-workers have described such an MCR in which R3 R3 O 11 12 β-amino carbonyl compounds were synthesized in high yields by O an NbCl5-catalyzed Mannich-type reaction between acetophenone, R1 Cl benzaldehydes, and anilines (eq 9).24 Similarly, Kim’s group reported

R2 a simple and efficient one-pot, NbCl5-catalyzed synthesis of R3 α-aminonitriles through an MCR involving aldehydes, amines, and trimethylsilyl cyanide (eq 10).25 R1 R2 R3 Yield

H H H 82% 2.6. Protection and Deprotection Reactions H MeO H 31% One of the major challenges in total synthesis is effecting the MeO MeO H 83% protection and deprotection of a variety of functional groups. Low MeO MeO MeO 95% concentrations of NbCl5 catalyze the acetylation of alcohols, phenols, amines, and thiols under mild reaction conditions (eq 11).26 These eq 6 (Ref. 20b) reactions are characterized by short reaction times, cleaner products, and high yields. Based on these conditions and results, the following order of ease of acetylation of OH groups was established: phenolic >

2 benzylic > primary aliphatic > secondary aliphatic > tertiary. R NbCl5 (0.5 equiv) R1 R1 R2 Zn (5.0 equiv) As a homogeneous catalyst, NbCl5 is even more effective in O 3 THF–PhH (1:1) 3 4 catalyzing the acetylation of the carbonyl group of aldehydes to form R 4 R R R 23 oC, 0.5–96 h 35–95% acetals. The transformation takes place with acetic anhydride in the Rx = alkyl, aryl, CO absence of solvent, and leads to excellent yields in short reaction times 27 eq 7 (Ref. 22) (eq 12). The acetals thus formed tend to be stable in neutral, basic, and acidic media and have been employed in synthesis as starting materials, intermediates, and cross-linking reagents for cellulose. Tetrahydropyrans (THPs) are attractive alcohol protecting groups because they are stable under a variety of reaction conditions, yet can ArNH2 OH OH O NbCl5 (10 mol %) H H be easily cleaved in dilute acid when needed. Niobium pentachloride N or 1 N 1 2 Ar R Ar R R CH2Cl2, rt, 1–4 h catalyzes the smooth tetrahydropyranylation of alcohols and phenols 1 R at room temperature, leading to the protected counterparts in high- 1 2 80–95% R = Ph, PhOCH2, ClCH2; R = H 28 1 2 to-excellent yields (eq 13). Some of the advantages of this method, R ,R = (CH2)4 Ar = mono- or disubstituted benzene when compared to previously reported hydroxyl-protection methods, are: (i) mild reaction temperature, (ii) shorter reaction times, (iii) eq 8 (Ref. 23) lower catalyst loadings, (iv) better yields, (v) greater tolerance of other functional groups, and (vi) easier workup.

It is important to note that, not only does NbCl5 readily promote functional-group-protection reactions, it can also effect the smooth 2 O R deprotection of functional groups such as methoxy methyl ethers PhC(O)Me NH O HN H 2 NbCl5 (10 mol %) (MOMs) of alkyl, allyl, propargyl, and benzyl alcohols; MOMs of + 29 EtOH, rt, 12 h Ph phenols; and the cleavage of MOM esters (Scheme 2). R1 R1 R2 76–96% 1 2.7. Demethylation Reactions R = H, 4-Me, 4-NO2 2 R = H, 4-Me, 3,4-Me2, 4-MeO, 3-Cl, 4-Cl, 4-NO2, 3-CO2H, 4-CO2H Hashimoto and co-workers showed that niobium pentachloride can effect the regioselective demethylation of 13, a key step in the catalytic asymmetric synthesis of descurainin (15), which is widely eq 9 (Ref. 24) used as a Chinese traditional medicine to relieve coughing, prevent

R2 NH NbCl5 (10 mol %) O 1 2 NbCl (10 mol %) R C(=O)H + R NH2 + TMSCN 1 5 MeCN, rt, 0.3–5 h R CN RXH + Ac2O CH Cl , rt, 1–3 h X Me 89–96% 2 2 R R1 = alkyl, vinyl, aryl, heteroaryl, and others groups 2 X = O, S, NH 86–100% R = Ph, Bn, 4-MeC6H4CH2, 4-MeOC6H4 R = alkyl, allyl, cycloalkyl, aryl

eq 10 (Ref. 25) eq 11 (Ref. 26) 23 VOL. 45, NO. 1 • 2012 asthma, reduce edema, and as a diuretic. Employing the Arai–Nishida protocol, they treated 13 with NbCl5 in 1,2-dichloroethane at 70 ºC to effect the regioselective demethylation of the 4-MeO group, affording O OAc NbCl5 (5 mol %) 30 phenol 14 as the sole product in 79% yield (Scheme 3). + Ac2O R H neat, rt, 10–15 min R OAc 90–98% 3. Other Niobium-Based Catalysts R = Ph, PhCH=CH, XC6H4 (X = 4-Me, 4-MeO, 4-F, 3.1. Solid-Phase Niobium(V) 2-Cl, 4-Cl, 4-Br, 2-NO2, 3-NO2, 4-NO2), 2,4-Cl2C6H3 Barbosa’s group has demonstrated a clean, efficient, and rapid method for esterifying sterically (biodiesels) or other inactive (aromatic) eq 12 (Ref. 27) carboxylic acids by using Lewis acids on solid supports. In all cases investigated, the yields obtained with the mixed catalysts were similar to, or higher than, those reported in the literature. Results for the 31 Nb2O5–Zn and Nb2O5–Fe are presented in eq 14. Trial experiments without the solid-phase Lewis acid did not produce any ester product. NbCl5 (10 mol %) ROH + Another study by the same group showed that alcohols and CH Cl , rt, 2–3 h O 2 2 RO O acids can be switched to produce ethers or esters by varying the R = alkyl, allyl, Bn, aryl 86–95 % alcohol-to-catalyst molar ratio in the case of NbCl5–Al2O3 under “solvent free” conditions and microwave irradiation. A “two sites” mechanism was proposed for the reaction to explain the tendency of eq 13 (Ref. 28) the catalyst effectiveness to be dependent on the steric and electronic characteristics of the alcohol alone during the esterification process.32

10 wt % Nb2O5 supported on silica–alumina catalyzes the liquid- phase esterification of acetic acid with a variety of alcohols at NbCl5 (1 equiv) 85–128 ºC. After 8 h, and using an acid:alcohol molar ratio of 2:1, (a) OBn OBn Me O O MeCN, 0 oC to rt, 2 h Me OH 100% selectivity in all cases and good conversions were observed for Me 77% ethyl (83%), n-butyl (87%), and isopentyl (91%) acetates.33 Following a series of tests, conversions with the supported catalyst were better 11 additional examples; 70–93% than those obtained with the isolated oxides, and better yet than those OMeO OMeO MeO Me MeO obtained in the absence of catalyst. O O NbCl5 (1 equiv) OH (b) Nb2O5–SiO2 mixed-oxide nanocomposites containing 7–37 wt % o MeO MeCN, 0 C to rt, 1 h MeO Nb were synthesized by a new sol–gel route, and their textural and 93% surface acid properties investigated. Their activity as heterogeneous 1 additional example; 91% catalysts was tested in the epoxidation of cyclooctene with H2O2. The materials containing up to 23.0 wt % Nb were stable and active catalysts for the epoxidation of cyclooctene. The highest activity and Scheme 2. Deprotection of MOM Ethers and Esters with NbCl5. (Ref. 29) selectivity for H2O2 was exhibited by the catalyst with the lowest Nb content, which also showed high stability in reuse. The catalytic properties were shown to be related to new acid sites that are different from those that exist in pure Nb and Si oxides, and to the presence 34 of NbOx species. Niobium(V) oxide efficiently catalyzes the MeO MeO transesterification of β-keto esters with a variety of alcohols. Good OMe OH NbCl5 (3 equiv) conversions and moderate-to-good isolated yields have been obtained OMe ClCH2CH2Cl OMe o at faster rates than those recently reported for various other catalysts H O 70 C, 1 h H O (eq 15).35 Gonçalves and co-workers have reported that biodiesel can OH OH be obtained from fatty acid raw materials through esterification, and 13 investigated, empirically and theoretically, the reactivity of lauric, 14, 79% palmitic, stearic, oleic, and linoleic fatty acids towards methanol by 4 steps using powdered niobic acid (niobium oxide solid) as a heterogeneous 36 catalyst. MeO OH Somma et al. have described the preparation of a series of OMe niobium-based aerogel samples (Nb2O5–SiO2, Nb2O5–Al2O3, Nb2O5– ZrO ) under acidic and basic conditions, and investigated the effect of H O 2 OH the matrix (SiO2, Al2O3, ZrO2) and of the gelation conditions (acidic or basic) on the surface area, the porosity, and the catalytic activity O of the solids in the oxidation of different substrates with hydrogen descurainin (15), 95% ee peroxide. The amount of niobium was constant in all samples tested in the oxidation of unsubstituted (cyclooctene) and substituted (geraniol, nerol, and trans-2-pentene-1-ol) olefins. It was found that, even though Scheme 3. Regioselective Demethylation with NbCl . (Ref. 30a) the catalysts were moderately active, they still produced the epoxides 5 in good yields, and that yields are influenced by the matrix properties The Growing Impact of Niobium in Organic Synthesis and Catalysis 24 Valdemar Lacerda, Jr.,* Deborah Araujo dos Santos, Luiz Carlos da Silva-Filho, Sandro José Greco, and Reginaldo Bezerra dos Santos

(surface acidity and surface area). The merits of this approach is that 3.2. NbCl3 and Niobium(III) Complexes

it permits the preparation of catalysts that are resistant to leaching Obora and co-workers have demonstrated that NbCl3(DME) and can be recycled several times without appreciable loss of catalytic successfully catalyzes the intermolecular [2 + 2 + 2] cycloaddition of activity.37 alkynes and alkenes, giving 1,4,5-trisubstituted 1,3-cyclohexadiene A new approach for functionalizing activated and deactivated derivatives in good yields.39 A year later, the same group reported arenes with iodine, promoted by heterogeneous catalysts, has been excellent yields and high chemo- and regioselectivities in the

reported by Carniti et al. The overarching goal of this study was the NbCl3(DME)-catalyzed intermolecular [2 + 2 + 2] cycloaddition development of “greener” iodination processes. Thus, the activity of of tert-butylacetylene with α,ω-dienes, affording 5-ω-alkenyl-1,4- several different types of solid acid catalysts (acid resins, zeolites, disubstituted-1,3-cyclohexadienes (eq 16).40 mixed oxides, niobium oxide, and niobium phosphate) was examined Interestingly, Ni(0) catalyzes the cross-coupling of Nb(III)– in the direct iodination reaction of phenol as a model arene. The mild, alkyne complexes with aryl iodides. An excess of lithium alkoxide,

eco-friendly conditions (50 ºC in methanol in the presence of H2O2 as as additive, is indispensable for the success of this reaction, which oxidant) led to the efficient introduction of one, two, or three iodines leads to good yields of the corresponding 1,2-diarylalkene products in the arene. Different selectivity distributions of the iodo compounds (Scheme 4).41

formed were obtained with the different catalysts, and the latter could NbCl3(DME) mediates the reaction of aliphatic ketones with be grouped into distinct families on the basis of their ortho or para aryl-substituted alkynes to form a variety of 1,1,2-trisubstituted-1H- directing tendencies.38 indenes in good yields (Scheme 5).42 This remarkable transformation is believed to be the first example of a preparative route to the relatively rare 1,1-disubstituted indenes from aliphatic ketones, and is thought to proceed as depicted in Scheme 5. O O + HO Me Me OH Me O Me 3.3. HNbMoO6•nH2O Me Me 2.8 equiv HNbMoO6 functions as a strong solid-acid catalyst in a number

360 W µw of commonly used reactions. It exhibits high catalytic activity in Cat (80 wt %) 70 oC, 1 min 60 oC, 1 h acetalization, esterification, and hydration reactions, and its activity exceeds those of zeolites and ion-exchange resins in the Friedel– Nb O on ZnCl 87% 84% 2 5 2 Crafts alkylation. In the first report of successful acid catalysis using Nb2O5 on FeSO4 87% 82% a layered transition-metal oxide, the catalytic activity of HNbMoO6 is attributed to the intercalation of reactants into the interlayer and the eq 14 (Ref. 31) development of strong acidity. Layered HNbMoO6•nH2O consists of

layers formed of randomly sited MO6 (M = Nb and Mo) octahedra with

H2O in the interlayer. In one example, the performance of HNbMoO6

was compared with those of niobic acid (Nb2O5•nH2O), zeolites,

O O O O Nb O (10 wt %) 4 2 5 3 1 3 + R OH 1 4 + R OH R OR PhMe, reflux R OR 1 R2 2 equiv 6–12 h R2 R ClCH2CH2Cl 1 2 R R + NbCl3(DME) NbCl3(DME) R1 = Me, MeO CCH ; R2 = H 50–100% conv removed by o 2 2 60 C, 16 h 2 1 2 33–98% yield distillation 1.2 equiv R R ,R = (CH2)4 R3 = Me, Et 1. i-PrOLi (3 equiv) R4 = i-Pr, n-Bu, t-Bu, Bn, allyl, glycidyl, (–)-menthyl hexane, rt, 0.5 h 2. Ar–I (4 equiv)

3. Ni(cod) 2 (20 mol %) eq 15 (Ref. 35) THF, 50 oC, 16 h Ar(R1)C=C(R2)Ar

Isolated Isomer RC≡CH R1 R2 Ar Yield Ratio R R NbCl3(DME) (10 mol %) Ph Me Ph 65% ---- + R n Ph Me 3-MeC6H4 63% 59:41 DCE R o n Ph Me 4-MeC6H4 62% 59:41 2 equiv 40 C, 2 h R Ph Me 4-FC H 58% nd 16 17 6 4 Ph Me 4-MeOC6H4 46% 55:45 Yield Ph Me 4-ClC6H4 35% 53:47 Ph Me 4-MeO2CC6H4 53% 55:45 n R 16 17 Ph Ph Ph 66% ---- 2 t-Bu 78% 6% 3-F3CC6H4 Me Ph 72% 51:49 4 t-Bu 84% trace 4-MeC6H4 Me Ph 61% 54:46 6 t-Bu 97% trace n-Pr Me Ph 55% 64:36 6 TMS 65% 18% n-Pr n-Pr Ph 51% 65:35 8 t-Bu 73% trace 10 t-Bu 56% nd Scheme 4. Ni(0)-Catalyzed Cross-Coupling of Nb(III)–Alkyne Complexes eq 16 (Ref. 40) with Aryl Iodides. (Ref. 41) 25 VOL. 45, NO. 1 • 2012 and ion-exchange resins in the Friedel–Crafts alkylation of anisole, toluene, and benzene with benzyl alcohol in the liquid phase over the OH 43 R R layered oxide (eq 17). The yield of benzyl anisole reached 99% after solid acid catalyst 30 min, whereas those obtained with the ion-exchange resins reached (0.2 g) + o only ca. 42% even after 1 h. The turnover rate of HNbMoO6 in the 80 C, 4 h alkylation of anisole was more than three times higher than that of 10 equiv Nafion® NR50. It is worth noting that other layered transition-metal Amount Yield for R = oxides such as HNb3O8 and HTiNbO5 did not exhibit the activity. Solid Acid (mmol•g–1) MeOa Me H

b 3.4. Niobium Alkoxides HNbMo6 1.9 99% 74% 22% c Kobayashi and co-workers have reported the first example of the HNbMo6 1.9 94% 22% 8% complementary stereoselective and catalytic desymmetrization of Nb2O5•nH2O 0.3 1% nd nd Nafion® NR50 0.9 42% 19% 10% meso epoxides and meso aziridines with anilines as nucleophiles. Amberlyst® 15 4.8 42% 14% 7% This approach utilizes an extremely unusual and highly selective H-ZSM-5 zeolited 0.2 9% 1% nd niobium catalytic system that promotes closely related reactions with H-Beta zeolitee 1.0 31% 1% nd a o b opposite stereochemical outcomes. This Lewis acid system is based At 100 C for 1 h. Protonated with H3PO4. c Protonated with HNO3; reaction time = 0.5 h. on the complex formed from niobium alkoxides and a tetradentate d e SiO2:Al2O3 = 90 (JRC-Z-5-90H). SiO2:Al2O3 BINOL derivative. The resulting (R,R)-1,2-amino alcohols and = 25 (JRC-Z-HB25). (S,S)-1,2-diamines are obtained in good-to-excellent yields and very eq 17 (Ref. 43) high-to-excellent enantioselectivities (Scheme 6).44 Because of its sensitivity to steric bulk at the β carbon of epoxides, the catalyst displays a remarkable ability to distinguish between different meso epoxides by selectively facilitating the ring-opening of less sterically Nb(OMe)5 (10 mol %) hindered epoxides in the presence of more sterically hindered ones. In 18 (11 mol %) OH (a) O + PhNH2 the ring-opening reactions of both epoxides and aziridines, formation 4 Å MS (then filter) NHPh PhMe–CH Cl (3:2) of the catalyst in the presence of molecular sieves—which were then 2 2 100%, 70% ee o filtered off before addition of reactants—was found to be important 0 C, 18 h for the realization of high yields and stereoselectivities.44 EtO EtO Katsuki’s group discovered a unique asymmetric catalysis by Nb(OMe)5 (5 mol %) H 18 (5.5 mol %) N niobium–salan complexes of the epoxidation of allylic alcohols with (b) N + PhNH2 3 Å MS (then filter) NHPh hydrogen peroxide. It was first shown that a μ-oxo [Nb(salan)]2 PhMe, rt, 78 h complex catalyzes the asymmetric epoxidation of allylic alcohols 96%, 72% ee with the adduct of urea and hydrogen peroxide. Following analysis of the time course of the epoxidation, it was also discovered that in i-Pr OH situ prepared Nb(salan) complexes catalyze the epoxidation of allylic OH (18) alcohols with hydrogen peroxide in aqueous media (eq 18).45 This OH OH latter method does not require the troublesome purification of the i-Pr catalyst, and allows easy tuning of the ligand. It is the first example of a highly enantioselective epoxidation of allylic alcohols with aqueous Scheme 6. Nb(V) Catalytic System Promotes the Desymmetrization of Closely Related Systems with Opposite Stereochemical Outcomes. (Ref. 44)

NbCl (DME) O 3 (1–4 equiv) 3 4 4 R R + 1 2 R R R ClCH CH Cl 2 2 R3 R2 reflux, 18 h R1 Nb(Oi-Pr)5 (4 mol %) 19 (4 mol %) O 34–78% R OH R OH 30% H2O2 (1.5 equiv) + –H CHCl3–brine –[Nb]=O 40 oC, 24 h H R4 4 + Ar Ar R O [Nb] R Yield ee Config. H+ [Nb] 1 R2 R 1 [Nb] DCE 3 R n-Pr 57% 91% 2S,3S R4 O R2 R H NH HN n-Pent 79% 93% 2S,3S Cy 82% 93% 2S,3S 1 t-Bu 52% 95% 2S,3S R = Et, n-Pr, n-Bu, Ph(CH2)2, Cl(CH2)3, EtO2C(CH2)2 OH HO Ph 61% 74% 2S,3S R2 = Me, Et, n-Pr, n-Bu; R3 = H, Me, Ph; R4 = Me, Et Ph Ph

19 Scheme 5. First Example of a Preparative Route to 1,1-Disubstituted Indenes. (Ref. 42) eq 18 (Ref. 45) The Growing Impact of Niobium in Organic Synthesis and Catalysis 26 Valdemar Lacerda, Jr.,* Deborah Araujo dos Santos, Luiz Carlos da Silva-Filho, Sandro José Greco, and Reginaldo Bezerra dos Santos

5. Acknowledgments

1. Nb(OMe)5 (5 mol %) The authors thank the Fundação de Amparo à Pesquisa do Estado Ot-Bu 20 (5.5 mol %) R1 R1 do Espírito Santo (FAPES/FUNCITEC), the Conselho Nacional de N NMI (5.5 mol %) N + Desenvolvimento Científico e Tecnológico (CNPq), the Coordenadoria 2 R 3 Å MS 2 Me3SiO O R de Aperfeiçoamento de Pessoal do Nível Superior (CAPES), the PhMe–CH2Cl2 (1:1) –20 oC, 48 h Fundação de Amparo à Pesquisa do Estado de São Paulo (FAPESP), 2. HCl (0.1 M in THF) the Companhia Brasileira de Metalurgia e Mineração (CBMM), NMI = N-methylimidazole 0 oC, 15 min and Laboratório Pesquisa e Desenvolvimento de Metodologias OH para Análise de Petróleos do Departamento de Química da UFES R1 R2 Yield ee i-Pr (LabPetro-DQUI/UFES) for financial support. The authors thank

2-HOC6H4 Ph 81% 96% all their co-workers whose names appear in the references for their OH 2-HOC6H4 1-Np 89% 92% (20) dedication and hard work. OH 2-HOC6H4 4-MeC6H4 90% 94% 2-HOC6H4 2-F3CC6H4 94% 99% 2-HOC6H4 2-ClC6H4 89% 91% 6. References 2-HOC6H4 3-Py 74% 92% (1) (a) Payton, P. H. In Kirk-Othmer Encyclopedia of Chemical NH a 2-HO-5-MeC6H3 Cy 67% 90% b Technology, 3rd ed.; Wiley-Interscience: New York, 1981; Vol. 15, N 2-HO-5-MeC6H3 i-Pr 63% 92% b 2-HO-5-MeC6H3 i-Bu 47% 90% pp 820–840. (b) For a review of niobium compounds, see Nowak, I.; Ziolek, M. Chem. Rev. 1999, 99, 3603. (c) Schlewtiz, J. H. Niobium (+)-anabasine a In . b In toluene. and Niobium Compounds. In Encyclopedia of Chemical Technology; eq 19 (Ref. 46) Kirk, R. E., Kroschwitz, J. I., Othmer, D. F., Howe-Grant, M., Eds.; Wiley, 1996; Vol. 17, p 43. (2) Hirao, T. Chem. Rev. 1997, 97, 2707.

Nb(OMe)5 (2–10 mol %) (3) Br own, D. The Chemistry of Niobium and Tantalum. In OH Me3SiO OMe 20 (12 mol %) OH + Comprehensive Inorganic Chemistry; Bailar, J. C., Jr., Emeléus, H. R2 N Me Me NMI (12 mol %), 3 Å MS R2 NH O J., Nyholm, R., Trotman-Dickenson, A. F., Eds.; Pergamon Press, PhMe–CH2Cl2 (1:1) 1 1 1973; Vol. 3, p 553. H R –10 or –20 oC, 18 or 48 h R OMe Me Me (4) Hu bert-Pfalzgraf, L. G. Niobium and Tantalum: Inorganic and 1 40–86%, 91–99% ee R = Ph, 4-ClC6H4, 4-MeOC6H4, 1-Np, 2-Np, thien-3-yl, 2-Fur Coordination Chemistry. In Encyclopedia of Inorganic Chemistry, 2 R = H, CF3 King, R. B., Ed.; Wiley, 1996; Vol. 3, p 2444. NMI = N-methylimidazole (5) Wigley, D. E. Niobium and Tantalum: Organometallic Chemistry. In i-Pr NMI OMe O Encyclopedia of Inorganic Chemistry, King, R. B., Ed.; Wiley, 1996; Me Vol. 3, p 2462. O O O Nb Nb O O O (6) Cardin, D. J. Niobium. In Dictionary of Organometallic Compounds, Me 2nd ed.; Chapman and Hall, 1995; Vol. 3. O MeO NMI i-Pr (7) Hatchett, C. Phil. Trans. R. Soc. London 1802, 92, 49. (8) Rose, H. Pogg. Ann. 1844, 63, 317. 21 eq 20 (Ref. 47) (9) Greenwood, N. N. Catal. Today 2003, 78, 5. (10) (a) Tanabe, K.; Okazaki, S. Appl. Catal., A: Gen. 1995, 133, 191. (b) Da Silva, C. L. T. Síntese e Caracterização de Óxido de Nióbio

H2O2 and of asymmetric catalysis by a niobium complex in aqueous Ancorado Sobre Alumina e Avaliação de suas Propriedades como medium. Suporte de Catalisadores de HDT. M.S. Thesis, UFRJ, Rio de A chiral, Nb(V)-based Lewis acid effectively catalyzes the aza- Janeiro, Brazil, 1997. Diels–Alder reaction of imines with Danishefsky’s diene. The reaction (11) Albrecht, S.; Cymorek, C.; Eckert, J. Niobium and Niobium proceeds in high yield and high enantioselectivity for aromatic Compounds. In Ullman’s Encyclopedia of Industrial Chemistry and aliphatic imines, and has been applied to the total synthesis of [Online]; Wiley-VCH, 2011; DOI: 10.1002/14356007.a17_251.pub2 (+)-anabasine (eq 19).46 (accessed Feb 2012). In an earlier study, Kobayashi’s group had identified a novel (12) Fairbrother, F. The Chemistry of Niobium and Tantalum; Elsevier,

dinuclear chiral niobium catalyst, 21, formed from Nb(OMe)5 and 20. 1967. In the presence of a catalytic amount of this complex, asymmetric (13) Andrade, C. K. Z. Curr. Org. Synth. 2004, 1, 333. Mannich-type reactions proceed smoothly to afford the desired (14) Andrade, C. K. Z.; Rocha, R. A. Mini-Rev. Org. Chem. 2006, 3, adducts in high yields and with high enantioselectivities (eq 20).47 271. (15) (a) Yadav, J. S.; Bhunia, D. C.; Singh, V. K.; Srihari, P. Tetrahedron 4. Conclusion Lett. 2009, 50, 2470. (b) Leelavathi, P.; Ramesh Kumar, S. J. Mol. The recently disclosed reactions presented in this review have Catal. A: Chem. 2005, 240, 99. highlighted the efficiency and versatility of niobium compounds (16) Yadav, J. S.; Subba Reddy, B. V.; Eeshwaraiah, B.; Reddy, P. N. as catalysts and reagents in organic synthesis. Interest in niobium Tetrahedron 2005, 61, 875. compounds and their applications in chiral catalysis and total synthesis (17) Da Silva-Filho, L. C.; Lacerda, V., Jr.; Constantino, M. G.; da Silva, of natural products is growing steadily as evidenced by the increasing G. V. J.; Invernize, P. R. Beil. J. Org. Chem. 2005, 1, 14. number of research groups around the world, who are studying these (18) Da Silva-Filho, L. C.; Lacerda, V., Jr.; Constantino, M. G.; da Silva, compounds and developing new applications for them. G. V. J. Synthesis 2008, 2527. 27 VOL. 45, NO. 1 • 2012

(19) Constantino, M. G.; de Oliveira, K. T.; Polo, E. C.; da Silva, G. V. J.; Trademarks. Amberlyst® (Rohm and Haas Co.); Nafion® (E. I. du Pont Brocksom, T. J. J. Org. Chem. 2006, 71, 9880. de Nemours and Company, Inc.). (20) (a ) Arai, S.; Sudo, Y.; Nishida, A. Tetrahedron 2005, 61, 4639. (b) Polo, E. C.; da Silva-Filho, L. C.; da Silva, G. V. J.; Constantino, M. About the Authors G. Quim. Nova 2008, 31, 763. Valdemar Lacerda, Jr. was born in 1975 in Goiânia, GO, Brazil. (21) Constantino, M. G.; Lacerda, V., Jr.; Invernize, P. R.; da Silva-Filho, He received a B.Sc. degree in chemistry in 1997 from the Federal L. C.; da Silva, G. V. J. Synth. Commun. 2007, 37, 3529. University of Goiás, where he worked in the laboratory of Professor (22) Oh, K.; Knabe, W. E. Tetrahedron 2009, 65, 2966. Pedro Henrique Ferri. He received his M.Sc. degree in 2000 and (23) Narsaiah, A. V.; Sreenu, D.; Nagaiah, K. Synth. Commun. 2006, 36, his Ph.D. degree in 2004 from São Paulo University, Ribeirão 3183. Preto, working with Professor Mauricio Gomes Constantino in (24) Wang, R.; Li, B.-g.; Huang, T.-k.; Shi, L.; Lu, X.-x. Tetrahedron Lett. organic synthesis and NMR studies. In 2004, he began working as 2007, 48, 2071. a postdoctoral researcher at the NMR laboratory coordinated by (25) Majhi, A.; Kim, S. S.; Kim, H. S. Appl. Organomet. Chem. 2008, 22, Professor Gil Valdo José da Silva. In 2006, he joined the department 466. of chemistry of the Federal University of Espirito Santo (ES State, (26) (a ) Yadav, J. S.; Narsaiah, A. V.; Reddy, B. V. S.; Basak, A. K.; Brazil) as an associate professor. His current research interests focus Nagaiah, K. J. Mol. Catal. A: Chem. 2005, 230, 107. (b) Yadav, J. S.; on organic synthesis, NMR studies, theoretical calculations, and Narsaiah, A. V.; Basak, A. K.; Goud, P. R.; Sreenu, D.; Nagaiah, K. petroleum studies. He has been Head of the Department of Chemistry J. Mol. Catal. A: Chem. 2006, 255, 78. since 2007, and is presently also a CNPq level 2 researcher. (27) Gao, S.-T.; Zhao, Y.; Li, C.; Ma, J.-J.; Wang, C. Synth. Commun. Deborah A. dos Santos was born in 1989 in Cachoeiro de 2009, 39, 2221. Itapemirim, ES, Brazil. She received her B.Sc. degree in chemistry (28) Nagaiah, K.; Reddy, B. V. S.; Sreenu, D.; Narsaiah, A. V. ARKIVOK in 2011 from the Federal University of Espírito Santo, where she 2005 (iii), 192. worked on organic synthesis projects in the research laboratory (29) Yadav, J. S.; Ganganna, B.; Bhunia, D. C.; Srihari, P. Tetrahedron of Professor Valdemar Lacerda, Jr. Currently, she is studying for Lett. 2009, 50, 4318. her M.Sc. degree, and continues to work in Professor Lacerda’s (30) (a ) Shimada, N.; Hanari, T.; Kurosaki, Y.; Anada, M.; Nambu, H.; laboratory. Hashimoto, S. Tetrahedron Lett. 2010, 51, 6572. (b) Sudo, Y.; Arai, Luiz Carlos da Silva-Filho was born in 1979 in Itapetininga, S.; Nishida, A. Eur. J. Org. Chem. 2006, 752. (c) Arai, S.; Sudo, Y.; SP, Brazil. He received his B.S. (2001) and Ph.D. (2006) degrees Nishida, A. Synlett 2004, 1104. from São Paulo University, Ribeirão Preto, where he worked in (31) Barbosa, S. L.; Dabdoub, M. J.; Hurtado, G. R.; Klein, S. I.; Baroni, organic synthesis in the laboratory of Professor Mauricio Gomes A. C. M.; Cunha, C. Appl. Catal., A: Gen. 2006, 313, 146. Constantino. In 2006, he began postdoctoral work in the Laboratory (32) Barbosa, S. L.; Hurtado, G. R.; Klein, S. I.; Lacerda, V., Jr.; Dabdoub, of Structural Biology and Zoochemistry coordinated by Professor M. J.; Guimarães, C. F. Appl. Catal., A: Gen. 2008, 338, 9. Mario Sergio Palma at São Paulo State University, and, in 2008, he (33) Braga, V. S.; Barros, I. C. L.; Garcia, F. A. C.; Dias, S. C. L.; Dias, was appointed Assistant Professor in the Department of Chemistry J. A. Catal. Today 2008, 133–135, 106. at São Paulo State University. His current research interests are (34) Ar onne, A.; Turco, M.; Bagnasco, G.; Ramis, G.; Santacesaria, E.; focused on organic synthesis and on the development and application di Serio, M.; Marenna, E.; Bevilacqua, M.; Cammarano, C.; Fanelli, of niobium compounds as catalysts for organic reactions. E. Appl. Catal., A: Gen. 2008, 347, 179. Sandro J. Greco was born in Rio de Janeiro, RJ, Brazil. He (35) De Sairre, M. I.; Bronze-Uhle, E. S.; Donate, P. M. Tetrahedron Lett. received his B.Sc. degree in chemistry in 1997 and his M.Sc. and Ph.D. 2005, 46, 2705. degrees in 2001 and 2005 from the Fluminense Federal University (36) De Araújo Gonçalves, J.; Ramos, A. L. D.; Rocha, L. L. L.; Domingos, (Rio de Janeiro, Brazil), working under the guidance of Professor A. K.; Monteiro, R. S.; Peres, J. S.; Furtado, N. C.; Taft, C. A.; Sergio Pinheiro on studies of the use of terpenes and terpenoids in Aranda, D. A. G. J. Phys. Org. Chem. 2011, 24, 54. the enantioselective synthesis of potential anticholinergic agents, and (37) Somma, F.; Canton, P.; Strukul, G. J. Catal. 2005, 229, 490. on the synthesis of amino alcohol based, new chiral phase-transfer (38) Carniti, P.; Colonna, S.; Gervasini, A. Catal. Lett. 2010, 137, 55. catalysts. In 2006, he joined Professor Maria D. Vargas’s group at the (39) Obora, Y.; Takeshita, K.; Ishii, Y. Org. Biomol. Chem. 2009, 7, 428. Fluminense Federal University as a postdoctoral researcher to work (40) Obora, Y.; Satoh, Y.; Ishii, Y. J. Org. Chem. 2010, 75, 6046. on the synthesis and pharmacological evaluation of new anticancer (41) Obora, Y.; Kimura, M.; Ohtake, T.; Tokunaga, M.; Tsuji, Y. drugs containing the ferrocenyl group. He is currently an associate Organometallics 2006, 25, 2097. professor of organic chemistry at the Federal University of Espírito (42) Obora, Y.; Kimura, M.; Tokunaga, M.; Tsuji, Y. Chem. Commun. Santo, with research interests in the design and synthesis of potential 2005, 901. bioactive compounds and the development of new organocatalysts (43) Tagusagawa, C.; Takagaki, A.; Hayashi, S.; Domen, K. J. Am. Chem. and chiral phase-transfer catalysts for asymmetric synthesis. Soc. 2008, 130, 7230. Reginaldo B. dos Santos was born in Matão, Brazil, and (44) Ar ai, K.; Lucarini, S.; Salter, M. M.; Ohta, K.; Yamashita, Y.; obtained his B.Sc. degree in chemistry in 1986 from the Federal Kobayashi S. J. Am. Chem. Soc. 2007, 129, 8103. University of São Carlos (SP State, Brazil). He then received his (45) Egami, H.; Oguma, T.; Katsuki, T. J. Am. Chem. Soc. 2010, 132, M.Sc. and Ph.D. degrees at the same University in 1990 and 1995, 5886. working under the supervision of Professor U. Brocksom in the (46) Jurcik, V.; Arai, K.; Salter, M. M.; Yamashita, Y.; Kobayashi, S. Adv. field of organic synthesis. In 1991, he was appointed Assistant Synth. Catal. 2008, 350, 647. Professor in the Department of Chemistry at the Federal University (47) Kobayashi, S.; Arai, K.; Shimizu, H.; Ihori, Y.; Ishitani, H.; of Espirito Santo (ES State, Brazil), and was promoted to Associate Yamashita, Y. Angew. Chem., Int. Ed. 2005, 44, 761. Professor in 1995. Looking for a diazoacetate?

E22201 – contains ≤15% 480754 – contains <10% dichloromethane dichloromethane 752150 – 15% solution 752169 – 15% solution in toluene in toluene

Diazoacetates have been widely employed for numerous organic transformations. Despite their utility, process chemists have been concerned with safety issues associated with the use of diazoacetates at large scale. Now, through advancements in our R&D and production facilities, we have expanded our diazoacetate portfolio and capacity to help 752177 – contains ≤15% advance your research. dichloromethane Now available as 15% solutions in toluene, which are safe 752185 – 15% solution • in toluene for use at process scale1 • Readily available for bulk scale-up • Aldrich can synthesize other diazoacetate products on a custom basis

References: (1) (a) Clark, J. D. et al. Org. Process Res. Dev. 2004, 8, 176. (b) Anthes, R. et al. Org. Process Res. Dev. 2008, 12, 168.

Advance your research. Add Aldrich. Aldrich.com/eda

78096

78096_ALD EDA Ad (Acta 45.1).indd 1 3/28/2012 11:59:08 AM Easily activated Palladium Precatalysts for facile C–N and C–C bond formations.

Use of XPhos-Pd-G2 in the Suzuki coupling of 2,6-difluoroboronic acids

F F XPhos-Pd-G2 B(OH) 2 Cat. No.: 741825 Ar R + Ar-X R rt , 30 min F F R = H, 4-F, 3-OnBu

F F F F iPr Buchwald palladium precatalysts are highly reactive, air-stable N N Me crystalline white solids that generate the active monoligated F F F F MeO S N Pd(0) species in situ, providing a more efficient and easily X = Cl X = OTf X = Cl X = Cl 95% 89% 99% 77% accessible catalytic system.

n F F OnBu O Bu F F F F G1 = 1st Generation Precatalysts (Aliphatic Amine) O H N N F 2 F G2 = 2nd Generation Precatalysts (2-Aminobiphenylamine) O F OMe O F MeO

X = Cl X = Cl X = Br X = Cl 95% 96% 45% 97% NH NH2 NH NH Pd 2 Pd Pd 2 Pd 2 t t Cl Cl BuOCH3 Cl BuOCH3 t Cl Cy2P i Cy2P Cy2P i Bu2P i Pr OCH3 O Pr Pr

References: iPr iPr (1) Surry, D. S.; Buchwald, S. L. Chem. Sci. 2011, 2, 27. (2) Martin, R.; Buchwald, S. L. Acc. i i i Chem. Res. 2008, 41, 1461. (3) Surry, D. S.; Buchwald, S. L. Angew. Chem. Int. Ed. 2008, 47, Pr H3CO PrO Pr 6338. (4) Kinzel, T.; Zhang, Y.; Buchwald, S. L. J. Am. Chem. Soc. 2010, 132, 14073. XPhos-Pd-G1 SPhos-Pd-G1 RuPhos-Pd-G1 t-BuXPhos-Pd-G1 704954 704946 707589 708739

NH Pd 2 t NH2 NH2 NH2 Cl BuOCH3 Pd Pd Pd Cy2P i H3CO Pr Cl Cl Cl Cy2P i Cy2P Cy2P i Pr OCH3 O Pr iPr For inquiries about bulk quantities, iPr i CH O Pr 3 i i contact [email protected]. Pr H3CO PrO BrettPhos-Pd-G1 XPhos-Pd-G2 SPhos-Pd-G2 RuPhos-Pd-G2 Aldrich.com/buchwald 718750 741825 753009 753246

78376

78376_ALD Buchwald Palladacycles Ad (Acta 45.1)_R.indd 1 4/5/2012 11:46:36 AM P.O. Box 14508 St. Louis, MO 63178 USA

Professor Bruce H. Lipshutz Winner of the 2012 EROS Best Reagent Award

Aldrich Chemistry and John Wiley, About the Reagent proud sponsors of the annual EROS Lipshutz and co-workers have developed a new method for the Best Reagent Award, congratulate this asymmetric hydrosilylation of various carbonyl compounds using year’s winner, Professor Bruce Lipshutz. a preformed DTBM-SEGPHOS®-CuH species in the presence of The award was created to honor excess polymethylhydrosiloxane (PMHS), with high yields and ee’s outstanding contributors to the online reported for the reaction products. The resulting chiral building edition of Encyclopedia of Reagents for blocks are valuable precursors for the synthesis of various drugs Organic Synthesis (EROS and e-EROS). and naturally occurring molecules. (Lipshutz, B. H. et al. Org. Lett. 2006, 8, 2969.) About Prof. Bruce Lipshutz OMe OMe t-Bu t-Bu t-Bu t-Bu t-Bu Bruce Lipshutz has been on the faculty t-Bu OMe OMe at UC Santa Barbara for the past 33 O O years. From his early contributions in O P t-Bu O P t-Bu the form of reagents such as SEM-Cl Cu-H Cu-H and higher order cyanocuprates to O P t-Bu O P t-Bu heterogeneous catalysts in the form O OMe O OMe of nickel- and copper-in-charcoal, his t-Bu t-Bu t-Bu t-Bu t-Bu t-Bu research focus has been on providing OMe OMe technologies that are broadly applicable 762458 762938 to synthetic problems. More recently, he has turned his attention to “green” chemistry, with the goal of developing micellar catalysis as an environmentally benign way of carrying out transition-metal-catalyzed cross-coupling reactions in water at room temperature.

Advance your research. Add Aldrich. Aldrich.com/catalysis

OEA SEGPHOS is a registered trademark of Takasago Intl. Corp. 77777-511700 78287 1032

78287_Lipshutz_EROS_2012_Acta.indd a 3/28/2012 12:16:46 PM Page intentially blank Page intentially blank VOL. 45, NO. 2 • 2012

Cp*Rh-Catalyzed C–H Activations: Versatile M ICHI ICA A DR C L TA A

Dehydrogenative Cross-Couplings of C 2 C–H Positions with sp

2 8 th 0

6

1

9 2 Olefins, Alkynes, and Arenes 1 4 A N 5 Y R Opening New Chemical Space through Novel N I V E R S A Dearomatization Reactions Looking for more efficient cross-coupling?

The Pd-N-heterocyclic carbene (NHC) based precatalyst featuring bis[(N,N-(2,6-diisopropylphenyl)]imidazolium (Pd-PEPPSI™-IPr) has been shown to be effective in Suzuki– Miyaura, Negishi, and Kumada–Tamao–Corriu couplings. The second-generation NHC catalyst Pd-PEPPSI™-IPent, where the ligand is replaced by the more reactive bis[N,N-(2,6-di(3-pentyl)phenyl)]imidazolium, has been further examined in catalyzed aminations and sulfinations. Pd-PEPPSI™-IPr catalyst Pd-PEPPSI™-IPent catalyst • Bulky NHC ligand 669032 732117 • Good reactivity and excellent conversions in couplings involving sp, sp2, and sp3 centers 669032 R-X + R1-M R-R1 • Best performance and mildest conditions in amination or or 732117 or and sulfination catalysis R-X + R1-SH R-SR1 or additives or R-X + HNR1R2 R-NR1R2

1 2 R, R , R = Csp, Csp2, Csp3

References: ( 1) Valente, C.; Çalimsiz, S.; Hoi, K. H.; Mallik, D.; Sayah, M.; Organ, M. G. Angew. Chem. Int. Ed. 2012, 51, 3314. (2) Hadei, N.; Achonduh, G. T.; Valente, C.; O’Brien, C. J.; Organ, M. G. Angew. Chem. Int. Ed. 2011, 50, 276. (3) Çalimsiz, S.; Organ, M. G. Chem. Commun. 2011, 47, 5181. (4) Hoi, K. H.; Çalimsiz, S.; Froese, R. D. J.; Hopkinson; A. C.; Organ, M. G. Chem.—Eur. J. 2011, 17, For a complete list of catalysts, visit 3086. (5) Çalimsiz, S.; Sayah, M.; Mallik, D.; Organ, M. G. Angew. Chem. Int. Ed. 2010, 49, 2014. (6) Dowlut, M.; Mallik, D.; Organ, M. G. Chem.—Eur. J. 2010, 15, 4279. (7) Organ, M. G.; Çalimsiz, Aldrich.com/catalysis S.; Sayah, M.; Hoi, K. H.; Lough, A. J. Angew. Chem. Int. Ed. 2009, 48, 2383.

PEPPSI is a trademark of Total Synthesis Ltd. 79062

79062 PEPPSI Catalysis Ad for Acta Vol. 45, No. 2.indd 1 8/2/2012 11:18:21 AM 29 VOL. 45, NO. 2 • 2012

Aldrich Chemical Co. LLC “PLEASE BOTHER US.” 6000 N. Teutonia Ave. Milwaukee, WI 53209, USA Dear Fellow Chemists,

To Place Orders A number of you have suggested we offer the two copper complexes shown below. We are now happy to announce that we are making them available to Telephone 800-325-3010 (USA) the research community. These complexes were developed very recently by FAX 800-325-5052 (USA) Dr. Peter Maligres at Merck Research Laboratories as effective catalysts (superior or 414-438-2199 reactivity and more reproducible results) for copper-catalyzed C–N and C–O Mail P.O. Box 2060 bond-forming reactions. They offer the added advantage of being stable, Milwaukee, WI 53201, USA nonhygroscopic, and easy-to-handle solids that are soluble in organic solvents. Customer & Technical Services Maligres, P. E.; Krska, S. W.; Dormer, P. G. A Soluble Copper(I) Source and Stable Salts of Volatile Ligands for Copper-Catalyzed C–X Couplings. J. Org. Chem. 2012, 77, submitted for publication. Customer Inquiries 800-325-3010 Me Technical Service 800-231-8327 O– SAFC® 800-244-1173 Me I– Me + N O Cu2+ Custom Synthesis 800-244-1173 CuI Me Flavors & Fragrances 800-227-4563 2 Me Me 2

International 414-438-3850 762504762504 762431762431 24-Hour Emergency 414-438-3850 762504 Bis[(tetrabutylammonium iodide)copper(I) iodide] 1 g Website sigma-aldrich.com 5 g Email [email protected] 762431 Bis(2-isobutyrylcyclohexanone)copper(II) 500 mg 2 g General Correspondence Editor: Sharbil J. Firsan, Ph.D. We welcome fresh product ideas from you. Do you need a compound that isn’t listed in our P.O. Box 2988, Milwaukee, WI 53201, USA catalog? Ask Aldrich! For over 60 years, your research needs and suggestions have shaped the [email protected] Aldrich product offering. To submit your suggestion, visit Aldrich.com/pleasebotherus.

Subscriptions Request your FREE subscription to the John Radke Aldrichimica Acta, through: Director of Marketing, Chemistry Web: Aldrich.com/acta TABLE OF CONTENTS Email: [email protected] Cp*Rh-Catalyzed C–H Activations: Versatile Dehydrogenative Cross-Couplings of

Phone: 800-325-3010 (USA) Csp2 C–H Positions with Olefins, Alkynes, and Arenes...... 31 Frederic W. Patureau,* Joanna Wencel-Delord, and Frank Glorius,* Technische Universität Mail: Attn: Marketing Services Kaiserslautern and Westfälische Wilhelms-Universität Münster Sigma-Aldrich Co. LLC 3050 Spruce Street Opening New Chemical Space through Novel Dearomatization Reactions...... 45 Daniel P. Harrison and W. Dean Harman,* University of Virginia St. Louis, MO 63103 ABOUT OUR COVER A View on a High Road (oil on canvas, 93.1 × 127.8 cm) was painted in 1665 by Meindert Hobbema (Amsterdam, 1638– 1709), one of the finest Dutch landscape painters. Few details about his personal and professional lives are known, save for the fact that his painting career seems to have lasted about a dozen years, from ca. 1658 to ca. 1669, and that he was an apprentice of another famous Dutch landscape painter, Jacob The entire Aldrichimica Acta archive is also available free of van Ruisdael. Following his marriage in 1668, he accepted a charge at Aldrich.com/acta. steady-pay job as a minor customs official gauging imported Detail from A View on a High Road. Photograph © Board Aldrichimica Acta (ISSN 0002-5100) is a publication of Aldrich. wines, and, as a consequence, painted infrequently thereafter. of Trustees, National Gallery of Art, Washington. Aldrich is a member of the Sigma-Aldrich Group. He, like several of his contemporary Dutch painters, was not appreciated and did not profit much from his artwork during his lifetime, lived on the edge of poverty, and died destitute. It wasn’t until over a century later, ©2012 Sigma-Aldrich Co. LLC. All rights reserved. SIGMA, chiefly in nineteenth-century England, that his work gained a wide appeal and his talent was finally recognized. SAFC, SIGMA-ALDRICH, ALDRICH, and SUPELCO are trademarks Hobbema specialized in landscape painting, in particular picturesque and serene rural scenes that draw the of Sigma-Aldrich Co. LLC, registered in the US and other viewer in, as exemplified by the painting featured here (for another example, see Aldrichimica Acta, Vol. 32, No. countries. Add Aldrich is a trademark of Sigma-Aldrich Co. LLC. Aldrich brand products are sold through Sigma-Aldrich, Inc. 1). His landscapes tend to have many elements in common: sky with billowing clouds penetrated by light that Purchaser must determine the suitability of the product(s) illuminates parts of the composition, abundance of trees and foliage rendered with exquisite detail, country for their particular use. Additional terms and conditions may roads and buildings, placid waters, and human subjects resting or going about their daily business. His patient apply. Please see product information on the Sigma-Aldrich attention to detail, the subdued tones and dark greens—prominent in this work and necessitated by the nature website at www.sigmaaldrich.com and/or on the reverse side of the scene and placement of the viewer—are also characteristic of his painting style. of the invoice or packing slip. This painting is part of the Andrew W. Mellon Collection at the National Gallery of Art, Washington, DC. X DG DG R5 Searching for the R1 H H R1 X R4 2 2 R R4 R5 R right catalyst for R3 R3 * [Cp RhCl2]2 AgSbF6 R1 DG 1 H your C–H activation? R DG Cu(Ac)2 X R2 2 X R H R3 R4 R3 R4

R1 DG R1 DG R3 H R2 R2 H R3

Me Activation of C–H bonds has emerged as a challenging MeO yet attractive tool for catalysis. Among its uses, catalytic N(i-Pr) N(i-Pr) 2 2 N(i-Pr)2 dehydrogenative cross-coupling has found increasing interest, O allowing C–C bond construction in an elegant way.1 Although O O palladium has been the metal of choice for most examples, Me Br F C Br several recent reports have shown rhodium to be a suitable 3 Cl Br promoter for this activation. Using this metal, important 84% 76% 73% couplings such as aryl–aryl, aryl–alkene, and alkene–alkene have proven to be viable routes to valuable organic frameworks O Me O Me 2-4 n-BuO (Scheme 1). NH2 Ph O O Tools for Rhodium-Catalyzed C–H Activation Et HN Me Me O N Ph CH3 Ph CF3 H3C CH3 AgSbF6 Me O H3C CH3 227730 Rh Cl 99% 59% 74% Cl Cl O O Cl Rh O H3C CH3 NH Me O Me 2 NH2 NH2 H3C CH3 2+ H3C O Cu Me Me CH3 2 On-Bu 686891, 338370 326755 SO2Ph Br O 47% 70% 72%

Scheme 1: Rhodium-catalyzed dehydrogenative cross-coupling examples.

References: For our complete (1) Yeung, C. S.; Dong, V. M. Chem. Rev. 2011, 111, 1215. (2) Besset, T.; Kuhl, N.; Patureau, catalysis portfolio, visit F. W.; Glorius, F. Chem.—Eur. J. 2011, 17, 7167. (3) Patureau, F. W.; Besset, T.; Glorius, F. Angew. Chem. Int. Ed. 2011, 50, 1064. (4) Wencel-Delord, J.; Nimphius, C.; Patureau, F. W.; Aldrich.com/catalysis Glorius, F. Angew. Chem. Int. Ed. 2012, 51, 2247.

79008

79008_C-H Catalysis Ad Acta Vol_45_2_v4.indd 1 8/2/2012 11:49:11 AM 31 VOL. 45, NO. 2 • 2012 Cp*Rh-Catalyzed C–H Activations: Versatile

Dehydrogenative Cross-Couplings of Csp2 C–H Positions with Olefins, Alkynes, and Arenes

Frederic W. Patureau,*,a Joanna Wencel-Delord,b and Frank Glorius*,b a Fachbereich Chemie Technische Universität Kaiserslautern Erwin-Schrödinger-Strasse, Geb. 52 67663 Kaiserslautern, Germany Email: [email protected] b Organisch-Chemisches Institut Prof. Frederic W. Patureau Dr. Joanna Wencel-Delord Prof. Frank Glorius Westfälische Wilhelms-Universität Münster Corrensstrasse 40 48149 Münster, Germany Email: [email protected]

2 Keywords. C–H activation; rhodium; [Cp*RhCl2]2; Cp*Rh; troublesome prefunctionalization of either reaction partner. The dehydrogenative cross-coupling; cross-dehydrogenative coupling field emerged in the late 1960s with the groundbreaking discovery (cdc); directing group. of the Pd-catalyzed dehydrogenative olefination of benzene by Fujiwara and Moritani3—a full three years before the discovery of the 4 Abstract. The Cp*Rh(III)-catalyzed Csp2 C–H activation and its now famous Mizoroki–Heck reaction. However, it was not until the subsequent cross-coupling with alkenes, alkynes, and arenes is a mid-to-late 1990s, and the key reports by Murai,5 Miura and Satoh,6 rapidly evolving research field. Many different directing groups, and Fujiwara3b,c himself, that C–H activation was truly accepted such as heterocycles, ketones, and carboxylic acid derivatives, can as a credible strategy in organic synthesis. Since then, many great efficiently be used for this transformation; even undirected C–H contributions have been made to this field, mostly relying on Pd-, activations have been reported recently. Ru-, and Rh-based catalysts.1 The number of such contributions is growing exponentially, as evidenced by the many excellent reviews Outline that have been written on this topic since 2007. We do not wish 1. Introduction to cover here all of these very recent and very well documented 2. Dehydrogenative Olefination Reactions results; we propose instead to selectively survey the Rh-catalyzed 2.1. Dehydrogenative Olefination of Aromatic Ortho C–H Bonds C–H activation and dehydrogenative cross-coupling reactions, 2.2. Dehydrogenative Olefination of Vinylic C–H Bonds which have been reported very recently by us and others.7 We wish 3. Versatile Cyclization Reactions to provide the reader with an easy-to-understand global perspective 3.1. Synthesis of Indoles and Pyrroles on the field of Rh-catalyzed C–H activation dehydrogenative cross- 3.2. Synthesis of Indenols and Annulated Fulvenes couplings, with the authors’ analysis of the associated advantages 3.3. Synthesis of Other Heterocycles and disadvantages, and their future impact on the field of organic 4. Nonchelate-Assisted C–H Activation Followed by Dehydrogenative chemistry. Cross-Coupling 4.1. Dehydrogenative Olefination of Aromatic C–H Bonds 2. Dehydrogenative Olefination Reactions 4.2. Dehydrogenative Arylation of Aromatic C–H Bonds 2.1. Dehydrogenative Olefination of Aromatic Ortho C–H 4.3. Dehydrogenative Arylation of Vinylic C–H Bonds Bonds 5. Conclusion and Outlook The construction of C–C bonds, especially the coupling of arenes 6. Acknowledgements with olefinic building blocks, has kept the scientific community 7. References and Notes actively involved for the last 50 years. The literature relating to this broad topic is extensive, and it is likely to remain a hot area of 1. Introduction research in the future, because it offers such a powerful array of The direct catalytic activation and synthetic utilization of C–H versatile tools for the rapid assembly of complex and relevant organic bonds has become an increasingly important synthetic strategy structures. Pentamethylcyclopentadienylrhodium(III) chloride dimer 1 ® for the atom-economical construction of C–C bonds. Among (CAS Registry Number 12354-85-7, [Cp*RhCl 2]2, or simply Cp*Rh) such transformations, those categorized as “cross-dehydrogenative was first (accidentally) prepared by Haszeldine in 1967, by the ring couplings” (cdc) are particularly interesting, because they oxidatively rearrangement of Dewar’s hexamethylbenzene in the presence of 8 couple two different C–H positions, obviating the need for the often- RhCl3•3H2O in aqueous methanol. However, the correct structure was Cp*Rh-Catalyzed C–H Activations: Versatile Dehydrogenative Cross-Couplings of Csp2 C–H Positions 32 with Olefins, Alkynes, and Arenes Frederic W. Patureau,* Joanna Wencel-Delord, and Frank Glorius*

assigned to this compound only when Kang and Maitlis reproduced exclusive domain of the (long-established) Pd(OAc)2 precatalysts, but 9 the experiment in 1968, which is its earliest mention in the literature. that [Cp*RhCl2]2-derived complexes would also be applicable, and, in

The compound remained under-investigated until the last decade or some cases, superior to Pd(OAc)2 (eq 1). so. Maitlis eventually found out in 1987 that it could cyclometallate The Rh-catalyzed dehydrogenative olefination of arylpyrazoles benzoic acids under certain conditions.10 However, its application in (e.g., 1) described by Satoh and Miura did not immediately catch the catalytic dehydrogenative cross-couplings only started in 2002 with attention of the chemistry community because the substrate scope the pioneering report by Matsumoto, Yoshida, and co-workers.11 is limited (pyrazoles cannot be easily opened or cleaved), and the Then, between 2007 and 2009, Miura, Satoh, and co-workers showed selectivity of the reaction is poor, requiring that the loading of the in four key reports12 that the Fujiwara–Moritani reaction was not the olefin coupling partner be biased in order to limit the formation of the di-olefinated product.12d With the benefit of hindsight, it becomes clear without a doubt that this reaction is a considerable breakthrough in terms of reactivity. Shortly afterwards (2010), we reported on the N N dehydrogenative olefination of aniline derivatives (acetanilides), a N [Cp*RhCl2]2 (0.5 mol %) N H + Ph Ph very important and broad substrate class since the aniline moiety is Cu(OAc)2•H2O (1 mmol) 13 DMF, 60 oC, 3 h ubiquitous in natural and other synthetic targets (Scheme 1). It is 2 interesting to note that the structurally analogous phenol derivatives, 1 (1 mmol) (0.5 mmol) 3, 81% phenyl acetates, are not reactive at all under these conditions. This + Monoarylation: 57–81% (10 examples) problem of lack of reactivity of phenol derivatives was overcome by Heterodiarylation: 55–70% (5 examples) N N Lei Liu and, independently, by Teck-Peng Loh, who astutely appended Ph Ph a carbamate to the phenol substrates, thus sufficiently enhancing their planar, as well as electron-rich character, to enable the dehydrogenative 14 4, 7% olefination to proceed (see Scheme 1). eq 1 (Ref. 12d) Fagnou showed, through a series of deuteration experiments, that, when acetanilides are coupled with alkynes, they undergo true C–H activation with Cp*Rh.15 This reaction mode became obvious when we discovered that versatile benzamides and phenones in general were even more potent and selective substrates in the dehydrogenative 16 (a) Dehydrogenative Olefination of Acetanilides, Acetophenones, and Benzamides olefination reaction (see Scheme 1a). This reactivity renders common Glorius (2010 & 2011), (Ref. 13,16a) electrophilic substitution-type mechanisms less likely because of the intrinsic electron-withdrawing character of these classes of substrate DG [Cp*RhCl2]2 (0.5 mol %) DG AgSbF (2 mol %) 16a H 6 R (Scheme 2). Therefore, it follows that the reaction should be Cu(OAc)2 (2.1 equiv) + R initiated by a true C–H activation event: The chelate assistance of the tert-amyl alcohol 120 oC, 16 h directing group guides the Rh intermediate to approach at the proper 1 mmol 1.5 mmol 41 examples angle and distance in order to interact with the C–H bond, and, hence, Me Me result in its cleavage. While the so-called “base-assisted metallation” Et2N O Et N O HN O HN O 2 Me On-Bu transition state, in which a carboxylate moiety participates in the Ph Ph 17 O deprotonation–metallation step, could be invoked here, isotopic competition and scrambling experiments in this case tend to favor the F3C 80% 65% 83%76% (E:Z = 81:19)a idea of high-oxidation-state hydride intermediates being involved (see Fagnou’s isotopic experiments, Section 3.1).15 In the latter scenario, Me O Me O O Et N O Me O the reductive elimination of hydride with, presumably, a carboxylate 2 On-Bu Ph On-Bu ligand occurs subsequently to the C–H activation step and leads to a Ph O Ac N Rh(III) metallacycle intermediate. Furthermore; as shown recently by Me Bergman and Ellman, Zhang-Jie Shi, and Chao-Jun Li; carboxylates Et Ph 7c-g 49% (E:Z = 92:8)a 57% 72% 99% are not always essential for Cp*Rh C–H activation reactivity, which supports a chronological distinction between the C–H activation and a [Cp*RhCl ] (2.5 mol %), 140 oC. 2 2 carboxylate protonation steps. In any case, the broad array of suitable

(b) Dehydrogenative Olefination of Carbamates of Phenols substrates, from eletron-rich acetanilides to electron-deficient Liu (April 2011), (Ref. 14a); Loh (July 2011), (Ref. 14b) benzamides and phenones, suggests that the acidity of the ortho C–H NMe2 NMe2 bond is only marginally relevant. In addition, we would like to point [Cp*RhCl ] (1–2.5 mol %) O O 2 2 O O out here that the mechanistic role of the Cu(OAc)2 salt, besides its AgSbF (4–10 mol %) H 6 R' Cu(OAc)2 (2 equiv) obvious oxidizing character, is largely unknown. Its replacement by R + R' R THP or DME analogous salts (either acetate-containing or Cu-containing) or organic o 110 C, 24 h 34–99% oxidants usually shuts down the reactivity. It is therefore sensible to assume that this component influences the reactivity through other specific modes of action that have not yet been elucidated. Related to this issue is the observation that substrates bearing an “internal Scheme 1. The Dehydrogenative Olefination of Ubiquitous Benzene oxidant” such as N-methoxybenzamides exhibit the reactivity without Derivatives as Reported by Frank Glorius, Lei Liu, and Teck-Peng Loh. Cu(OAc)2, but benefit from a new additive, CsOPiv or CsOAc. The presence of the internal oxidant allows for a somewhat broader olefin 33 VOL. 45, NO. 2 • 2012 scope, a lower reaction temperature, and, of course, perfect regio- and DG chemoselectivity, but is otherwise limited because the substrates have H DG to be pre-functionalized with a reactive N–O bond (Scheme 3, right [RhIII] side, R1 = Me).18 Control experiments by Fagnou (Guimond et al.) have R (a) shown that the C–H activation step occurs first (as opposed to N–O (c) bond activation),18c and is even reversible in the absence of a coupling III DG DG [Rh ] partner. The N–O bond cleavage must occur at a later stage, and is H [RhIII] presumably linked to the β-H elimination step. These observations R (b) shed some light on the specific role of the external oxidant Cu(OAc)2, whereby the Cu salt is closely linked to the β-H elimination event, R partly explaining the narrow specificity of this reaction component. (a) C–H activation. (b) Olefin insertion. (c) β-H Intriguingly, using an internal oxidant with the ability to chelate (see elimination and reoxidation by Cu(OAc)2 1 Scheme 3, left side, R = Piv), β-H elimination seems to get suppressed, Cp* ‡ 2+ + providing N-heterocycles presumably by reductive elimination from a DG Cp* DG [RhIII] O 18 [RhV] R 7-membered-ring rhodacycle. H H O

2.2. Dehydrogenative Olefination of Vinylic C–H Bonds Oxidative-addition-type C–H Simultaneous agostic The reactivity discussed in the preceding section transfers well activation would lead to a interaction and concerted V to vinylic C–H activations, and leads efficiently to a broad range Rh high-energy intermediate metallation–deprotonation (shown here). Cp*RhIII without the high-oxidation- 19 of complex, highly functionalized, linear dienes (eq 2). Several catalysts may sometimes state RhV hydride esteemed specialists have argued that this reactivity is self-evident; follow this pathway. intermediate. (Ref. 17) however, olefins are very versatile compounds indeed, and their Scheme 2. Postulated General Mechanism of the Dehydrogenative Olefination. use in such (high-temperature and acidic) coupling reactions is far (Different Modes of C–H Activation Are Possible; Only Two Are Depicted Here.) from trivial. Because of the diverse reactions that can take place at a (Ref. 16a) vinylic C–H position, it is not always obvious how to control the fate of a multiply functionalized reactive olefin. Numerous elegant reports O 5d,20 O have skillfully covered these topics; thus, we shall only mention 1 2 N R R H + R the specifics of this particular reaction. The reason acrylates and H acrylate-like derivatives are suitable substrates for C–H activation 1 1 and subsequent exclusively linear dehydrogenative olefination lies R = COt-Bu R = Me mostly in the fact that they react in this catalytic cycle faster than [Cp*RhCl2]2 (2.5 mol %) [Cp*RhCl2]2 (1.0 mol %) in their decomposition pathways through oligomerization and/or cyclization CsOPiv (30 mol %) CsOAc (30 mol %) olefination PivOH (20 mol %) MeOH, 60 oC, 3–16 h polymerization. It also lies in the fact that the products resulting anhyd. EtOH, 80 oC, 16 h from dehydrogenative olefination are remarkably resilient under the O O reaction conditions, given their highly functionalized character. In NH NH R2 R2 2 point of fact, the starting materials are not always detectable at the R end of the reaction. Given the breadth of this reaction—preparation R of unnatural amino acid precursors, highly functionalized cinnamic 43–88% 40–99% derivatives, such as b-substituted cinnamamides, and many 9 examples 34 examples different kinds of tri- and tetrasubstituted olefins, all prepared Scheme 3. The “Internal Oxidant Strategy” in Dehydrogenative Olefinations. by dehydrogenative cross-coupling in only one step from readily (Ref. 18a) accessible substrates—one quickly realizes its importance in synthesis. The yields and diastereoselectivities are typically moderate [Cp*RhCl2]2 (2.5 mol %) to fair, but many of the relevant examples shown would be quite DG AgSbF6 (10 mol %) DG Cu(OAc) (2.1 mmol) H 2 R difficult to prepare by any other means. To this we should add that the + R mechanism is probably similar to the general model (see Scheme 2), 1,4-dioxane, 120 oC, 16 h especially in the case of electron-poor acrylate derivatives, which 0.5 mmol 0.75 mmol 29 examples are unlikely to proceed to the metallacycle through anything other n-BuO O H2N O Et2N O Et2N O than a true C–H activation event. This is less obvious, however, in Ph Ph Ph Ph the case of more electron-rich acetamidoacrylate derivatives (amino Me Me Me Me Me acid precursors), where an electrophilic attack may not be excluded as Z:E = 45:55 Z:E = 79:21 initiating the catalytic cycle and otherwise leading to the same classes 54% 67%a 68% 65% of highly functionalized linear dienes. Me Et N O Et N O 2 2 On-Bu HN O Ph MeO Ph 3. Versatile Cyclization Reactions O 3.1. Synthesis of Indoles and Pyrroles Ph Ph O The first Cp*Rh-mediated C–H activation that is followed by a 43%b 49%b 75% (+12% of E isomer)a condensation or cyclization step was reported as early as 2007 and a 120 oC in tert-amyl alcohol. b At 140 oC. 2008 by none other than Miura and Satoh.12a–c It consists of the eq 2 (Ref. 19) oxidative coupling of benzoic acids with internal alkynes to produce Cp*Rh-Catalyzed C–H Activations: Versatile Dehydrogenative Cross-Couplings of Csp2 C–H Positions 34 with Olefins, Alkynes, and Arenes Frederic W. Patureau,* Joanna Wencel-Delord, and Frank Glorius*

mostly a variety of isocoumarin derivatives (see Section 3.3). Not long 15a [Cp*RhCl ] (2.5 mol %) after, in 2008, Fagnou published his innovative indole synthesis. 2 2 2 2 R H R AgSbF6 (10 mol %) O These seminal works resulted in a “Rh Rush” in the scientific Cu(OAc)2•H2O (2.1 equiv) (a) + R1 community. Acetanilides can be C–H activated (see Section 2.1) N Me o H t-AmOH,120 C, 1 h N and undergo insertion of internal alkynes followed by oxidative R1 O Me cyclization to produce a variety of substituted indoles as reported by 1 mmol 1.1 mmol 16 examples Fagnou and co-workers (Scheme 4).15a Through astute H/D scrambling Me Me n-Pr n-Hex experiments, Fagnou proved that the metallation of C–H is reversible, Ph even in the presence of the alkyne coupling partner. This suggests Ph n-Pr N N Ac N N S two very important things: (i) that the C–H activation step does Ac Ac Ac indeed go through a high-oxidation-state metal hydride intermediate 79% 72% 68% 71% (presumably RhV), hence the scrambling; and (ii) that, in this case, the metallation step is possibly not rate-determining, but rather the (b) D D 77% D subsequent alkyne insertion step is. As the authors point out, the sheer same Me D D D H D bulk of the doubly substituted internal alkyne may be responsible for O conditions O + Ph t-AmOH this unexpected behavior. While we were investigating analogous D N Me D N Me D N H ~50% conv H Pd-catalyzed transformations, we were impressed by this amazing D H H O Me transformation.21 We eventually came across a Cp*Rh system that 77% 26% would convert a series of protected β-amino acid precursors into same 21% Me the corresponding tri- and tetrasubstituted pyrroles with amazing Ha D O conditions O 22,23 + Ph simplicity. Depending on the substitution pattern and protecting t-AmOD MeO N Me MeO N Me MeO N groups, we discovered a way to steer the C–H activation from H ~50% conv H Hb D D O 2 Me an α-vinylic (sp ) C–H one in the case of nitrile-protected amino 21% 50% acids to a γ-allylic (sp3) C–H activation in the case of methyl ester protected amino acids, followed by coupling of the alkyne to produce the pyrrole (Scheme 5).22 The change of reactivity is exclusive and Scheme 4. Fagnou’s 2008 Indole Synthesis and the Proof of the Reversibility leads to very high regioselectivities. The key to the selectivity was of the C–H Activation Step under Catalytic Conditions. (Ref. 15a) identified through H/D exchange experiments which showed that both structures (ester and nitrile derivatives) first undergo vinylic C–H activation at the α position to give the initial Rh metallacycle

(a) α-Vinylic Csp2–H Activation (see Scheme 5, Structures A-1 and A-2). This species then rearranges CN NC Et to a second metallacycle (see Scheme 5, Structure B-2) in the case CN (a) α γ [Rh] H Ph Et of the ester-substituted unit (γ position). We should mention here H H Me Ph O NH N that α-amino acid precursors, such as methylacetamidoacrylate (see NHAc Ac Me also Section 2.2) and related substrates, were also found to react A-1 with internal alkynes under similar conditions (activation of the β (b) γ-Allylic Csp3–H Activation C–H position), leading to a number of analogous, highly substituted pyrroles and substantially broadening the scope and versatility of this CO2Me CO2Me MeO O reaction. These results were reported by Fagnou’s co-authors in a (a) α γ [Rh] H [Rh] 24 H H full paper in late 2010 (Stuart et al.) and independently by Glorius’s O NH NHAc H group in the first half of 2011.19 Me NHAc starting material A-2 B-2 3.2. Synthesis of Indenols and Annulated Fulvenes Ph Et The idea of combining phenones and internal alkynes came quite

H Et naturally to us in late 2010 in light of our findings on dehydrogenative olefination reactions (Section 2). Sure enough, we quickly established MeO2C N Ph that a large array of phenone derivatives would undergo C–H Ac activation, alkyne insertion, and a new step best described as an 25 NC R R Ph Me Ph electrophilic cyclization. The resulting 5-membered-ring closure O O O leads to a series of functionalized indenol derivatives in high MeO EtO MeO Me N Ph N Ph N Ph N Ph yields and regioselectivities (Scheme 6). It is worth mentioning that Ac Ac R Ac Ac this transformation is redox neutral, that is, no external oxidant is R = Et, 70% R = Et, 60% R = Me, 81% 31%b R = Ph, 72% R = Ph, 72% R = Bn, 74% required. However, Cu(OAc)2 is still essential for the reactivity, probably due to its unique and multiple, but still not clearly defined, a o [Cp*RhCl2]2 (2.5 mol %), AgSbF6 (10 mol %), Cu(OAc)2 (2.1 equiv), DCE, 120 C, 16 h. roles in the mechanisms of these transformations. Probably both b o [Cp*RhCl2]2 (5.0 mol %), AgSbF6 (20 mol %), Cu(OAc)2 (2.1 equiv), DCE, 140 C, 24 h. phenomena (transmetallation to release the active Rh catalyst and participation in the key intermediates of the cross-coupling reaction)

take place, as the Cu(OAc)2 could not be replaced by other analogous Scheme 5. Glorius’s 2010 Pyrrole Synthesis. (Ref. 22) Lewis acids. Furthermore, depending on the electronic properties of the substrates, dehydration steps would sometimes follow, mostly in 35 VOL. 45, NO. 2 • 2012

(a) Glorius (2011), (Ref. 25) R3–C≡C–Ph [Cp*RhCl2]2 1 R 1 (0.5 or 2.5 mol %) R OH O AgSbF6 (2 or 10 mol %) R2 R2 Ph H Cu(OAc)2 (2.1 equiv) α or γ protons 1,4-dioxane or PhCl R3 electron-donating –H2O 120–140 oC, 16 h groups R1 fulvenes 20 examples O R2 R3 Ph dehydration [Rh] –H O (–Hα or –Hγ) 2 C alkyne 1 C–H insertion R Hα activation O O [Cu] R2 [Rh] [Rh] R2 Ph

3 Ph R Hγ transmetallation D G regenerates [Cu] OAc Rh catalyst electrophilic 1 R O [Rh] activation–cyclization 1 R O [Cu] R2 Ph R2 Ph E R3 R3 F

1 R OH Me OH t-Bu OH

Ph Ph Ph Ph R2 Ph Ph Et Ph

1 2 R = t-Bu, 99% R = CF3, 90% 90% α route, 70% 1 2 R = 3,5-(CF3)2C6H3, 67% R = Br, 80% γ route, 77%

(b) Cheng (2011), (Ref. 27): with Cu(OAc)2•H2O

Me OH 91% Ph 24 examples Ph

Scheme 6. Synthesis of Indenols and Annulated Fulvenes. the presence of electron-donating substituents, leading to the related such as for example Miura & Satoh,12,30 Fagnou,31 Rovis,32 Xingwei fulvene derivatives in high yields. The synthesis of these classes of Li & Zhengyin Du,33 Chiba,34 Chen Zhu,35 and Glorius.16a,36 These relevant compounds had been described to some degree before, but cyclizations are either redox neutral (typical intramolecular Michael their preparation typically required systematic prefunctionalization addition induced by the nucleophilic character of the directing group) steps.26 This reaction, described in Scheme 6, is appealing because or oxidizing, in which case the ring closure is believed to proceed it is atom-economical, as no waste is formally produced, and allows through a reductive elimination. These two pathways are very case- the rapid buildup of structural complexity through a simple cross- dependent, and sometimes even occur at the same time. Because coupling reaction. Shortly after our report appeared, Chien-Hong each known example is almost a story in itself, we shall only show

Cheng and co-workers showed that the use of Cu(OAc)2•H2O prevents a selection of them to highlight again the power and versatility of the dehydration step, thus extending the scope of the reaction to the Cp*Rh catalysts in the construction of such, usually biologically electron-rich indenol derivatives, although in this case the reaction active, cyclic molecules (Figure 1). time must be kept short.27–29 4. Nonchelate-Assisted C–H Activation Followed by 3.3. Synthesis of Other Heterocycles Dehydrogenative Cross-Coupling As far as Cp*Rh cyclizations are concerned, electrophilic coupling 4.1. Dehydrogenative Olefination of Aromatic C–H Bonds partners (e.g., Michael acceptors, aromatic alkynes, etc.) tend to C–H activations of benzene derivatives in the absence of a chelating provoke cyclization steps, especially in the presence of protic or group are very challenging, and only a few methods exist to effect nucleophilic directing groups in the substrate (see indoles and pyrroles them.37 In one of their most impressive reports, Miura and Satoh in Section 3.1). The tandem C–H oxidative functionalization and described a method for performing the dehydrogenative olefination cyclization, typically involving the directing group, lead to a variety of benzoic acids, that involves Cp*Rh-catalyzed C–H activation of 5- and 6-membered-ring systems with backbones as interesting as followed by a directing-group-removing decarboxylation step, they are diverse. Many key players have observed this phenomenon, yielding unprecedented olefinated products that lack the original Cp*Rh-Catalyzed C–H Activations: Versatile Dehydrogenative Cross-Couplings of Csp2 C–H Positions 36 with Olefins, Alkynes, and Arenes Frederic W. Patureau,* Joanna Wencel-Delord, and Frank Glorius*

Ph O O Ph O O Ph N Ph N Ph O O OEt Ph Me EtO Ph 86% 76% 76% 90%

13 examples 11 examples 6 examples Miura & Satoh (2007), (Ref. 12a,b) Miura & Satoh (2009), (Ref. 30a)

Me O O N Ph O N Ph Me N N N

Ph Et OBn O 96% 82%, rr > 19:1 91% 86% 16 examples Rovis (May 2010), (Ref. 32a) Xingwei Li (2010) 19 examples Miura & Satoh (2010) Miura & Satoh (Apr 2010), (Ref. 30c) (Ref. 33b) Zhengyin Du & (Ref. 30b) Xingwei Li (Aug 2010), (Ref. 33a) Xingwei Li (2011) Guimond & Fagnou (Feb 2011), (Ref. 31) (Ref. 33d)

H Me Me O O N O N Ph NH O N On-Bu N OMe Ph Ph O Ph Me OEt 82% 78% 62% 75% 11 examples 17 examples 19 examples Miura & Satoh (2011) Miura & Satoh (2011) Glorius (2011) Glorius (2011) (Ref. 30d) (Ref. 30e) (Ref. 16a) (Ref. 36)

Figure 1. A Selection from 2007 to 2011 of Significant, Mainly Heterocyclic, Fused-Ring Systems That Have Been Accessible through Cp*Rh-Catalyzed C–H Activation–Cyclization.

ortho-directing functional groups (eq 3).16e In the summer of 2011, HO O 1. [Cp*RhCl2]2 (1 mol %) AgOAc (2–3 equiv) we stumbled upon what is probably the most significant Cp*Rh- MeO H DMAc, 120 oC, 8 h MeO Ph mediated reactivity discovered in our laboratory to date. While + Ph 2. AgOAc (2 equiv) the Pd-catalyzed C–H activation of highly biased heterocyclic 38 K2CO3 (2 equiv) 66% structures, and usually electron-deficient arenes, was the state of o 160 C, 4 h 11 examples the art in this field,11b,39 we discovered that bromoarenes could be C–H activated with Cp*Rh without the need for a directing group eq 3 (Ref. 16e) assisting through chelation.40 The bromoarene is believed to be the substrate, the terminal oxidant,41 and possibly a critical catalyst modifier. The lack of a directing group makes the reaction very slow (TOF = 1.3 h–1 at best), with the C–H activation step being Cp*Rh(SbF6)2 (5 mol %) 40 PivOH (1.1 equiv) (predictively) the rate-limiting step (kH/kD = 3.4, Scheme 7). Br R Cu(OAc)2 (2.2 equiv) While the products are typically obtained as poorly useful 2:1 Ar–Br as solvent (statistical) mixtures of meta- and para-bromostilbene derivatives, meta:para = 2:1 Ar–Br 140 oC this breakthrough clearly opens a whole new Cp*Rh era in terms 14 examples incubation: 6 h of C–H activation reactivity and selectivity (Sections 4.2 and 4.3), Ar–Br Ar–H Ar–OPiv Ar–Br without the structural limitations of a directing group. [Rh] β-H C–H activation reoxidation: active species elimination rate-limiting step: 4.2. Dehydrogenative Arylation of Aromatic C–H Bonds kH/kD = 3.4 H Shortly after the discovery of the potential of this Rh(III) catalyst to Br H Br [Rh] undirected perform a nonchelate-assisted C–H bond activation of bromoarenes, our collision R group focused its attention on applying this strategy to the development [Rh] of an unprecedented, Cp*Rh-catalyzed aryl–aryl cross-coupling. The Br [Rh] biaryl moiety is one of the key scaffolds in organic chemistry, and its olefin R insertion R construction by a two-fold C–H bond activation strategy is a highly 1 mmol valuable tool for modern and environmentally friendly chemistry. However, examples of this type of dehydrogenative cross-coupling Scheme 7. Nonchelate-Assisted C–H Activation and Dehydrogenative are still scarce and limited mainly to Pd-catalyzed transformations.1d,42 Olefination of Bromoarenes.(Ref. 40) Our efforts rapidly turned out to be highly rewarding when the aryl– aryl cross-coupling reaction between benzamides (or phenones) and 37 VOL. 45, NO. 2 • 2012 aromatic compounds bearing a halide substituent was observed.43 We Indeed, due to the directing-group-assisted insertion of the rhodium feel that this method is attractive and competitive for the formation into the vinylic C–H bond and the formation of a new C–C bond of regioselectively functionalized biaryls (ortho on one ring, meta on in the reductive elimination step, an exclusive Z-selective coupling the other). This original transformation displays some very interesting is observed. Even highly substituted alkenes can be employed as features when compared to the first report on the nonchelate-assisted substrates, leading to the formation of tri- and tetrasubstituted olefins. C–H bond activation (see Section 4.1): (i) This dehydrogenative cross- Even though this transformation still suffers from moderate-to-low coupling is compatible not only with bromoarene derivatives, but also yields, probably due to the versatility of the alkene starting material iodobenzene and chlorobenzene can be efficiently used as coupling under the reaction conditions (see Section 2.2), this is quite a unique partners. (ii) The undirected C–H bond activation event is more and simple way to access highly substituted Z olefins bearing additional selective: no detectable ortho functionalization (with regard to the sites for further functionalization (Scheme 9).45 halogen or other substituent on the aromatic ring) is observed and, The nonchelate-assisted Rh-catalyzed C–H bond activation is still therefore, application of appropriately selected bromo-disubstituted in its infancy and suffers from important limitations such as the need arenes leads to the selective formation of the desired biaryl products. (iii) for halide-substituted arenes, low selectivity (formation of mixtures of The nonchelate-assisted insertion of the Rh into the C–H bond seems meta and para products), and the requirement of a large excess of one of to be enhanced in the presence of the benzamide substrate;44 thus the the coupling partners (the haloarene). However, its compatibility with C–H activation/metallation of the benzamide part must occur first and electron-poor arenes clearly indicates its orthogonality with regard produce a more reactive Rh intermediate (compared to that described to the Pd-catalyzed arene–arene dehydrogenative cross-coupling in Section 4.1). Studies of the kinetic isotope effect clearly show that and other Pd-enabled cross-coupling reactions. The potential of this true C–H bond activation occurs for both coupling partners [kH/kD new reactivity may well lead to important breakthroughs in the near 46 (with regard to benzamide) ≈ 2.0; kH/kD (with regard to bromobenzene) future. ≈ 3.5]. Moreover, the important H/D scrambling on either benzamide or bromoarene coupling partner suggests the reversibility of both C–H 5. Conclusion and Outlook activation events and, again, the probable formation of high-oxidation- The versatility of this class of Cp*Rh-catalyzed cross-couplings has state rhodium hydride intermediates (Scheme 8).43 already gone, in an incredibly short time, far beyond what we had imagined when we first joined this field (Figure 2). The importance 4.3. Dehydrogenative Arylation of Vinylic C–H Bonds and diversity of the products and of the technology itself—improved Encouraged by the discovery of the ability of the Cp*Rh catalyst step by step by research groups from around the world—leave no to perform arylation reactions, we extended the scope of this doubt that dehydrogenative cross-coupling involving Cp*Rh will transformation to coupling partners consisting of vinylic substrates continue to garner the interest of the chemistry community for a bearing a directing group.45 This novel dehydrogenative alkene– long time. We think this is especially true of the nonchelate-assisted arene coupling reaction is an interesting alternative to the already C–H activation cross-couplings that we have just discovered and well-established dehydrogenative olefination reaction (see Section 2). highlighted here.

+ DG [Cp*RhCl2]2 (2.5 mol %) DG DG Cp* X AgSbF6 (10 mol %) X H H [RhV] H Cu(OAc)2 (2.2 equiv) + PivOH (1.1 equiv) CsOPiv (20 mol %) X 1 mmol 5 mL 140–160 oC, 21 h ~33 examples proposed key intermediate

(i-Pr)2N O (i-Pr)2N O (i-Pr)2N O (i-Pr)2N O Cl Br I Br

76% 81% 66% 89% m:p = 2.6:1 m:p = 2.8:1 m:p = 3.1:1 m:p = 2.9:1 OMe m:p = meta:para Br Br Br O (i-Pr)2N (i-Pr)2N O (i-Pr)2N O (i-Pr)2N O Br Br

CF3 Cl 74% 84% 75% 73% F3C Me MeO m:p = 3.1:1

Br Br Br Br

(i-Pr)2N O (i-Pr)2N O t-Bu O Br t-Bu O

F Cl Me

Me 74% 76% 62% 47%

Scheme 8. Directed and Undirected Aryl–Aryl Bond Formation. (Ref. 43) Cp*Rh-Catalyzed C–H Activations: Versatile Dehydrogenative Cross-Couplings of Csp2 C–H Positions 38 with Olefins, Alkynes, and Arenes Frederic W. Patureau,* Joanna Wencel-Delord, and Frank Glorius*

+ Cp* [Cp*RhCl2]2 (2.5 mol %) DG DG Br AgSbF6 (10 mol %) V H [Rh ] H H Cu(OAc)2 (2.2 equiv) DG + Br PivOH (1.1 equiv) Br CsOPiv (20 mol %) 1 mmol 5 mL 140–160 oC, 21 h 21 examples proposed key intermediate

(i-Pr)2N O (i-Pr)2N O (i-Pr)2N O (i-Pr)2N O Br Br Br Br Me Me Me Me Me Ph a 65% 51% 49% 41% m:p = 3.0:1 m:p = 3.0:1 m:p = 2.7:1 m:p = 2.2:1

Br Br Br Br

(i-Pr)2N O (i-Pr)2N O Br (i-Pr)2N O (i-Pr)2N O Br

Me Cl Me Cl Me Ph 60% 60% 47% 35%

Br Br Br Br

(i-Pr)2N O (i-Pr)2N O n-BuO O n-BuO O

Me Cl Me CF3 Me Cl Me CF3 Me Me Me Me 45% 43% 46% 45%

a 2,4,6-(i-Pr)3C6H2CO2H used instead of PivOH. m:p = meta:para.

Scheme 9. Directed and Undirected Alkene–Arene Bond Formation. (Ref. 45)

O Me N N O R N O NMe2 HN O H H H H H R2 R R Y R1 Y = O, NR R NR O OH O OR O NHR O NR Cp*Rh 2 H H H H H H X X = Cl, Br, I

Commonly Used Coupling Partners chelate-assisted nonchelate-assisted valuable H cross-coupled X products X = Cl, Br, I

Commonly Employed C–H Activation Partners

Figure 2. Typical Substrates of the Cp*Rh(III)-Catalyzed Cross-Coupling through C–H Activation.

6. Acknowledgements (FP7 2007–2013)/ERC Grant agreement no 25936, and the Alexander We express our gratitude to all the students and professors, who von Humboldt Foundation. The research of F. G. has been supported have worked creatively and tirelessly to promote the field of Rh(III)- by the Alfried Krupp Prize for Young University Teachers of the catalyzed C–H activations. We thank Professor P. M. Maitlis for Alfried Krupp von Bohlen und Halbach Foundation. helpful discussions. F. P. acknowledges support for the writing of this work by the DFG-funded transregional collaborative research 7. References and Notes center SFB/TRR 88 “Cooperative effects in homo- and heterometallic (1) For recent reviews on C–H activation, see: (a) Colby, D. A.; Tsai, A. complexes (3MET)” and the Alexander von Humboldt Foundation. S.; Bergman, R. G.; Ellman, J. A. Acc. Chem. Res. 2012, 45, 814. (b) F. G. also acknowledges the support of the European Research Council Engle, K. M.; Mei, T.-S.; Wasa, M.; Yu, J.-Q. Acc. Chem. Res. 2012, under the European Community’s Seventh Framework Program, 45, 788. (c) Wencel-Delord, J.; Dröge, T.; Liu, F. Glorius, F. Chem. 39 VOL. 45, NO. 2 • 2012

Soc. Rev. 2011, 40, 4740. (d) Yeung, C. S.; Dong, V. M. Chem. Rev. 2012, 14, 2304. For mechanistic studies of nucleophilic addition, 2011, 111, 1215. (e) McMurray, L.; O’Hara, F.; Gaunt, M. J. Chem. see: (j) Tauchert, M. E.; Incarvito, C. D.; Rheingold, A. L.; Berman, Soc. Rev. 2011, 40, 1885. (f) Satoh, T.; Miura, M. Chem.—Eur. J. R. G.; Ellman, J. A. J. Am. Chem. Soc. 2012, 134, 1482. (k) Li, Y.; 2010, 16, 11212. (g) Lyons, T. W.; Sanford, M. S. Chem. Rev. 2010, Zhang, X.-S.; Li, H.; Wang, W.-H.; Chen, K.; Li, B.-J.; Shi, Z.-J. 110, 1147. (h) Colby, D. A.; Bergman, R. G.; Ellman, J. A. Chem. Rev. Chem. Sci. 2012, 3, 1634. (l) For acylation of indoles, Zhou, B.; Yang, 2010, 110, 624. (i) Xu, L.-M.; Li, B.-J.; Yang, Z.; Shi, Z.-J. Chem. Y.; Li, Y. Chem. Commun. 2012, 48, 5163. (m) Very recently, Cp*Rh Soc. Rev. 2010, 39, 712. (j) Sun, C.-L.; Li, B.-J.; Shi, Z.-J. Chem. catalysis was used for the efficient and general ortho halogenation Commun. 2010, 46, 677. (k) Ackermann, L.; Vicente, R.; Kapdi, A. of various arenes: Schröder, N.; Wencel-Delord, J.; Glorius, F. J. Am. R. Angew. Chem., Int. Ed. 2009, 48, 9792. (l) Chen, X.; Engle, K. M.; Chem. Soc. 2012, 134, 8298.

Wang, D.-H.; Yu, J.-Q. Angew. Chem., Int. Ed. 2009, 48, 5094. (m) (8) The first (accidental) preparation of [Cp*RhCl2]2: Booth, B. L.; Giri, R.; Shi, B.-F.; Engle, K. M.; Maugel, N.; Yu, J.-Q. Chem. Soc. Haszeldine, R. N.; Hill, M. Chem. Commun. 1967, 1118.

Rev. 2009, 38, 3242. (n) Li, B.-J.; Yang, S.-D.; Shi, Z.-J. Synlett 2008, (9) The first mention of [Cp*RhCl2]2: Kang, J. W.; Maitlis, P. M. J. Am. 949. (o) Kakiuchi, F.; Kochi, T. Synthesis 2008, 3013. (p) Satoh, T.; Chem. Soc. 1968, 90, 3259. Miura, M. Top. Organomet. Chem. 2007, 24, 61. (q) Alberico, D.; (10) Kisenyi, J. M.; Sunley, G. J.; Cabeza, J. A.; Smith, A. J.; Adams, H.; Scott, M. E.; Lautens, M. Chem. Rev. 2007, 107, 174. (r) Campeau, Salt, N. J.; Maitlis, P. M. J. Chem. Soc., Dalton Trans. 1987, 2459.

L.-C.; Stuart, D. R.; Fagnou, K. Aldrichimica Acta 2007, 40, 35. (s) (11) (a) For an early report in which [Cp*RhCl2]2 was singled out as Song, G.; Wang, F.; Li, X. Chem. Soc. Rev. 2012, 41, 3651. effective in the oxidative dehydrogenative cross-coupling arylation (2) For a comprehensive review on C–C-bond-forming cdc couplings, of ethylene, see Matsumoto, T.; Periana, R. A.; Taube, D. J.; Yoshida, see Li, C.-J. Acc. Chem. Res. 2009, 42, 335. H. J. Catal. 2002, 206, 272. (b) See also Matsumoto, T.; Yoshida, H. (3) (a) Fujiwara, Y.; Moritani, I.; Danno, S.; Asano, R.; Teranishi, S. J. Chem. Lett. 2000, 29, 1064. Am. Chem. Soc. 1969, 91, 7166. For other selected references, see: (12) (a) Ueura, K.; Satoh, T.; Miura, M. Org. Lett. 2007, 9, 1407. (b) (b) Jia, C.; Piao, D.; Oyamada, J.; Lu, W.; Kitamura, T.; Fujiwara, Y. Ueura, K.; Satoh, T.; Miura, M. J. Org. Chem. 2007, 72, 5362. (c) Science 2000, 287, 1992. (c) Jia, C.; Kitamura, T.; Fujiwara, Y. Acc. Umeda, N.; Tsurugi, H.; Satoh, T.; Miura, M. Angew. Chem., Int. Ed. Chem. Res. 2001, 34, 633. 2008, 47, 4019. (d) Umeda, N.; Hirano, K.; Satoh, T.; Miura, M. J. (4) For recent reviews on the Mizoroki–Heck reaction: (a) Bräse, S.; Org. Chem. 2009, 74, 7094. de Meijere, A. Cross-Coupling of Organyl Halides with Alkenes: (13) Patureau, F. W.; Glorius, F. J. Am. Chem. Soc. 2010, 132, 9982. The Heck Reaction. In Metal-Catalyzed Cross-Coupling Reactions, (14) (a ) Gong, T.-J.; Xiao, B.; Liu, Z.-J.; Wan, J.; Xu, J.; Luo, D.-F.; Fu, 2nd ed.; de Meijere, A., Diederich, F., Eds.; Wiley-VCH: New York, Y.; Liu, L. Org. Lett. 2011, 13, 3235. (b) Feng, C.; Loh, T.-P. Chem. 2004; Vol. 1, Chapter 5, p 217–315. (b) Nicolaou, K. C.; Bulger, P. Commun. 2011, 47, 10458. G.; Sarlah, D. Angew. Chem., Int. Ed. 2005, 44, 4442 and citations (15) (a) Stuart, D. R.; Bertrand-Laperle, M.; Burgess, K. M. N.; Fagnou, therein. K. J. Am. Chem. Soc. 2008, 130, 16474. (b) Shortly before, we (5) Murai’s famous Ru-catalyzed C–H activation: (a) Murai, S.; reported an alternative Pd-catalyzed indole formation: Würtz, S.; Kakiuchi, F.; Sekine, S.; Tanaka, Y.; Kamatani, A.; Sonoda, M.; Rakshit, S.; Neumann, J. J.; Dröge, T.; Glorius, F. Angew. Chem., Chatani, N. Nature 1993, 366, 529. For related works, see: (b) Int. Ed. 2008, 47, 7230. Kakiuchi, F.; Kan, S.; Igi, K.; Chatani, N.; Murai, S. J. Am. Chem. (16) (a ) Patureau, F. W.; Besset, T.; Glorius, F. Angew. Chem., Int. Ed. Soc. 2003, 125, 1698. (c) Martinez, R.; Chevalier, R.; Darses, S.; 2011, 50, 1064. (b) Benzoates and especially benzaldehydes are less Genet, J.-P. Angew. Chem., Int. Ed. 2006, 45, 8232. (d) Simon, M.-O.; efficient in this transformation: Park, S. H.; Kim, J. Y.; Chang, S. Genet, J.-P.; Darses, S. Org. Lett. 2010, 12, 3038. (e) Watson, A. J. A.; Org. Lett. 2011, 13, 2372. (c) For oxime directing groups, see Tsai, Maxwell, A. C.; Williams, J. M. J. Org. Lett. 2010, 12, 3856. A. S.; Brasse, M.; Bergman, R. G.; Ellman, J. A. Org. Lett. 2011, 13, (6) For example, see Miura, M.; Tsuda, T.; Satoh, T.; Pivsa-Art, S.; 540. For benzoic acids, see: (d) Mochida, S.; Hirano, K.; Satoh, T.; Nomura, M. J. Org. Chem. 1998, 63, 5211. Miura, M. J. Org. Chem. 2011, 76, 3024. (e) Mochida, S.; Hirano, K.;

(7) Wh ile the present review is restricted to the Cp*Rh-catalyzed Csp2 Satoh, T.; Miura, M. Org. Lett. 2010, 12, 5776. C–H couplings with alkenes, alkynes, and arenes, it is worth noting (17) Fo r lead references on the “base-assisted metallation” mechanism,

that [Cp*RhCl2]2 is also effective in the C–H oxidative amination see: (a) Li, L.; Brennessel, W. W.; Jones, W. D. Organometallics 2009, reaction (formation of C–N bonds). This is still a very recent and 28, 3492. (b) Lapointe, D.; Fagnou, K. Chem. Lett. 2010, 39, 1119. (c) hot topic, and we invite the reader to find out more at: (a) Ng, K.-H.; Ackermann, L. Chem. Rev. 2011, 111, 1315 and references therein. Zhou, Z.; Yu, W.-Y. Org. Lett. 2012, 14, 272. (b) Grohmann, C.; (18) (a) Rakshit, S.; Grohmann, C.; Besset, T.; Glorius, F. J. Am. Chem. Wang, H.; Glorius, F. Org. Lett. 2012, 14, 656. The reader should also Soc. 2011, 133, 2350. (b) For a recent highlight, see Patureau, F. be aware that Cp*Rh is effective in the C–H activation and addition W.; Glorius, F. Angew. Chem., Int. Ed. 2011, 50, 1977 and citations to aldehydes, imines, and isocyanates. For these and related topics, therein. (c) See also Guimond, N.; Gouliaras, C.; Fagnou, K. J. see: (c) Li, Y.; Zhang, X.-S.; Chen, K.; He, K.-H.; Pan, F.; Li, B.-J.; Am. Chem. Soc. 2010, 132, 6908. (d) For a related and insightful Shi, Z.-J. Org. Lett. 2012, 14, 636. (d) Tsai, A. S.; Tauchert, M. E.; computational study, see Xu, L.; Zhu, Q.; Huang, G.; Cheng, B.; Xia, Bergman, R. G.; Ellman, J. A. J. Am. Chem. Soc. 2011, 133, 1248. Y. J. Org. Chem. 2012, 77, 3017. For the pioneering use of O-methyl (e) Li, Y.; Li, B.-J.; Wang, W.-H.; Huang, W.-P.; Zhang, X.-S.; Chen, hydroxamates as directing groups in C–H activation chemistry, see: K.; Shi, Z.-J. Angew. Chem., Int. Ed. 2011, 50, 2115. (f) Hesp, K. D.; (e) Wang, D.-H.; Wasa, M.; Giri, R.; Yu, J.-Q. J. Am. Chem. Soc. Bergman, R. G.; Ellman, J. A. J. Am. Chem. Soc. 2011, 133, 11430. 2008, 130, 7190. (f) Wasa, M.; Yu, J.-Q. J. Am. Chem. Soc. 2008, (g) Yang, L.; Correia, C. A.; Li, C.-J. Adv. Synth. Catal. 2011, 353, 130, 14058. (g) Li, D.-D.; Yuan, T.-T.; Wang, G.-W. Chem. Commun. 1269. (h) Park, J.; Park, E.; Kim, A.; Lee, Y.; Chi, K.-W.; Kwak, J. 2011, 47, 12789. (h) For a recent coupling of allenes with arenes H.; Jung, Y. H.; Kim, I. S. Org. Lett. 2011, 13, 4390. (i) For addition bearing an internal oxidant directing group, see Wang, H.; Glorius, to imines, see Hesp, K. D.; Bergman, R. G.; Ellman, J. A. Org. Lett. F. Angew. Chem., Int. Ed. 2012, 51, DOI: 10.1002/anie.201201273. Cp*Rh-Catalyzed C–H Activations: Versatile Dehydrogenative Cross-Couplings of Csp2 C–H Positions 40 with Olefins, Alkynes, and Arenes Frederic W. Patureau,* Joanna Wencel-Delord, and Frank Glorius*

(19) Besset, T.; Kuhl, N.; Patureau, F. W.; Glorius, F. Chem.—Eur. J. Umeda, N.; Hirano, K.; Satoh, T.; Shibata, N.; Sato, H.; Miura, M. J. 2011, 17, 7167. Org. Chem. 2011, 76, 13. (20) Fo r selected examples of vinylic C–H functionalization reactions, (31) Gu imond, N.; Gorelsky, S. I.; Fagnou, K. J. Am. Chem. Soc. 2011, see: (a) Sato, T.; Kakiuchi, F.; Chatani, N.; Murai, S. Chem. Lett. 133, 6449. 1998, 893. (b) Jun, C.-H.; Moon, C. W.; Kim, Y.-M.; Lee, H.; Lee, J. (32) (a) Hyster, T. K.; Rovis, T. J. Am. Chem. Soc. 2010, 132, 10565. (b) H. Tetrahedron Lett. 2002, 43, 4233. (c) Colby, D. A.; Bergman, R. Hyster, T. K.; Rovis, T. Chem. Sci. 2011, 2, 1606. For the formation of G.; Ellman, J. A. J. Am. Chem. Soc. 2006, 128, 5604. (d) Gooßen, heterocycles, see: (c) Du, Y.; Hyster, T. K.; Rovis, T. Chem. Commun. L. J.; Rodríguez, N. Angew. Chem., Int. Ed. 2007, 46, 7544. (e) Tsai, 2011, 47, 12074. (d) Hyster, T. K.; Rovis, T. Chem. Commun, 2011, A. S.; Bergman, R. G.; Ellman, J. A. J. Am. Chem. Soc. 2008, 130, 47, 11846. 6316. (f) Parthasarathy, K.; Jeganmohan, M.; Cheng, C.-H. Org. Lett. (33) (a) Song, G.; Chen, D.; Pan, C.-L.; Crabtree, R. H.; Li, X. J. Org. 2008, 10, 325. (g) Giri, R.; Yu, J.-Q. J. Am. Chem. Soc. 2008, 130, Chem. 2010, 75, 7487. (b) Chen, J.; Song, G.; Pan, C.-L.; Li, X. Org. 14082. (h) Simon, M.-O.; Martinez, R.; Genêt, J.-P.; Darses, S. Adv. Lett. 2010, 12, 5426. (c) Wang, F.; Song, G.; Li, X. Org. Lett. 2010, 12, Synth. Catal. 2009, 351, 153. (i) Neisius, N. M.; Plietker, B. Angew. 5430. (d) Wang, F.; Song, G.; Du, Z.; Li, X. J. Org. Chem. 2011, 76, Chem., Int. Ed. 2009, 48, 5752. (j) Shibata, Y.; Otake, Y.; Hirano, 2926. See also: (e) Li, X.; Gong, X.; Zhao, M.; Song, G.; Deng, J.; Li, M.; Tanaka, K. Org. Lett. 2009, 11, 689. (k) Su, Y.; Zhao, M.; Han, X. Org. Lett. 2011, 13, 5808. (f) Li, X.; Zhao, M. J. Org. Chem. 2011, K.; Song, G.; Li, X. Org. Lett. 2010, 12, 5462. (l) Colby, D. A.; 76, 8530. (g) Song, G.; Gong, X.; Li, X. J. Org. Chem. 2011, 76, 7583. Bergman, R. G.; Ellman, J. A. J. Am. Chem. Soc. 2008, 130, 3645. (34) (a) Wang, Y.-F.; Toh, K. K.; Lee, J.-Y.; Chiba, S. Angew. Chem., Int. (m) Lindhardt, A. T. (neé Hansen); Mantel, M. L. H.; Skrydstrup, Ed . 2011, 50, 5927. (b) Too, P. C.; Chua, S. H.; Wong, S. H.; Chiba, T. Angew. Chem., Int. Ed. 2008, 47, 2668. (n) Ebran, J.-P.; Hansen, S. J. Org. Chem. 2011, 76, 6159. (c) Too, P. C.; Wang, Y.-F.; Chiba, A. L.; Gøgsig, T. M.; Skrydstrup, T. J. Am. Chem. Soc. 2007, 129, S. Org. Lett. 2010, 12, 5688. 6931. (o) Mochida, S.; Hirano, K.; Satoh, T.; Miura, M. J. Org. Chem. (35) Zhu, C.; Falck, J. R. Chem. Commun. 2012, 48, 1674. 2009, 74, 6295. (p) Ackermann, L.; Lygin, A. V.; Hofmann, N. Org. (36) (a) Willwacher, J.; Rakshit, S.; Glorius, F. Org. Biomol. Chem. 2011, 9, Lett. 2011, 13, 3278. 4736. (b) Wang, H.; Glorius, F. Angew. Chem., Int. Ed. 2012, 51, 7318. (21) Fagnou, K. Presented at the ROCCAT Meeting, Münster, Germany, (c) Shi, Z.; Schröder, N.; Glorius, F. Angew. Chem., Int. Ed. [Early View] June 18, 2009; Presentation No. 8. Westfälische Wilhelms- 2012, 51, DOI: 10.1002/anie.201203224. Universität Münster Web site. http://www.uni-muenster.de/Chemie. (37) (a) For a review on undirected arene C–H activations, see Kuhl, N.; oc/glorius/en/roccat.html. For related content, see reference 15a. Hopkinson, M. N.; Wencel-Delord, J.; Glorius, F. Angew. Chem., Int. (22) Rakshit, S.; Patureau, F. W.; Glorius, F. J. Am. Chem. Soc. 2010, 132, Ed. 2012, 51, DOI: 10.1002/anie.201203269. (b) See also Neufeldt, 9585. S. R.; Sanford, M. S. Acc. Chem. Res. 2012, 45, 936. (23) These β-amino acid precursors (see Scheme 5) do not lead to the (38) (a ) For nonchelate-assisted C–H oxidative olefination of some oxidative olefination of any of their C–H positions (as opposed to specific heterocyclic structures, see Grimster, N. P.; Gauntlett, C.; methylacetamidoacrylates, see Section 2.2). The reason for this is Godfrey, C. R. A.; Gaunt, M. J. Angew. Chem., Int. Ed. 2005, 44, unclear. 3125. For reviews, see: (b) Beck, E. M.; Gaunt, M. J. Top. Curr. (24) Stuart, D. R.; Alsabeh, P.; Kuhn, M.; Fagnou, K. J. Am. Chem. Soc. Chem. 2010, 292, 85. (c) Seregin, I. V.; Gevorgyan, V. Chem. Soc. 2010, 132, 18326. Rev. 2007, 36, 1173 and references therein. (25) Patureau, F. W.; Besset, T.; Kuhl, N.; Glorius, F. J. Am. Chem. Soc. (39) For other pioneering examples of nonchelate-assisted C–H oxidative 2011, 133, 2154. olefinations, see: (a) Dams, M.; De Vos, D. E.; Celen, S.; Jacobs, P. (26) Fo r selected reports describing the metal-catalyzed synthesis of A. Angew. Chem., Int. Ed. 2003, 42, 3512. (b) Yokota, T.; Tani, M.; indenols from ortho-acylphenylboronic acids, see: (a) Matsuda, T.; Sakaguchi, S.; Ishii, Y. J. Am. Chem. Soc. 2003, 125, 1476. (c) Tani, Makino, M.; Murakami, M. Chem. Lett. 2005, 34, 1416. (b) Gourdet, M.; Sakaguchi, S.; Ishii, Y. J. Org. Chem. 2004, 69, 1221. (d) Yamada, B.; Rudkin, M. E.; Lam, H. W. Org. Lett. 2010, 12, 2554. From ortho- T.; Sakakura, A.; Sakaguchi, S.; Obora, Y.; Ishii, Y. New J. Chem. halophenyl ketones, see: (c) Chang, K.-J.; Rayabarapu, D. K.; Cheng, 2008, 32, 738. (e) Zhang, Y.-H.; Shi, B.-F.; Yu, J.-Q. J. Am. Chem. C.-H. J. Org. Chem. 2004, 69, 4781. (d) Chang, K.-J.; Rayabarapu, Soc. 2009, 131, 5072. (f) Ye, M.; Gao, G.-L.; Yu, J.-Q. J. Am. Chem. D. K.; Cheng, C.-H. Org. Lett. 2003, 5, 3963. (e) Rayabarapu, D. K.; Soc. 2011, 133, 6964. (g) Hickman, A. J.; Sanford, M. S. ACS Catal. Yang, C.-H.; Cheng, C.-H. J. Org. Chem. 2003, 68, 6726. (f) Quan, L. 2011, 1, 170. (h) Li, Z.; Zhang, Y.; Liu, Z.-Q. Org. Lett. 2012, 14, 74. G.; Gevorgyan, V.; Yamamoto, Y. J. Am. Chem. Soc. 1999, 121, 3545. (40) Patureau, F. W.; Nimphius, C.; Glorius, F. Org. Lett. 2011, 13, 6346. (27) (a) Muralirajan, K.; Parthasarathy, K.; Cheng, C.-H. Angew. Chem., (41) Th e use of bromobenzene as reoxidant in Rh catalysis had been Int. Ed. 2011, 50, 4169. (b) Li, B.-J.; Wang, H.-Y.; Zhu, Q.-L.; Shi, postulated previously: (a) Barrett, A. G. M.; Itoh, T.; Wallace, E. Z.-J. Angew. Chem., Int. Ed. 2012, 51, 3948. M. Tetrahedron Lett. 1993, 34, 2233. (b) Bouffard, J.; Itami, K. Top. (28) For an Ir-catalyzed analogous reaction, see Tsuchikama, K.; Curr. Chem. 2010, 292, 231. Kasagawa, M.; Endo, K.; Shibata, T. Synlett 2010, 97. (42) For recent reviews on aryl–aryl bond formation, see: (a) McGlacken, (29) Th e analogous amino indene products have also been prepared by G. P.; Bateman, L. M. Chem. Soc. Rev. 2009, 38, 2447. (b) You, S.-L.; using a quite different C–H activation technology based on a Rh(I) Xia, J.-B. Top. Curr. Chem. 2010, 292, 165. (c) Ashenhurst, J. A. catalyst and a bidentate phosphine ligand; see Sun, Z.-M.; Chen, Chem. Soc. Rev. 2010, 39, 540. S.-P.; Zhao, P. Chem.—Eur. J. 2010, 16, 2619. (43) Wencel-Delord, J.; Nimphius, C.; Patureau, F. W.; Glorius, F. Angew. (30) (a) Fukutani, T.; Umeda, N.; Hirano, K.; Satoh, T.; Miura, M. Chem. Chem., Int. Ed. 2012, 51, 2247. Commun. 2009, 5141. (b) Morimoto, K.; Hirano, K.; Satoh, T.; Miura (44) In contrast to the report on the nonchelate-assisted dehydrogenative M. Org. Lett. 2010, 12, 2068. (c) Mochida, S.; Umeda, N.; Hirano, olefination (see reference 40), no latent period was observed here. K.; Satoh, T.; Miura, M. Chem. Lett. 2010, 39, 744. (d) Morimoto, K.; (45) Wencel-Delord, J.; Nimphius, C.; Patureau, F. W.; Glorius, F. Hirano, K.; Satoh, T.; Miura, M. J. Org. Chem. 2011, 76, 9548. (e) Chem.—Asian J. 2012, 7, 1208. 41 VOL. 45, NO. 2 • 2012

(46) For a selective Rh(III)-catalyzed dehydrogenative cross-coupling Joanna Wencel-Delord graduated from the École Nationale of furan and thiophene derivatives, see Kuhl, N.; Hopkinson, M. Supérieure de Chimie de Rennes, and received her M.Sc. degree in N.; Glorius, F. Angew. Chem., Int. Ed. 2012, 51, DOI: 10.1002/ organic chemistry in 2007 from the Université de Rennes I (France). anie.201203792. She obtained her Ph.D. degree in 2010 from the same university, under the supervision of Drs. C. Crévisy and M. Mauduit, by working on ® Trademarks. CAS Registry Number (The American Chemical Society). the development of new chiral ligands for asymmetric catalysis. She investigated Rh-catalyzed C–H activation reactions as a postdoctoral About the Authors fellow in the research group of Prof. Frank Glorius at the University Frederic William Patureau was born in 1982 in St. Germain en of Münster until May 2012. She is currently a lecturer (ATER) at the Laye, France. He studied biology at the University of Versailles University of Strasbourg (École Européenne de Chimie, Polymères (France, 2000–2002), and obtained his bachelor’s degree in chemistry et Matériaux (ECPM) de Strasbourg). in 2003 from the University of Paris-Sud Orsay (France). There, Frank Glorius studied chemistry at the Universität Hannover, he completed his master’s degree (2005) in the group of Prof. H. Stanford University (Prof. P. A. Wender), Max-Planck-Institut für B. Kagan and Prof. J.-C. Fiaud (mentors: Dr. E. Schulz and Dr. M. Kohlenforschung Mülheim and Universität Basel (Ph.D., 2000, with Mellah). He then moved to the Van’t Hoff Institute for Molecular Prof. A. Pfaltz), and Harvard University (Prof. D. A. Evans). In 2001, Sciences at the University of Amsterdam (Netherlands), where he he began his independent research career at the Max-Planck-Institut obtained his doctoral degree in 2009 (mentor: Prof. J. N. H. Reek). He für Kohlenforschung in Germany (mentor: Prof. A. Fürstner). In 2004, then joined the group of Prof. F. Glorius at the University of Münster he was appointed Associate Professor at the Philipps-Universität (Germany), where he became an Alexander von Humboldt Fellow. Marburg and, since 2007, he has been Professor of Organic Chemistry In November 2011, he started his independent career as Junior at the Westfälische Wilhelms-Universität Münster, Germany. His Professor of Organometallic Catalysis at the Technische Universität research program focuses on the development of new concepts for Kaiserslautern. catalysis and their implementation in organic synthesis.

Looking to introduce into your molecule?

At Aldrich® Chemistry, we recognize the increasing importance of uorination in pharmaceutical and agricultural research. We have recently added the new uorination products below to our growing portfolio of reagents that: • Are e ective for uorination, di uoromethylation, and tri uoromethylation • Permit highly e cient reactions to take place under mild conditions • Tolerate a variety of functional groups From milligram to bulk quantities, your research will move forward faster when you add trustworthy products, services, and information from Aldrich Chemistry.

Fluorination Difluoromethylation Trifluoromethylation

Add Aldrich to your research program Phenofluor™ Baran DFMS Reagent Togni’s Reagent Aldrich.com/fluoro SFL00001 L510084 696641; 754218

PhenoFluor is a trademark of SciFluor Life Sciences, LLC 79242 New a-Boryl Aldehyde MIDA Boronates MeN MeN MeN MeN B O O For a rare and MeN O You asked, B O O B O O B O O O O O O O O O O O B O O H O O 704415 698709 721573 698032 stable a-Boryl Aldrich listened. MeN MeN MeN MeN MeN O O MeN B O O B O O MIDA071 B O O B O O B O O H H O O O O B O O Bu 3Sn O O Br O O O O Any MIDA Boronate: US$15/1g Me O O Aldehyde Reagent. Me MIDA072 700231 701831 704873 703478 Me MeN MeN MeN MIDA070 O MeN MeN MeN B O O O B O O B O O N B O O B O O O O MeN H O O B O O O O N O O O Me O O R H O O SO2Ph B O O H O O 700657 697311 697443 719390 Amphoteric MeN MIDA076 MeN MeN MeN a-Boryl Boc Me2N O N N B O O Aldehydes OHC B O O B O O B O O O O O O O O O O MIDA073 Br N MeN MeN New, lower cost than most boronic acids, 704547 733539 724629 702269 MIDA boronates are valuable reagents for highly O O • B O O B O O H H MeN chemoselective reaction sequences. A new line of MIDA O O O O boronic esters, and trifluoroborates MeN MeN MeN B O O boronates, developed by the Yudin group, are now available B O O MIDA074 MIDA075 B O O B O O O O • Shelf- and bench-stable O O O O O O from Aldrich® Chemistry. The amphoteric nature of these Me Me F (HO)2B Br a-boryl aldehydes allows: • Rapidly expanding selection 707252 704997 698040 699136 Handling in the presence of air and moisture MeN • MeN MeN MeN OMe Br Building Blocks Obtainable through B O O Highly diastereoselective additions without the need O O B O O B O O B O O • a-Boryl Aldehyde MIDA Boronates O O O O O O O F for an expensive auxiliary N OMe F Br O The preparation of several building blocks from one reagent O O O 707902 701084 698164 707880 • O O O O MeN O O MeN O MeN O [B] O B O MeN O One step B B O Highly useful B O Ph building block Ph Ph OTf R N O Ph Visit Aldrich.com/mida to vote for the next MeN NBn2 OTIPS MIDA Boronate Aldrich should add [B] = B O O Enamines and Vinyl silyl ether O O MeN MeN enamides and triflate MeN MeN Me B O O B O O F3C N O O B O O O O B O O N I O O Bu 3Sn O O O N O O O H O O MeN O MeN MeN O MeN O O MeN MeN MeN O O B O B B O O B Br Me B O O N B O O O H Me B O O B O O Ph COOH O O O O O O Me O N Ph Br Ph COOEt N S Me Me Olefination Configurationally adducts stable boryl acids MeN MeN Me Me Me MeN B O O B O O O O B O O O O Cl Bn N O O O N Me N O MeN O O For a complete list of MeN MeN MeN For more information, B Homoallyl B O O B O O MeO B O O Me Ph alcohols O O O O O O N MIDA Boronates, visit Ph see our technical spotlight at N O N OH F N N OMe Aldrich.com/mida HO2C CO2H Aldrich.com/yudinmida syn:anti = 93:7

79188 79127

79188_MIDA Ad (Acta 45.2).indd 1 7/27/2012 2:07:09 PM 79127 Yudin MIDA Ad for Acta Vol. 45, No. 2.indd 1 8/2/2012 12:17:18 PM New a-Boryl Aldehyde MIDA Boronates

For a rare and MeN O B O O stable a-Boryl H O O MeN MeN O O B O O B O O MIDA071 H H O O O O Aldehyde Reagent. Me MIDA072 Me MeN MIDA070 O MeN B O O O MeN H O O B O O O R H O O B O O H O O Amphoteric a-Boryl MIDA076 Aldehydes MIDA073

MeN MeN MIDA boronates are valuable reagents for highly O O B O O B O O chemoselective reaction sequences. A new line of MIDA H O O H O O boronates, developed by the Yudin group, are now available MIDA074 MIDA075 from Aldrich® Chemistry. The amphoteric nature of these Me F a-boryl aldehydes allows: • Handling in the presence of air and moisture Building Blocks Obtainable through Highly diastereoselective additions without the need • a-Boryl Aldehyde MIDA Boronates for an expensive auxiliary O The preparation of several building blocks from one reagent O O O • O O O O MeN O O MeN O MeN O [B] O B O MeN O One step B B O Highly useful B O Ph building block Ph Ph OTf R N O Ph MeN NBn2 OTIPS

[B] = B O O Enamines and Vinyl silyl ether O O enamides and triflate

O O O O O O MeN O MeN O MeN O O O O B B Br B Me Ph COOH Ph Br Ph COOEt Olefination Configurationally adducts stable boryl acids

O O

MeN O O For more information, B Homoallyl Ph alcohols see our technical spotlight at OH Aldrich.com/yudinmida syn:anti = 93:7

79127

79127 Yudin MIDA Ad for Acta Vol. 45, No. 2.indd 1 8/2/2012 12:17:18 PM Looking for versatile Small-Molecule Synthons?

Aldrich® Chemistry is pleased to introduce two air- and H moisture-stable tungsten complexes. These complexes serve B as precursors to highly functionalized cyclohexenones and N N N η2-coordinated arenes that undergo a broad range of organic N N transformations with predictable stereochemistry. They are N W CH3 ideal for synthesizing highly functionalized cyclohexene and O P CH piperidine derivatives. NO 3 CH • Air- and moisture-stable; bench-top storage 3 • Mild and rapid reactions 738530 • Convenient spectroscopic reaction monitoring and analysis H • Excellent regio- and stereoselectivities B Umpolung electrophilic modifications of a • N N wide array of arenes and heteroarenes N N N N New reactivity patterns leading to • W novel dearomatized molecules Br Br NO

738026

Advance your research. Add Aldrich. Reference: Welch, K. D.; Harrison, D. P.; Lis, E. C., Jr.; Liu, W.; Salomon, R. J.; Harman, W. D.; Myers, W. H. Aldrich.com/structuresearch Organometallics 2007, 26, 2791

79120

79120 Harman Product Ad for Acta Vol. 45, No. 2.indd 1 8/2/2012 12:19:42 PM 45 VOL. 45, NO. 2 • 2012 Opening New Chemical Space through Novel Dearomatization Reactions

Daniel P. Harrison and W. Dean Harman* Department of Chemistry University of Virginia Charlottesville, VA 22904-4319, USA Email: [email protected]

Dr. Daniel P. Harrison Prof. W. Dean Harman

Keywords. tungsten complex; aromatic; dearomatize; arenes; dearomatized) and more electron-rich. Hence, the arene is activated 2 toward electrophilic addition reactions (Figure 1).12 dihapto-coordinated; TpW(NO)(PMe3)(h -C6H6). Several generations of dihapto-coordinated dearomatization Abstract. The attachment of a tungsten complex to various aromatic (DCD) agents have been developed. The first complex reported molecules across two carbons localizes the remaining p system in to coordinate benzene in a dihapto fashion and activate it toward II 2+ the molecules and renders them highly activated toward electrophilic organic transformations was {Os (NH3)5} . This was followed by I 0 0 0 reagents. As a result, new bonds are formed between the {TpRe (CO)(MeIm)} and finally by {TpW (NO)(PMe3)} , where 13 and the aromatic scaffold, giving rise to stereogenic centers with MeIm is methylimidazole and Tp is tris(pyrazolyl)borate (Figure 2). predictable configurations. Treatment of the resulting adducts with As one moves from Os(II) to W(0), the reactivity toward various oxidants frees the organic product from the tungsten fragment. increases owing to the increase in π back-bonding associated with the 13 This approach permits the synthesis of a number of small molecules lower oxidation state of the metal. that are difficult to prepare by conventional methods. Herein, we survey the synthesis of tungsten-based transition-metal– arene complexes and the transformation of their aromatic ligands into Outline novel, highly functionalized, alicyclic structures in relatively few steps. 1. Introduction 2. Sy nthesis, Characterization, and Reactivity of Tungsten–Arene 2. Synthesis, Characterization, and Reactivity of Tungsten– Complexes Arene Complexes 2.1. Derivatization of Phenol and Aniline Complexes have been prepared for a large number of aromatic compounds 2 2.2. Derivatization of Pyridine by ligand substitution of the benzene analogue TpW(NO)(PMe3)(h -benzene). 2.3. Derivatization of Other Aromatic Compounds The resulting complexes have been systematically studied in an effort to 2.3.1. Substituted h2-Pyridine Complexes fundamentally understand the effect that coordination of an extremely 2.3.2. Pyrrole π-basic metal has on the aromatic compound. One of the most useful 2.3.3. Anisole outcomes of the initial studies was revealed when analyzing 31P NMR 3. Demetallation spectra of the complexes.14 Tungsten has one spin = ½ isotope of 4. Conclusion sufficient abundance 183( W, 14% abundance) to display W–P coupling. 5. References and Notes 183W–31P coupling constants are highly sensitive to the coordination environment about the W center (e.g., κ1-, η2-, 7-coordinate oxidative 1. Introduction insertion complexes; Figure 3).14 As a result, it is possible to utilize The reactivity of arenes and aromatic heterocycles is profoundly 31P NMR data to facilitate the analysis of reactions without the need affected by their coordination to a transition metal. While much is for deuterated solvents, providing information about (i) the number known about the chemistry of η6-coordinated arenes, the chemistry of products and (ii) the types of complexes (i.e., η2, κ1, etc.) that are associated with dihapto-coordinated aromatics has been slow to generated throughout the course of a reaction. Thus, optimization of develop, owing to the relative instability and inaccessibility of such reaction conditions via 31P NMR is both rapid and economical. In complexes. The arene moiety in complexes such as (h6-arene)Cr(CO) ,1,2 addition to NMR, infrared (NO stretch) and electrochemical (W(I/0): + + 3 h6 3,4 h6 5–8 h6 9 [( -arene)Mn(CO)3] , [( -arene)RuCp] , and ( -arene)Mo(CO)3 is Ep,a) data also give valuable information regarding the purity and nature susceptible to nucleophilic substitution, addition, or side-chain of the products.14 activation,10 leading to the formation of substituted arenes or cyclohexadienes.11 While the arene ligands of such complexes are more 2.1. Derivatization of Phenol and Aniline reactive than their uncomplexed counterparts, they nevertheless remain Among their respective three tautomers, phenol and aniline primarily largely aromatic. In the complementary approach of η2 coordination, exist in their enolic and enaminic forms due to aromatic stabilization the metal–arene bond is stabilized primarily by interaction of a filled (Figure 4).15–17 The equilibrium is significantly altered when these metal dπ orbital with a π* orbital of the aromatic ligand. Through this arenes are coordinated to {TpW(NO)(PMe3)}. For phenol, only the interaction, the aromatic π system becomes both more localized (i.e., 2H-tautomer complex, 2, is isolated when phenol is combined with the Opening New Chemical Space through Novel Dearomatization Reactions 46 Daniel P. Harrison and W. Dean Harman*

2 16 benzene precursor TpW(NO)(PMe3)(η -benzene) (1) (Scheme 1). In the case of aniline, isolation of a neutral η2 complex is difficult due + E to a competing oxidative addition across an N–H bond, which serves M ≡ [M] ~ to reduce the electron density at the metal center.17 To avert the N–H insertion, replacement of the amine protons in aniline with alkyl groups 2 filled π* is necessary. However, isolation of the neutral η -aniline complex 3a orbital dπ is still difficult, due to its highly electron-rich nature (see the benzene orbital analogue, 1). This problem is solved by addition of a weak acid to the substitution reaction, which leads to the 2H-η2-anilinium complex, 3b, Figure 1. Fundamental Metal–Ligand Interaction and Activation. (Ref. 12) that is very stable to both oxidation and substitution. For both 2 and 3b, selective precipitation procedures have been developed that allow for the isolation of a single coordination diastereomer. Also of note, both 2 and 3b are air-stable, and most of the reactions presented here can be 2+ performed in a fume hood without the need for glove boxes or special II (NH3)5Os techniques (i.e., Schlenk techniques). 1st generation The C3–C4 alkene bond of the 2H-phenol or 2H-anilinium ligand is

Me highly activated toward electrophilic addition reactions. Ketenes readily N undergo [2 + 2] cycloadditions with alkenes,18,19 and similar reactivity is found with h2-phenol complexes. Ketenes, generated in situ via the N Increasing electron CO donation to aromatic dehydrohalogenation of acid chlorides, react exclusively at the exposed ReI ligand by metal leads 16 Tp to increased activation alkene (C3=C4) to generate a cyclobutanone core, 4a–c (Scheme 2). of the ligand toward The reaction is highly stereoselective, with addition occurring at the electrophilic addition 2nd generation reactions. face of the ligand opposite to that to which the tungsten is coordinated: the metal complex sterically blocks addition syn to the metal. Addition Me3P NO of ketenes is also highly regioselective, with the electrophilic portion W0 of the ketene adding to the meta position of the phenol ring. This is Tp particularly noteworthy for phenol because it represents an umpolung,

1 or reversal, of reactivity that is typically associated with phenol, where 3rd generation electrophiles add to C2 (ortho carbon), C4 (para carbon), or the O atom.20 2 Figure 2. Dearomatization Agents. (Ref. 13) Note that, not only does η coordination increase the electron density of the exposed double bond, it also polarizes the bond such that electrophiles add at the terminal carbon of the bound diene fragment and the resulting allylic cation is stabilized by the metal (see 4ai–4ci in Scheme 2). PMe3 PMe3 PMe3 Pz NO Pz NO Pz NO The principles outlined above for ketenes may be generalized to 21,22 W W Y W Y other types of electrophiles. Addition of meta-chloroperoxybenzoic Pz L Pz Pz X Pz Pz X Pz acid (mCPBA), a combination of [bis(trifluoroacetoxy)iodo]benzene (PIFA) and methyltriphenylphosphonium bromide (MTPBr), and κ1 η2 7-coordinate ® 366 < JWP < 415 255 < JWP < 314 100 < JWP < 212 Selectfluor (i.e., sources of electrophilic oxygen, bromine, and fluorine,

Figure 3. Coordination Environments of Tungsten and Associated Coupling W W Constant Ranges in Hz. (Ref. 14) OH O PhOH 2 H PhNH2 (5 equiv) W W Ph DME, rt, 8 h N H 1 PhNMe2 90% DiPAT O OH O DME 4H-phenol 1H-phenol 2H-phenol rt,18 h H+ W W

TfO– N N Me Me Me + Me 3a 3b, 57% NH NH2 NH DiPAT = (i-Pr)2NH2OTf 4H-aniline 1H-aniline 2H-aniline W = {TpW(NO)(PMe3)}

Scheme 1. Ligand Substitution Reactions of Phenol, Aniline, and N,N- Figure 4. Tautomeric Forms of Phenol and Aniline. (Ref. 15–17) Dimethylaniline. (Ref. 16,17) 47 VOL. 45, NO. 2 • 2012 respectively) to a methanolic solution of 2 generates 5-substituted OMe 21 H E 4-methoxycyclohexenones exclusively as their syn isomers (Scheme 3). 1. E+ Additionally, when amines or thiols are added to the reaction W W H 2. MeOH solution following addition of mCPBA, 1,2-amino alcohols and O O 1,2-hydroxy thioethers are produced. An epoxide intermediate is 2 5a–c suspected, but, despite significant efforts, this epoxide has not yet E been isolated. However, starting with the meta-cresol analogue, 6, W + 3,3-dimethyldioxirane (DMDO) oxidation delivers the corresponding epoxide, 7, in good yield (Scheme 4).22 Subjecting the meta-cresol– O tungsten complex, 6, to DMDO in the presence of different nucleophiles 5 E Conditions for Step 1 Yield produces 4-substituted 5-hydroxy-5-methylcyclohexenone complexes o a OH mCPBA, CH2Cl2, 0 C 82% 8–11 with high stereofidelity. o b Br PIFA, MTPBr, CH2Cl2, –40 C 74% The reactivity displayed by the dimethylanilinium analogue, c F Selectfluor® 79% 17 3b, closely resembles that of complexed phenol 2 (Scheme 5). PIFA = PhI(O2CCF3)2; MTPBr = Ph3PMeBr. Electrophiles add to the meta position of the aniline ring, despite such action creating a dicationic intermediate. The more electron-deficient Scheme 3. Tandem Electrophilic and Nucleophilic Additions to the anilinium system, 3b, offers greater resistance to the competing metal- 2H-Phenol Complex. (Ref. 21) centered oxidation, compared to its neutral analogue, 3a, and the phenol analogue, 2. PhNH OH OH OH 2.2. Derivatization of Pyridine W Me W Me Inorganic and organometallic chemistry has a rich history of using O O pyridine-based ligands to modify the steric and electronic properties of 9, 88% 8, 67% transition-metal complexes. All of this chemistry is possible because of (i) coordination of the pyridine nitrogen to metal centers. Unfortunately, H2O 1. K2CO3, H2O, H2C(OMe)2 CH2Cl2, rt this is also true for pyridine when combined with {TpW(NO)(PMe )}. ® 3 MeCN, Me2CO, OXONE 4 O Me o However, nitrogen coordination may be prevented by substituents 0 C, 2 h 3 Me W W at the 2 position, provided they are sufficiently bulky or are good 2. H2O, CH2Cl2, rt 2 1 23 π-electron donors (Scheme 6). Furthermore, blocking the nitrogen as O [DMDO oxidation] O in organoborane 12 prevents N coordination and provides access to the 6 7, 91% 2H-meta-cresol– 2 24 indoline parent η -pyridine, isolated as its conjugate acid, 13 (Scheme 7). The tungsten complex formation of organoborane adduct 12 and its conversion into 13 can be Cl CH2Cl2, 24 h performed in high yield (87% and 92%, respectively) on a >10 g scale. mCPBA, CH2Cl2 H O, 0 oC, 15 min When exposed to acetic anhydride, pyridinium 13 is readily converted 2 O O N 24,25 OH into N-acetylpyridinium complex 14 (94%; see Scheme 7), setting OH the stage for an extensive range of carbon-centered reactions governed W Me W Me by the metal complex. O O Two synthetic pathways may be utilized to convert the bound 10, 74% 11, 86% pyridine into functionalized tetrahydropyridines. The first path (i) PhNH , CHCl , PhNH OTf, CH Cl , rt, 5 h. involves modification of the exposed alkene followed by nucleophilic 2 3 3 2 2 Scheme 4. Isolation of Suspected Epoxide Intermediate and Subsequent Ring-Opening Additions. (Ref. 22)

(i-Pr)2NEt R1R2CHCOCl R1 R2 OMe H O Selectfluor® F R1 δ+ W W W + δ– • O W H MeCN, MeOH R2 rt, 10 min N N O O Me + Me Me + Me 24a–c 3b 91% ® 1 2 NCS Selectfluor R R MeCN, MeOH MeCN, H O, 2 min 5 min 2 O– W + OMe OH Cl F O W W 4ai–4ci 1 2 1 2 N N 4a: R = H, R = Cl (92%); 4b: R = H, R = 3-MeOC6H4 (81%); Me + Me Me + Me 4c: R1 = R2 = Cl (57%) 83% 77% Scheme 2. Ketene [2 + 2] Cycloaddition Reaction with the 2H-Phenol Scheme 5. Tandem Addition Reactions with the 2H-Anilinium Complex. The Complex. (Ref. 16) Triflate Counterion Is Omitted for Clarity.(Ref. 17) Opening New Chemical Space through Novel Dearomatization Reactions 48 Daniel P. Harrison and W. Dean Harman*

addition to the iminium carbon atom (Scheme 8, Path 1).26 To this end, addition of Selectfluor® or N-chlorosuccinimide to 14 in methanol OMe NW results in the dialkoxylation of the exposed alkene (Scheme 9).27 W N W N Addition of nucleophiles to the resulting acyl iminium salt 15 results in OMe Me tetrahydropyridine complexes 16 and 17 in good yields. Me The second pathway reverses the alkene modification and W N W N nucleophilic addition steps (see Scheme 8, Path 2). N-Acetylpyridinium

Me Et is invoked as an intermediate in the catalytic acylation of alcohols and

Me3P amines with acetic anhydride. Conversely, addition of mild nucleophiles NO W W N to 14 results solely in addition to one position of the pyridine ring and N W 2 24,27 Tp produces η -1,2-dihydropyridine (DHP) complexes 18–28 (eq 1). -coordinated NMe2 Bn -coordinated 2 1 η η As with phenol and aniline complexes, the metal fragment blocks attack from one face of the pyridine ligand, making the addition not W N W N only regioselective but also stereoselective. OMe MeHN Functionalization of the C5–C6 double bond via Path 2 in Scheme 8 Me 28 O is also possible. Enamides, like enamines, are polarized such that the W N b carbon is nucleophilic.29 This implies that addition of an electrophile W O TMS N 28 W MeS should occur at C5, as shown in Figure 5, Part (a). However, studies N of h2-coordinated 1,3-diene complexes with π-basic metals indicate a clear regiochemical preference for electrophilic addition at the terminal Scheme 6. Coordination Modes of Pyridines with Different Substituent and carbon of the uncoordinated double bond.30,31 By analogy, electrophiles Electronic Effects.(Ref. 23) react with dihydropyridine (DHP) complexes at C6. Addition of a proton to 18, followed by isolation and characterization of the cationic complex, 29, reveals that the metal, not the enamide, dictates the placement of the electrophile, representing an umpolung of N BH (neat) 3 28 W W typical enamide reactivity (eq 2). Similar results were obtained upon rt, 15 h NBH3 addition of acid to other DHP complexes. It should be noted here that 1 12, 87% both rotational isomers of the amide bond are often observed in the cdr = 3:1 28 Ph2NH•HOTf NMR spectra of the products. ether–acetone rt, 14.5 h It is worth noting that the metal–allyl bonds, both in crystallographic data and in solution NMR spectra, are observed to be highly distorted. This type of s–π or “η2-allyl” distortion has been attributed to the Ac2O W W interaction of the allyl π* orbital and the d orbital orthogonal to the N H N Me + DTBP, MeCN + nitrosyl, and has significant implications for the reactivity of the o – TfO– 55 C, 5.5 h TfO O complexes (and of other coordinated ligands as well).28,32 This asymmetry 13, 92% 14, 94% indicates that one of the allylic carbon atoms (C3) is considerably more cdr = 1:1 cdr > 10:1 electrophilic than the other (C5). Nucleophilic reagents react with the cdr = coordination diastereomer ratio DTBP = 2,6-di-tert-butylpyridine pyridine allyl exclusively at the C3 position, thereby desymmetrizing the heterocyclic ring to form D3-piperidinamides 31–34 (Scheme 10).28 As above, all the tetrahydropyridine (THP) compounds are typically Scheme 7. Synthesis of a Trapped h2-Pyridine: N-Acetylpyridinium. (Ref. 24,25)

1. Selectfluor® R R OMe MeOH, MeCN OMe N N E rt, 2 h W W M N Me 2. THF, rt, 16 h N Me Nu + + R R O O 14 15, 58% E+ [O] cdr = 10:1 cdr > 20:1 N E (i) M (ii) NaCN, MeCN NaBH4 Nu– R R Path 1 rt, 1.5 h THF–MeOH 4 N E N E –20 oC, 3 h M Path 2 M 23 Nu– Nu (ii) OMe OMe N E (i) OMe OMe M W W Nu N Me N Me (i) Alkene modification; (ii) nucleophilic addition. CN O O 16, 72% 17, 71% Scheme 8. Divergent Synthetic Pathways for Converting Pyridine into Piperidines. (Ref. 26) Scheme 9. Alkene Modification Followed by Nucleophilic Addition. (Ref. 27) 49 VOL. 45, NO. 2 • 2012 observed as mixtures of conformational isomers (i.e., amide rotational (i.e., C=O) adds α-to-N in the DHP ring. Addition of trichloroacetyl isomers). isocyanate resulted in a mixture of [4 + 2] cycloadducts, but eventually When nucleophiles are added to a cationic complex containing a converted solely to an unproductive electrophilic substitution product. 2-ethyl substituent, 30, they do not add to C3 but rather to the other Addition of acid to the [4 + 2] tosyl isocyanate cycloadducts produced allylic position, C5, resulting in complexes 35 and 36 (Scheme 11).28 highly asymmetric allyl complexes 47 and 48 (Scheme 14).34 When Presumably, the vicinal addition of two nucleophiles creates a steric nucleophiles were added to the ethyl allyl derivative 47, in an attempt to interaction that overcomes the electronic bias for C3 addition described above.

In a special case, addition of two equivalents of acid to 19 produces (a) Enamide Influence E a dicationic allylic isoxazolium complex, 37, often referred to as 5 33 28 + a Reissert-like salt (Scheme 12). Attempts to add nucleophiles 6 E WWW compatible with 29 and 30 have resulted in deprotonation to generate a N Me N Me N Me tautomer of the 2-cyano DHP complex. However, 1-methoxy-2-methyl- R O R O R O 1-(trimethylsilyloxy)propene reacts as a nucleophile at C3, producing versus (b) Metal Influence ® 38. Addition of NaBH4 or DABCO to 38 results in the reduction or 5 E deprotonation of the isoxazolium to produce the novel structures 39 6 E+ W W + W + and 40. N Me N Me N Me In the presence of Lewis acids, addition of methyl vinyl ketone R O R O R O or trans-cinnamaldehyde to the dihydropyridine complex 18 results in cycloadducts 41 or 42 (Scheme 13).34 Again, the relative stereochemistry and connectivity of the [4 + 2] cycloadducts are Figure 5. Competing π-Donor Polarizations. (Ref. 28) consistent with a reversed polarization of the C5–C6 bond.35-40 Notably, the cycloaddition products are formed with endo stereochemistry. 2-Substituted DHP complexes (e.g., 20) also appear to be capable of cycloaddition reactions with Michael acceptors, but, to date, have not Me O been explored in detail. N O N Me W W + W + Addition of tosyl isocyanate to DHPs 20, 22, 25, and 27 produces N Me [4 + 2] cycloaddition products 43–46 (eq 3).34 X-ray crystallography R R R O ab has confirmed that the polarization imparted by the metal, as described 18, R = H; 20, R = Et 29, R = H; 30, R = Et above, is maintained in that the electrophilic portion of the isocyanate a and b are C–N rotational isomers.

R = H: TfOH, CH2Cl2, rt, 2 min; 96%, 29a:29b = 3.1:1 R = Et: TfOH, MeCN, rt, 1 min; 97%, 30a:30b = 2.7:1 – TfO– Nu W W N Me N Me eq 2 (Ref. 28) H 14 O Nu O

W W W N Me N Me N Me H H O O CN O Et O N Me 18, 89% 19, 88% 20, 88% W

W W W CN O N Me N Me N Me 31, 57% H H H LiO N Me O MeO O O O O W O2N OMe O NaCN, CH2Cl2 MeO DMSO, 0 oC, 1 h MeO OMe Henry reaction Reformatsky reaction Baylis–Hillman reaction O CH2Cl2 21, 88% 22, 87% 23, 92% h O O 5 oC, 1.3 N Me 0 W + 32, 67% Me W W W 4 N Me N Me N Me 3 Me OTMS H H H 29 CH Me O O Me O 2 Cl OMe O ZnEt2, CuCN 2 , rt, 10 min O CH Cl , THF N Me 2 2 W 24, 57% 25, 89% 26, 47% 0 oC, 3 h O MeO Me W W N Me Me N Me N Me W O H H 33, 58% O O Et NH 34, 43% TMS 27, 89% 28, 61% Scheme 10. Stereoselective Nucleophilic Addition to C3 of the Asymmetric eq 1 (Ref. 24,27) h2-Allyl Group. (Ref. 28) Opening New Chemical Space through Novel Dearomatization Reactions 50 Daniel P. Harrison and W. Dean Harman*

synthesize additional tri- and tetrasubstituted tetrahydropyridine (THP) complexes, deprotonation resulted in regeneration of the [4 + 2] adduct, O Et 43, or an overall electrophilic substitution product, 48 (see Scheme 5 N Me ZnEt2, CuCN 14). Substituting a proton for the ethyl group, allows for nucleophilic W + W + W N Me N Me 4 Et CH2Cl2, THF addition to occur at the allyl regioselectively, producing 50–52 from 3 o H 34 –30 C, 52 h Et O Et O 49 (Scheme 15). Apparently, bulky substituents at the C2 and C6 30 35 20 positions lead to deprotonation of the allylic fragment while the lack 1.9 : 1 O OLi 28% of one of these substituents allows access to a terminal allylic position.

MeO OMe O O o 2.3. Derivatization of Other Aromatic Compounds MeCN, 0 C, 2 h 2 MeO OMe 2.3.1. Substituted h -Pyridine Complexes As noted above, other η2-pyridine complexes have been isolated in W N Me reasonable yields (see Scheme 6; pink, blue, and brown compounds). Et O These coordinated ligands have also been successfully modified under 36, 41% mild conditions. For example, complexes of 2-(N,N-dimethylamino)- pyridine (53), 2,6-dimethoxypyridine (54), and 2,6-lutidine (55) all undergo concerted [4 + 2] Diels–Alder cycloadditions at the exposed Scheme 11. Stereoselective Nucleophilic Addition to C5, Demonstrating That Steric Interaction of the Ethyl Group at C2 with the Incoming and activated 2-azadiene system to provide complexes 56–58, 2 Nucleophile Steers the Latter to the Less Electrophilic C5 Position. (Ref. 28) respectively (eq 4). This type of reaction has not been realized in the absence of the dihapto-coordinated metal.41–44

2.3.2. Pyrrole Me + Much like aniline, when the nitrogen is not sufficiently blocked or HOTf (2 equiv) 2 N – W W + O ( OTf)2 substituted, h -pyrrole complexes undergo N–H oxidative insertion N Me MeCN, rt reactions (eq 5).45,46 Treatment of the benzene complex 1 with CN O NH2 2,5-dimethylpyrrole results in a dihapto-coordinated 3H-pyrrole, 59. 19 37, 98% This unusual tautomer has kinetic access to the 1H form, generating MMTP, MeCN Et rt, 10 min an enamine, where addition of carbon electrophiles readily occurs N W H O Me H NaBH + MeO NH 4, MeOH N O Me 2 W O –OTf Me rt, 10 min H O Ts-NCO Ts H Me W N 39, 84% NH2 NAc MeO NAc CH2Cl2, rt Me W R ® H H O O R H DABCO N Me 38, 88% W H MeCN SM R Time Prod Yield o C, 7.5 h CN 58 H OTMS OMe 20 Et 1.1 h 43 84% Me MMTP = Me Me MeO 22 MeO CCH 1.8 h 44 67% O 2 2 Me 25 η1-C H 1.6 h 45 80% 40, 82% 3 5 27 TMSC≡C 7.5 h 46 38% eq 3 (Ref. 34)

Scheme 12. Synthesis and Elaboration of a Reissert-like Allyl Salt. (Ref. 28)

O O NHTs H H O Ts N O NAc TfOH, MeCN NAc W + Me W Et rt, 9 min Me H Et NAc H H 3, MeOH Yb(OTf) 43 47, 88% 5 rt, 17.5 h W 6 41, 44% + Nu– W HOAc (cat) NAc BF CH2Cl2, rt, 5 h 18 DTBP3 •Et 2 O, C – HNu , rt, D O Ph Ph 2 C 1.5 h l3 O H NAc O NHTs W H W NAc 42, 65% H Et DTBP = 2,6-di-tert-butylpyridine 48, 87%

Scheme 13. Michael Acceptor Additions to a Simple DHP Complex. (Ref. 34) Scheme 14. Allyl Synthesis and Addition of Nucleophiles. (Ref. 34) 51 VOL. 45, NO. 2 • 2012 at C3. For example, α,β-unsaturated ketones, esters, and aldehydes undergoes conjugate addition at C3 to produce 67 (Scheme 18).47 It can undergo Michael additions stereo- and regioselectively at C3 to should be noted that mixtures are sometimes observed resulting from produce 60a–g (Scheme 16).46 In some cases, the ketones of the a competing nucleophilic addition directly to C3 (68). Compounds Michael addition products (i.e., 61a–c) undergo stereoselective Aldol of type 67 are often most easily isolated as their conjugate acids reactions with one of the methyl groups of pyrrole to produce bi- (62) 69a–c (see Scheme 18). This feature also allows for the possibility and tricyclic (63) structures (see Scheme 16). This “Michael–aldol– of stereoselectively reducing the resulting methyl enonium to an allyl ring-closure” sequence is facilitated by metal coordination, which ether, which can be further modified (e.g.,70 , 71a,b). stops deprotonation at C3 from pre-empting formation of the external In one unusual example, addition of thiophenolate to 66h occurs enamine. The stereoselectivity of the ring-closure appears to be a result at C3 rather than the carbonyl. The product, 72, then undergoes of hydrogen bonding of the ketone with the enamine in the transition an aldol reaction to form 73, followed by methyl cleavage to state, which favors a synclinal approach. generate 74 (Scheme 19).47 This sequence produces a complex cis- oxahexahydronaphthalene core and sets three stereocenters. 2.3.3. Anisole

When anisole is coordinated to {TpW(NO)(PMe3)}, either by reduction of TpW(NO)(PMe3)Br in anisole or by ligand substitution from benzene complex 1, an equilibrium mixture of the coordination isomers W + 2 1 is isolated (64).47 However, addition of acid to the mixture produces a R N R H 1 single isomer of a 2H-anisolium complex, 65. This anisolium complex R 1 pentane, = rt,R 15 h 47 2 may be used as a starting material for other reactions (Scheme 17). = Me For example, 65 undergoes Michael addition to produce compounds of DME, rt type 66 in high de’s. Nucleophiles, such as methoxide, cyanide, and the

3 enolate of 2,4-pentanedione, react with these complexes in a cascade 3 W 1 W 2 R2 R 2 reaction to produce 1-oxahexahydronaphthalene complexes of type 67. N Me Me H Me N Me N Nucleophilic addition to the ketone produces an alkoxide which then W H 7-coordinated 59, 85% tungsten η2-coordinated R1 R2 Time Yield dimethylpyrrole O NHTs O NHTs O OLi H H H H 4 h 64% 5 NAc MeO OMe NAc H Me 18 h 31% W + W 4 MeCN, 0 oC, 16 h 3 H ZnEt MeO OMe 49 THF, rt, 10 2 , CuC eq 5 (Ref. 45) O O N min 50, 78% N H O NHTs CHCl3, EtOH H rt, 21 h NAc W R1 O NHTs 1 3 3 R H H 3 W W Z W 2 Z Et (a) 2 2 NAc Me Me Me Me W 51, 47% Me N Me N THF or DME N H rt, 20–24 h 59 60a–g, 30–64% H Michael addition NH 1 R = H, Me, CO2Me; Z = CHO, COMe, COEt, CO2Me 52, 66% R2 R2 R3 O R3 O R3 THF, CH2Cl2, H OH Scheme 15. Nucleophilic Addition to C3. (Ref. 34) W or CHCl3 W W (b) R2 Me N Me rt, 12–48 h Me N CH2 Me N H 61a–e 62a–d 36–89% (from 59) R2 = H, Me, Et; R3 = H, Me

Z Z O R2 1 R H W O N N W W W R2 (c) 59 + OH Me R1 MeOH–H2O N Me Me N rt, 48 h SM R1 R2 Prod 1 : 1 63, 15%

53 Me2N H 56 54 MeO MeO 57 55 Me Me 58

Scheme 16. Michael Additions and Michael–Aldol–Ring-Closures to eq 4 (Ref. 2) 2,4-3H-Pyrrole Complexes. (Ref. 46) Opening New Chemical Space through Novel Dearomatization Reactions 52 Daniel P. Harrison and W. Dean Harman*

3. Demetallation O The strategy most commonly employed for removing the {TpW(NO)(PMe )} O 3 R fragment involves oxidation of the metal. Metal oxidation reduces the Ph2NH2OTf R 3 W W W electron density utilized to form stable π bonds with the coordinated DMF, rt, 1 h 2 ligand, thus weakening the metal’s hold on the π-coordinated ligand. OMe OMe OMe + + After the ligand is released, chromatography is often employed to 64 65 66 facilitate the separation of the metal decomposition products from the Me O R = H, 94%; R = Me, 91% R = H h desired organic compound. Here we provide several examples of the O o C, 22 O –20 liberation of novel organic compounds. THF, O O Me O Reduction of the keto group in 4b with NaBH4, followed by W Me Me Nu oxidation with N-bromosuccinimide (NBS) allows for the isolation + Na 16,21 OMe W of the corresponding tricyclic cyclobutanol (Scheme 20). Other 67, 59% tandem addition products (75–79) derived from phenol may be isolated dr = 4:1:1 OMe 1-oxahexahydro- 68 through the use of ceric ammonium nitrate (CAN) or 2,3-dichloro-5,6- naphthalene core dicyanoquinone (DDQ).21,22 Although isolation of organic compounds derived from aniline have not yet been reported, preliminary experiments Scheme 17. Synthesis of Anisolium Complexes and Oxahexahydronaphtha– are promising for their decomplexation and isolation as enones.48 lenes. (Ref. 47) Treatment of various D3-piperidine complexes with one equivalent

of either CAN, DDQ, or I2 successfully liberates the heterocyclic ligand.28 Molecular oxygen can also be used as a decomplexing agent, and the highest recovery of organic compound by this method R R O Nu1 Nu1 is usually obtained by stirring MeCN or EtOAc solutions of the NaCN or 49 R O O complex and silica overnight in a flask under 1 atm of2(g) O . As a NaOMe Ph2NH2OTf W W W general rule, complexes with anodic peak potentials (E ) of more rt, 0.5 h p,a DMF–H2O than ~ 0.5 V (vs NHE), are resistant to oxidation with O2(g). In these + OMe or MeOH OMe + OMe 0 oC, 1–3.5 h cases, the use of CAN is indicated. Isolating the tetrahydropyridine 66h, R = H 67 69a–c, 75–93% (THP) complexes by precipitation is often inefficient. Better yields are 66m, R = Me Nu1 = CN, OMe dr = 1:1 to 4:1 obtained if THP complexes are generated in situ (Scheme 21),28 then NaBH4, MeOH 0 oC, 0.5 h directly oxidized. The reactions described here constitute a procedure to generate 1,2,5-trisubstituted tetrahydropyridinamides, 80–90, with a CN CN CN diverse range of substituents, in overall yields from pyridine borane O O O of 21–28% for the five-step process (>75% per step). Derivatives of CAN Nu2 TfOH, MeCN W + W the [4 + 2] cycloaddition products, 91–99, may be isolated using DDQ –40 oC, 0.5 h or O2(g), generating 1,2,5-trisubstituted and 1,2,5,6-tetrasubstituted Nu2 OMe tetrahydropyridinamides (Scheme 22).34 2 73% 71a,b (Nu = H, CN) 70, 54% Metal oxidation yields a variety of new 2-azabicyclo[2.2.2]- 36–40% (2 steps) R = H, Nu1 = CN

Scheme 18. Modified Procedure for Forming Oxahexahydronaphthalene OMe OMe OMe Cores. (The Final Complexes Were Not Isolated but Were Carried on to the Demetallation Step.) (Ref. 47) H O H OH H OH NaBH4 NBS (a) W H W H H O CH2Cl2–MeOH CH2Cl2–MeCN (1:1) (1:1) O O rt, 5 min O rt, 10 min O 4b 66%48% H PhSH, NaH SPh W W OMe SPh THF, rt, 1.5 h F OH OMe OMe + 2H

66h 72 2CO R3 CAN, PhCO O O MeCN–Mert, 24 h HSPh R2 75, 57% 76, 46% (b) W R1 DDQ, Me H H rt, 15– 2CO NHPh SPh OBz O 25 min SPh SPh OH OH OH W H W H H H Me Me Me H H O OH + O OH CH3 O O O 74, 93% 73 77, 55% 78, 59% 79, 54% –SPh

Scheme 19. Synthesis of a Bicyclo[3.3.1]nonenone. (Ref. 47) Scheme 20. Demetallation of 2H-Phenol Derivatives. (Ref. 16,21) 53 VOL. 45, NO. 2 • 2012 octadienes, 100–103, derived from substituted pyridines (Scheme 23).2,50 Liberation of the 3H-pyrrole–Michael addition product and (a) [4 + 2] Cycloadducts and 1,2,5,6-Tetrasubstituted THPs Michael–Aldol ring-closure product with CAN is achieved, but such X X action is accompanied by a rearomatization of the pyrrole portion of the H H Y (i) or (ii) Y new organic compounds, 104–106 (Scheme 24).46 NAc NAc W R R H H Oxidation of anisole-derived complexes 71a,b with CAN produces H H 47 o new oxadecalin cores, 107a,b (Scheme 25). In one interesting case, (i) DDQ, MeCN or Me2CO, rt or 55 C, 1–1.2 h the action of Cu(II) in the presence of molecular oxygen triggers a (ii) CAN, MeCN, rt, 0.7–1 h O O O conjugate addition of pyrazole (either derived from decomposition of Ph Ts the Tp ligand or independently added) to the enone during the oxidation Me H N NAc NAc NAc procedure to form the tricyclic ketone 109. In the absence of air, the Et corresponding enone, 108, is isolated. 91, 43% 92, 86% 93, 37%

As proof of concept that the {TpW(NO)(PMe3)} metal fragment can O O O produce organic compounds in enantiomeric excess, a procedure was Ts Ts Ts N N N employed in which one enantiomer of the metal irreversibly binds to NAc NAc NAc O (R)-α-pinene, (R)-110, and the other coordinates reversibly to (R)-α- 2 OMe pinene, (S)-110 (Scheme 26). Heating the mixture of diastereomers TMS in a solution of 2,6-lutidine containing some acid, releases the more 94, 37% 95, 49% 96, 30% weakly coordinated (R)-α-pinene–tungsten complex, (S)-110, and allows for the isolation of a cationic η2-lutidinium complex, (S)- (b) 1,2,5-Trisubstituted THP 111, in enantiomeric excess. Addition of base, followed by [4 + 2] Ts Ts cycloaddition with acrylonitrile, and oxidation with AgOTf allows for O NH O NH H H the isolation of the azabicyclic compound (S)-113, in an enantiomeric NAc (iii) or (iv) NAc ratio (er) of 9:1 (ee = 80%).2 W H H R R

(iii) DDQ, MeCN, rt, 1–1.2 h. (iv) O2(g), SiO2, EtOAc, rt, 21 h R3 R3 R2 R2 Ts Ts Ts CAN, I2, O2, or DDQ W W O NH O NH O NH NAc NAc NAc H H H R1 R1 R1 NAc NAc NAc dihyropyridine tetrahyropyridine tetrahydropyridine complex (DHP) complex (THP) H H H MeO OMe Et O O Me Me Me Me O O NH MeO MeO Et

O 97, 19% (DDQ) 98, 31% (DDQ) 99, 26% (O2(g)) NAc NAc NAc Et Scheme 22. DDQ and CAN Demetallation of Tetrahydropyridine Compounds a c 80, 34% (O2(g)) 81, 31% (I2) 82, 40% (O2(g)) Derived from the Pyridine–Tungsten Complex. (Ref. 34) O O O O O O O O

MeO OMe MeO OMe MeO OMe MeO OMe

Z Z 2 2 NAc NAc NAc NAc R [O] R N N Et MeO W R1 R1 ON2 O a a a 83, 34% (CAN) 84, 40% (O2(g)) 85, 28% (O2(g)) 86, 67% (CAN) Z Z a OMe OMe 36% (DDQ), 34% (O2(g)) N N O O O O MeO MeO Me Me 100a 101b,c OMe MeO OMe MeO MeO NH2 H Me H Me Z = CN (35%) Z = CH2OH; 51%, dr < 2:1 OMe CN O Z = PhSO (35%) Z = MeCHOH; 34%, dr > 10:1 H H 2 NAc NAc NAc N Et H NC NC CN NMe2 O N NH TMS MeO d e a b c 102, 87% (NMR) 103, 29% 87, 40% (CAN) 88, 55% (CAN) 89, 55% (O2(g)) 90, 34% (O2(g))

a b c a b From DHP (3 steps, 1 pot). From THP (1 step). From isoxazolium CAN, CD3CN, rt, 3 or 14 h. mCPBA, CDCl3, rt, 14 h. c d monocation (2 steps, 1 pot). CAN, CDCl3, rt, 14 h. AgOTf, (CD3)2CO, rt, 18 h. e CuBr2, CD3CN, rt, 14 h Scheme 21. Comparison of Demetallation Methods of Tetrahydropyridine Compounds Derived from the Pyridine–Tungsten Complex. (Ref. 28) Scheme 23. Liberation of 2-Azabicyclo[2.2.2]octadienes. (Ref. 2,50) Opening New Chemical Space through Novel Dearomatization Reactions 54 Daniel P. Harrison and W. Dean Harman*

4. Conclusion chemical space for small molecules derived from aromatic compounds. This review surveyed organic transformations that are facilitated by These new compounds are of potential biological or medicinal

the tungsten complex {TpW(NO)(PMe3)} and which have no organic significance. To this end, a number of samples have been submitted to or organometallic precedent. The dramatic activation of the aromatic the National Institutes of Health Molecular Libraries Small Molecule substrates is a direct result of the strong π back-bonding interaction Repository (MLSMR) for storage and high-throughput biological demonstrated by the tungsten. Not only can the metal be thought of as a screening.51 protecting group for one of the double bonds, as the coordinated double bond is essentially rehybridized to an sp3 center, but the metal is also an electron-donor group that activates the π system toward electrophilic (R)-α-pinene Me addition reactions. Often this results in an electronic repolarization of (97% ee) W W(S) + W(R) K2CO3, DME alkene fragments in conjugation with the metal. rt, 18 h Me Several methodologies have been developed to liberate and isolate 1 (racemic) (S)-110 (R)-110 mismatched matched the modified ligands from the {TpW(NO)(PMe3)} fragment. Thus, this tungsten system provides a methodology capable of accessing new mixture of coordination diastereomers

lutidine (solvent) lutidinium triflate (0.5 equiv) 60 oC, 24 h

Me Me OH OH DBU H W + W(S) + W(R) Me Me N (S) acetone N CAN, CDCl3 H Me (a) W – o Me TfO Me Me N 40 C, 24 h Me N H (S)-112, 64% (S)-111a (R)-110 62a 104, 63% a H2C=CHCN (S)-111 precipitates out of ether O O DME–lutidine solution and is separated from (R)-110. rt, overnight H

W Me Me CN CN Me AgOTf, Me2CO Me Me N Me N 105, 6% CAN, CDCl3 H N rt, overnight N (b) + + Me W(S) Me 40 oC, 24 h H (S)-58 (Z = CN) (S)-113 OH OH 80% 86%, 80% ee W Me N Me N H Scheme 26. Example of the Synthesis and Isolation of an Organic in 63 106, 26% Enantiomeric Excess. (Ref. 2) 105 and 106 were cleanly separated by column chromatography. Yields of 105 and 106 are of the separated products.

5. References and Notes Scheme 24. Demetallation of 3H-Pyrrole Derivatives. (Ref. 46) (1) Todd, M. A.; Grachan, M. L.; Sabat, M.; Myers, W. H.; Harman, W. D. Organometallics 2006, 25, 3948. (2) Graham, P. M.; Delafuente, D. A.; Liu, W.; Myers, W. H.; Sabat, M.; Harman, W. D. J. Am. Chem. Soc. 2005, 127, 10568.

CN CN (3) Pike, R. D.; Sweigart, D. A. Synlett 1990, 565. (4) Sun, S.; Dullaghan, C. A.; Sweigart, D. A. J. Chem. Soc., Dalton O O CAN, CDCl3 (a) W Trans. 1996, 4493. rt, 24 h (5) Astruc, D. Tetrahedron 1983, 39, 4027. Nu Nu (6) Kane-Maguire, L. A. P.; Honig, E. D.; Sweigart, D. A. Chem. Rev. 71a,b 107a,b 1984, 84, 525. Nu = H (40%), CN (35%) (7) Pearson, A. J.; Park, J. G. J. Org. Chem. 1992, 57, 1744. H H (8) Pearson, A. J.; Chelliah, M. V. J. Org. Chem. 1998, 63, 3087. CuBr , CD CN H SPh 2 3 H SPh (9) Kündig, E. P.; Fabritius, C.-H.; Grossheimann, G.; Romanens, P.; (b) W H H rt, 1.5 h Butenschön, H.; Wey, H. G. Organometallics 2004, 23, 3741. H H O OH O OH (10) Davies, S. G.; McCarthy, T. D. In Transition Metal Organometallics 74 108, 50% in Organic Synthesis; Comprehensive Organometallic Chemistry II Series, Vol. 12; Abel, E. W., Stone, F. G. A., Wilkinson, G., Series N NH H N N Eds.; Pergamon: Oxford, 1995; Chapter 9.3. O2, Cu(OTf)2 H SPh (11) Kündig, E. P.; Pape, A. Dearomatization via h6-Arene Complexes. CD3CN, rt, 16 h H H In Transition Metal Arene p-Complexes in Organic Synthesis and H OH O Catalysis; Kündig, E. P., Ed.; Topics in Organometallic Chemistry 109, 95% (NMR) Series, Vol. 7; Springer: Berlin, 2004; pp 71–94. (12) Keane, J. M.; Harman, W. D. Organometallics 2005, 24, 1786. Scheme 25. Demetallation of Anisole Derivatives. (Ref. 47) (13) Graham, P. M.; Meiere, S. H.; Sabat, M.; Harman, W. D. Organometallics 2003, 22, 4364, and references 1–3 therein. 55 VOL. 45, NO. 2 • 2012

(14) Welch, K. D.; Harrison, D. P.; Lis, E. C., Jr.; Liu, W.; Salomon, R. J.; (42) Neunhoeffer, H.; Lehmann, B. Justus Liebigs Ann. Chem. (presently Harman, W. D.; Myers, W. H. Organometallics 2007, 26, 2791. Eur. J. Org. Chem.) 1975, 1113. (15) Shiner, C. S.; Vorndam, P. E.; Kass, S. R. J. Am. Chem. Soc. 1986, 108, (43) Gompper, R.; Heinemann, U. Angew. Chem., Int. Ed. Engl. 1980, 19, 5699. 216. (16) Todd, M. A.; Sabat, M.; Myers, W. H.; Harman, W. D. J. Am. Chem. (44) Boger, D. L. Chem. Rev. 1986, 86, 781. Soc. 2007, 129, 11010. (45) Myers, W. H.; Welch, K. D.; Graham, P. M.; Keller, A.; Sabat, M.; (17) Salomon, R. J.; Todd, M. A.; Sabat, M.; Myers, W. H.; Harman, W. D. Trindle, C. O.; Harman, W. D. Organometallics 2005, 24, 5267. Organometallics 2010, 29, 707. (46) Welch, K. D.; Harrison, D. P.; Sabat, M.; Hejazi, E. Z.; Parr, B. T.; (18) Brady, W. T. Tetrahedron 1981, 37, 2949. Fanelli, M. G.; Gianfrancesco, N. A.; Nagra, D. S.; Myers, W. H.; (19) Ketenes, Allenes and Related Compounds: Volume 2; Patai, S., Ed.; Harman, W. D. Organometallics 2009, 28, 5960. Patai’s Chemistry of Functional Groups Series; Wiley: New York, (47) Lis, E. C., Jr.; Salomon, R. J.; Sabat, M.; Myers, W. H.; Harman, W. D. 1980. J. Am. Chem. Soc. 2008, 130, 12472. (20) Amouri, H.; Le Bras, J. Acc. Chem. Res. 2002, 35, 501. (48) A manuscript is currently in preparation describing the isolation of (21) Todd, M. A.; Sabat, M.; Myers, W. H.; Smith, T. M.; Harman, W. D. J. substituted enones derived from the N,N-dimethylanilinium complex. Am. Chem. Soc. 2008, 130, 6906. (49) Control reactions found that silica was not necessary for demetallation

(22) Zottig, V. E.; Todd, M. A.; Nichols-Nielander, A. C.; Harrison, D. P.; with O2(g), but that it dramatically decreases the required reaction time Sabat, M.; Myers, W. H.; Harman, W. D. Organometallics 2010, 29, (from 1 week to <15 h). 4793. (50) Kosturko, G. W.; Graham, P. M.; Myers, W. H.; Smith, T. M.; Sabat, (23) Delafuente, D. A.; Kosturko, G. W.; Graham, P. M.; Harman, W. H.; M.; Harman, W. D. Organometallics 2008, 27, 4513. Myers, W. H.; Surendranath, Y.; Klet, R. C.; Welch, K. D.; Trindle, C. (51) The compounds are listed on Pubchem and can be found via a substance O.; Sabat, M.; Harman, W. D. J. Am. Chem. Soc. 2007, 129, 406. search. (24) Harrison, D. P.; Welch, K. D.; Nichols-Nielander, A. C.; Sabat, M.; Myers, W. H.; Harman, W. D. J. Am. Chem. Soc. 2008, 130, 16844. Trademarks. DABCO® and Selectfluor® (Air Products and (25) Other electrophiles could be used in place of acetic anhydride. Chemicals, Inc.); OXONE® (E. I. du Pont de Nemours and Company, (26) Harrison, D. P.; Zottig, V. E.; Kosturko, G. W.; Welch, K. D.; Sabat, Inc.). M.; Myers, W. H.; Harman, W. D. Organometallics 2009, 28, 5682. (27) Kosturko, G. W.; Harrison, D. P.; Sabat, M.; Myers, W. H.; Harman, W. About the Authors D. Organometallics 2009, 28, 387. Daniel P. Harrison was born in 1983, and received his B.Sc. degree (28) Harrison, D. P.; Sabat, M.; Myers, W. H.; Harman, W. D. J. Am. Chem. from the Virginia Military Institute in December of 2004. He then Soc. 2010, 132, 17282. worked for the Virginia Institute of Forensic Science and Medicine on a (29) Carbery, D. R. Org. Biomol. Chem. 2008, 6, 3455. project funded by the FBI designed to develop a database of physical and (30) Lis, E. C., Jr.; Delafuente, D. A.; Lin, Y.; Mocella, C. J.; Todd, M. A.; chemical distinguishing characteristics of polyethylene bags. In January Liu, W.; Sabat, M.; Myers, W. H.; Harman, W. D. Organometallics 2006, Dan joined Dean’s group at the University of Virginia (UVA) and 2006, 25, 5051. studied the ability of {TpW(NO)(PMe3)} to activate pyridine toward (31) Liu, W.; You, F.; Mocella, C. J.; Harman, W. D. J. Am. Chem. Soc. mild additions, the isolation of new piperidine small molecules, and 2006, 128, 1426. the origin of the complexes’ ability to produce hyperdistorted allyls. (32) Harrison, D. P.; Nichols-Nielander, A. C.; Zottig, V. E.; Strausberg, L.; Dan obtained his Ph.D. degree in 2011, and is currently a postdoctoral Salomon, R. J.; Trindle, C. O.; Sabat, M.; Gunnoe, T. B.; Iovan, D. A.; associate with Prof. Thomas J. Meyer at the U.S. DOE’s Center for Myers, W. H.; Harman, W. D. Organometallics 2011, 30, 2587. Solar Fuels at Chapel Hill’s Energy Frontier Research Center at the

(33) McEwen, W. E.; Cobb, R. L. Chem. Rev. 1955, 55, 511. University of North Carolina, where he is studying H2O oxidation and

(34) Harrison, D. P.; Iovan, D. A.; Myers, W. H.; Sabat, M.; Wang, S.; CO2 reduction catalysts for application in renewable energy. Zottig, V. E.; Harman, W. D. J. Am. Chem. Soc. 2011, 133, 18378. W. Dean Harman was born in 1960 in Stanford, California, (35) Arakawa, Ya.; Murakami, T.; Ozawa, F.; Arakawa, Yu.; Yoshifuji, S. and received his B.Sc. degree in 1983 from Stanford University. He Tetrahedron 2003, 59, 7555. remained at “The Farm” to attend graduate school under the guidance (36) Hirama, M.; Kato, Y.; Seki, C.; Nakano, H.; Takeshita, M.; Oshikiri, of Professor Henry Taube. In 1987, he received his Ph.D. degree from N.; Iyoda, M.; Matsuyama, H. Tetrahedron 2010, 66, 7618. Stanford U., and stayed on as a research associate with Taube until 1989, (37) Krow, G. R.; Huang, Q.; Szczepanski, S. W.; Hausheer, F. H.; Carroll, when he joined the faculty at the University of Virginia. In 1997, he P. J. J. Org. Chem. 2007, 72, 3458. was promoted to Full Professor and named the Cavalier Distinguished (38) Marazano, C.; Yannic, S.; Genisson, Y.; Mehmandoust, M.; Das, B. C. Teaching Chair. Dean has been named a Camille and Henry Dreyfus Tetrahedron Lett. 1990, 31, 1995. Teacher–Scholar (1992–1995), an NSF Young Investigator (1993– (39) Sundberg, R. J.; Hamilton, G.; Trindle, C. J. Org. Chem. 1986, 51, 1998), an Alfred P. Sloan Research Fellow (1994–1996), and has 3672. been the recipient of several UVA teaching awards. He is a coauthor (40) Weinstein, B.; Lin, L.-C. C.; Fowler, F. W. J. Org. Chem. 1980, 45, of ca. 150 refereed journal publications that collectively explore the 1657. diverse interactions of electron-rich transition-metal complexes with (41) One specificexample of a highly specialized pyridine cycloaddition is unsaturated organic molecules. Dean currently lives in Earlysville, known and was discovered independently by two groups. The product Virginia, at the edge of the Blue Ridge Mountains with his wife, Lisa, rapidly degrades into aromatic species (see reference 42). and his children, Alorah and Dustin. Need to Attenuate Your Searching for a Structure Activity? Rare Alkyne?

O NH I 2 HC Boc N N N N OH H Br N Br N H H 737011 778923 ADE000066 ADE000951

OH Br Br Br OH I O H3C Aldrich® Chemistry is proud to offer a portfolio of more than N N N Aldrich® Chemistry is proud to offer over 600 alkyne building N N CH OH N TIPS 3 30,000 quality building blocks including the new indole Boc Cl blocks, including a wide array of propargyl alcohols, acetylenic isosteres featured here. Substituted indoles have frequently boron reagents, and halogenated substrates for cross-coupling. ADE00037 CBR01106 ADE000072 ADE001030 been referred to as privileged structures because they are Many of these are terminal alkynes that can participate as capable of binding to multiple receptors with high affinity, substrates in the Sonogashira cross-coupling reaction. I and thus have applications across a wide range of therapeutic O Cl I 1 • Wide array of reactive handles to help incorporate H3C BF3K CH areas. However, researchers still have a need to attenuate or H N amplify the activity of their target compound without altering N the alkyne moiety in your target molecule N N N 730432 728594 its steric bulk; thus, isosteres of the indole ring have proven H TIPS • Many additional new alkynes available from our very valuable to synthetic and medicinal chemists. 708119 ADE000076 partnerships with Adesis and ChemBridge®

CH3 O H3C Si CH3 O O OH HO OCH 3 N O N FNN N N N H H N 701521 ADE001003 O O

H3C CH3 NHN CH3

ADE000794 ADE001261 N N N OH N N O O H C HO 3 CH N Cl Si 3 TMS CH3 CBR00028 ADE001297 N N CH 3 N To view our complete offering For a complete list of ADE001225 ADE000811 of indole isosteres, visit available alkynes, visit Reference: Aldrich.com/indoleiso Horton, D. A. et al. Chem. Rev. 2003, 103, 893 and references therein. Aldrich.com/alkyne

79187 ChemBridge is a registered trademark of ChemBridge Corp. 79051

79187_Indole Isosteres Ad (Acta 45.2).indd 1 7/27/2012 2:49:16 PM Searching for a Rare Alkyne?

O HC Boc N OH H 737011 778923

OH Br OH O H C Aldrich® Chemistry is proud to offer over 600 alkyne building 3 N CH OH blocks, including a wide array of propargyl alcohols, acetylenic 3 boron reagents, and halogenated substrates for cross-coupling. ADE00037 CBR01106 Many of these are terminal alkynes that can participate as substrates in the Sonogashira cross-coupling reaction. I • Wide array of reactive handles to help incorporate H3C BF3K CH the alkyne moiety in your target molecule 730432 728594 • Many additional new alkynes available from our partnerships with Adesis and ChemBridge®

CH3 H3C Si CH3

HO

O FNN N N

O O

H3C CH3 CH3

ADE000794 ADE001261

H C HO 3 CH N Cl Si 3 CH3 N N CH 3 N For a complete list of ADE001225 ADE000811 available alkynes, visit Aldrich.com/alkyne

ChemBridge is a registered trademark of ChemBridge Corp. 79051 P.O. Box 14508 St. Louis, MO 63178 USA

Speed, Convenience, Service

SEARCH

Aldrich® Market Select is a comprehensive solution for identification and procurement of readily available screening compounds and building blocks. The structure database and purchasing website include regularly updated catalog data from over 60 of the most reliable suppliers around the world. SELECT

We make compound and building block procurement easy! • Quote & Order Management • Custom Formatting & Packaging • International Shipping and Compliance • Consolidate Invoices and Supplier Payment PURCHASE • Post-procurement Follow-up Support

Purchase your chemistry with ease at www.AldrichMarketSelect.com

OSY 78634-512540 79236 1082 Page intentially blank Page intentially blank VOL. 45, NO. 3 • 2012

Recent Advances in the Buchwald–Hartwig Amination M ICHI ICA A DR C L TA A

Reaction Enabled by the Application of Sterically

2 8 th 0

6

1

9 2 Demanding Phosphine Ancillary Ligands 1 4 A N 5 Y R Transition-Metal-Mediated Fluorination, N I V E R S A Difluoromethylation, and Trifluoromethylation Want a Cleaner Coupling Substrate?

779601 713457 731560

Aldrich® Chemistry is proud to offer over 1,000 iodinated building blocks for your research. Iodine-containing 729396 756334 728594 substrates are valuable building blocks for a diverse array of synthetic methodologies. They can participate in various cross-coupling reactions for the generation of carbon–carbon, carbon–nitrogen and carbon–oxygen bonds. Iodinated substrates are often preferred to their brominated analogues, because of their cleaner reactions. 778540 728586 728268 They are also handy precursors for organolithium, organozinc and organomagnesium reagents.

739758 755389 ADE000435

751251 ADE000340 716286

707120 ADE000915 ADE001132

For a complete list of available building blocks, visit Aldrich.com/bbacta

79658

79658_Acta 45.3 Ad - Building Blocks Coupling Substrates.indd 1 11/6/2012 1:31:21 PM 57 VOL. 45, NO. 3 • 2012

Aldrich Chemical Co. LLC. “PLEASE BOTHER US.” 6000 N. Teutonia Ave. Milwaukee, WI 53209, USA Dear Fellow Chemists,

To Place Orders Professor Michael Willis of the University of Oxford suggested we offer DABSO, a charge-transfer complex between DABCO® and SO and a convenient Telephone 800-325-3010 (USA) 2 replacement for gaseous SO . It is a bench-stable, colorless, crystalline solid that FAX 800-325-5052 (USA) 2 has found numerous applications in the synthesis of sulfamides, sulfonamides, or 414-438-2199 N-aminosulfonamides, and sulfolenes. Mail P.O. Box 2060 Milwaukee, WI 53201, USA (a) Emmett, E. J.; Richards-Taylor, C. S.; Nguyen, B.; Garcia-Rubia, A.; Hayterb, B. R.; Willis, M. C. Org. Biomol. Chem. 2012, 10, 4007. (b) Ye, S.; Wu, J. Chem. Commun. 2012, 48, 7753. (c) Woolven, Customer & Technical Services H.; González-Rodríguez, C.; Marco, I.; Thompson, A. L.; Willis, M. C. Org. Lett. 2011, 13, 4876. (d) Nguyen, B.; Emmett, E. J.; Willis, M. C. J. Am. Chem. Soc. 2010, 132, 16372. Customer Inquiries 800-325-3010 Technical Service 800-231-8327 DABCO is a registered trademark of Air Products and Chemicals, Inc. SAFC® 800-244-1173 O2S•NN•SO2 Custom Synthesis 800-244-1173 Flavors & Fragrances 800-227-4563 742937742937 International 314-771-5765 742937 1,4-Diazabicyclo[2.2.2]octane bis(sulfur dioxide) adduct (DABSO), 1 g 24-Hour Emergency 314-776-6555 ≥95% (sulfur, elemental analysis) 5 g Website sigma-aldrich.com Email [email protected] We welcome fresh product ideas from you. Do you need a compound that isn’t listed in our catalog? Ask Aldrich! For more than 60 years, your research needs and suggestions have shaped the Aldrich product offering. To submit your suggestion, visit Aldrich.com/pleasebotherus. General Correspondence Editor: Sharbil J. Firsan, Ph.D. P.O. Box 2988, Milwaukee, WI 53201, USA [email protected] John Radke Director of Marketing, Chemistry Subscriptions Request your FREE subscription to the TABLE OF CONTENTS Aldrichimica Acta, through: Recent Advances in the Buchwald–Hartwig Amination Reaction Enabled by the Application Web: Aldrich.com/acta of Sterically Demanding Phosphine Ancillary Ligands...... 59 Rylan J. Lundgren and Mark Stradiotto,* Dalhousie University Email: [email protected] Transition-Metal-Mediated Fluorination, Difluoromethylation, Phone: 800-325-3010 (USA) and Trifluoromethylation...... 67 Zhuang Jin, Gerald B. Hammond,* and Bo Xu,* University of Louisville Mail: Attn: Marketing Services Sigma-Aldrich Corporation ABOUT OUR COVER 3050 Spruce Street St. Louis, MO 63103 Forest of Fontainebleau (oil on canvas, 175.6 × 242.6 cm) was painted in 1834 by the French artist Jean-Baptiste-Camille Corot (1796–1875). Corot was a prolific painter with a talent that ranged widely from mainly landscapes to portraits, nudes, etchings, and other forms. He even dabbled in photography later on in his life. Unlike a number of his famous contemporaries, his rise to prominence in the artistic world was slow: He did not take up painting until his mid-twenties, and it wasn’t until his mid-forties that he began to get the Detail from Forest of Fontainebleau. Photograph recognition he deserved, first from fellow artists, and then from © Board of Trustees, National Gallery of Art, Washington. The entire Aldrichimica Acta archive is also available free of art critics, official France, collectors, and the public. His artistic training consisted mainly of apprenticeships charge at Aldrich.com/acta. with Neoclassicist painters Achille-Etna Michallon and Jean-Victor Bertin and three trips a few years apart to Aldrichimica Acta (ISSN 0002-5100) is a publication of Aldrich. Italy where he honed his landscape painting skills. Corot lived his life as a simple and humble man who was a Aldrich is a member of the Sigma-Aldrich Group. mentor and a philanthropist to many young and struggling artists, and his work is thought to foreshadow the impressionist movement. ©2012 Sigma-Aldrich Co. LLC. All rights reserved. SIGMA, Forest of Fontainebleau is a good example of the genre of painting known as “historical” landscape, SAFC, SIGMA-ALDRICH, ALDRICH, and SUPELCO are trademarks which Corot excelled at, and in which a historical or biblical theme is incorporated into the painting in order of Sigma-Aldrich Co. LLC, registered in the US and other countries. Add Aldrich is a trademark of Sigma-Aldrich Co. LLC. to “elevate” it in the eyes of art critics and gain public attention. In this case, the biblical connection is inferred Aldrich brand products are sold through Sigma-Aldrich, Inc. from several clues* Corot inserted in the painting. Corot’s strong naturalist strain, controlled paint strokes, and Purchaser must determine the suitability of the product(s) tendency to favor brown and black colors are reflected in this unpretentious and simply rendered composition for their particular use. Additional terms and conditions may depicting a rough forest that stands in sharp contrast to the serene young woman. apply. Please see product information on the Sigma-Aldrich This painting is part of the Chester Dale Collection at the National Gallery of Art, Washington, DC. website at www.sigmaaldrich.com and/or on the reverse side * Can you find the clues in the painting? What do the clues reveal about the young woman reading? To find out, of the invoice or packing slip. visit Aldrich.com/acta/aoc453 Dalphos Ligands for Cross-Coupling

A Dalphos ligand contains the bulky di(1-adamantyl)phosphino Pd-Catalyzed Monoarylation of Ammonia, Hydrazine

[P(1-Ad)2] fragment. These chelating N,P ligands are useful for and Acetone Pd-catalyzed C–N and C–C bond formation. The more reactive Mor-Dalphos improves the scope and utility of ammonia coupling at room temperature, and is also effective in the coupling of hydrazine and acetone. • Monoarylation of ammonia, hydrazine and acetone • Air-stable • Good functional group tolerance • Mild conditions

751618 751413 759198 Mor-DalPhos Me-DalPhos New

References: (1) Lundgren, R. J.; Peters, B. D.; Alsabeh, P. G.; Stradiotto, M. Angew. Chem. Int. Ed. 2010, 49, 4071. (2) Lundgren, R. J.; Stradiotto, M. Angew. Chem. Int. Ed. 2010, 49, 8686. (3) Hesp, K. D.; Lundgren, R. J.; Stradiotto, M. J. Am. Chem. Soc. 2011, 133, 5194. (4) Lundgren, R. J.; Sappong-Kumankumah, A.; Stradiotto, M. Chem.—Eur. J. 2010, 16, 1983. (5) Hesp, K. D.; Stradiotto, M. J. Am. Chem. Soc. 2010, 132, 18026.

To learn more about DalPhos ligands, visit Aldrich.com/dalphosacta

79653

79653_Acta 45.3 Ad - DalPhos Ligands.indd 1 11/14/2012 12:30:54 PM 59 VOL. 45, NO. 3 • 2012 Recent Advances in the Buchwald–Hartwig Amination Reaction Enabled by the Application of Sterically Demanding Phosphine Ancillary Ligands

Rylan J. Lundgren and Mark Stradiotto* Department of Chemistry Dalhousie University 6274 Coburg Road P.O. Box 15000 Halifax, NS, Canada B3H 4R2 Email: [email protected]

Dr. Rylan J. Lundgren Prof. Mark Stradiotto

Keywords. amination; catalysis; cross-coupling; ligand design; ligands. However, the subsequent observation that the use of more palladium. structurally complex ancillary ligands can enable more challenging substrate pairings, often with excellent functional group tolerance Abstract. This review highlights a selection of important recent and under milder conditions (e.g., low catalyst loadings, mild reaction developments in the catalysis of the Buchwald–Hartwig amination temperatures), has given rise to the burgeoning field of ancillary ligand reaction. These developments have been enabled through the use of development within the domain of BHA catalysis. Indeed, it is now sterically demanding phosphine ancillary ligands. Our aim is to inform well-established that the use of sterically demanding and electron- the reader of recent methodology advances, and to encourage further rich phosphine ligands, as well as N-heterocyclic carbenes,5 serves development and understanding within the field of ancillary ligand design. to facilitate the formation of low-coordinate Pd(0) species that can

readily engage in otherwise challenging Csp2–X bond oxidative addition Outline reactions (e.g., X = Cl), as well as to encourage, through the alleviation 1. Introduction of steric congestion, C–N bond reductive elimination from requisite

2. Selective Arylation of Challenging Substrates LnPd(aryl)(NR2) intermediates (R = aryl, alkyl, H, etc.) 2.1. Selective Monoarylation of Ammonia The aim of this review is to highlight a selection of important recent 2.2. Selective Monoarylation of Hydrazine developments in the rapidly expanding field of BHA catalysis that have 2.3. Regioselective Arylation of Imidazoles and 1,2,3-Triazoles been enabled through the design and/or implementation of sterically 2.4. Amination of Aryl Mesylates demanding phosphine ancillary ligands. In particular, emphasis will be 3. Chemoselective Arylation of Substrates Featuring Multiple N–H placed on examples from the recent literature in which the novel ligand Functionalities architecture has played a central role in advancing the state of the art for 4. Towards a Universal Catalyst for the Buchwald–Hartwig Amination reaction classes where achieving broad substrate scope, selectivity, and 5. Conclusions and Outlook catalytic efficiency has otherwise proven to be particularly challenging. 6. Acknowledgements The ancillary ligands that will be discussed in this review include the 7. References bisphosphine JosiPhos ligand (L1) utilized by Hartwig’s group, biaryl monophosphine ligands (L2–L6) developed by Buchwald’s group, 1. Introduction the P,N DalPhos ligands (L7 and L8) developed by our group, and the The palladium-catalyzed cross-coupling of (hetero)aryl (pseudo)halides substituted indole ligand L9 disclosed by Kwong’s group (Figure 1). and N–H containing substrates (i.e., Buchwald–Hartwig amination or BHA) has emerged as a highly effective C–N bond-forming protocol 2. Selective Arylation of Challenging Substrates that is employed on both bench-top and industrial scales for the Notwithstanding the tremendous progress that has been made with construction of (hetero)arylamines.1 A prototypical cross-coupling regard to expanding the scope of BHA in terms of both the (hetero)aryl reaction of this type involving 4-chlorotoluene and aniline is depicted (pseudo)halide and amine reaction partners, some transformations have in eq 1. In the time following the initial development of such catalytic until recently proven to be stubbornly problematic. Notable examples of cross-coupling reactions independently by Buchwald’s2 and Hartwig’s3 such challenging transformations include the selective monoarylation groups, significant research effort has been directed toward expanding of ammonia and hydrazine, the regioselective arylation of imidazoles the scope and utility of such methods, including by examining how the and 1,2,3-triazoles, and the use of aryl mesylates as coupling partners. choice of solvent, base, palladium precursor, and, perhaps most notably, In the sections that follow, we discuss in detail the recent advances in the ancillary co-ligand influences the course of the BHA reaction.4 ancillary ligand development that have enabled significant progress in Early studies employed relatively simple triarylphosphine ancillary addressing these reactivity challenges. Recent Advances in the Buchwald–Hartwig Amination Reaction Enabled by the Application 60 of Sterically Demanding Phosphine Ancillary Ligands Rylan J. Lundgren and Mark Stradiotto*

2.1. Selective Monoarylation of Ammonia ligand, CyPF-tBu (L1), which had been developed previously and The direct preparation of nitrogen-containing molecules from commercialized by Solvias for use in asymmetric hydrogenation ammonia—an inexpensive commodity chemical—remains an attractive applications.8 By comparison, it was reported that the use of

goal in modern chemical synthesis. Unfortunately, many metal-catalyzed alternative ancillary ligands including P(tBu)3, XPhos, DPPF, QPhos, chemical transformations, including BHA, that are well-established for IPr, or BINAP did not result in any reaction under the rather forcing other amine classes do not proceed with useful efficiency and selectivity conditions employed (80 psi ammonia at 90 ºC), thereby supporting when employing ammonia as a substrate using commonly employed the view that the rigid, sterically demanding, and electron-rich nature reaction protocols.6 Challenges associated with the use of ammonia of L1 may serve both to enhance catalyst lifetime and to discourage in BHA chemistry include: (i) catalyst deactivation arising from the unwanted polyarylation. A significant improvement to this protocol was

formation of either Werner-type ammine complexes upon displacement published by Vo and Hartwig in 2009, in which the [Pd(P(o-Tol)3)2]–L1 of the ancillary co-ligand, or of amide-bridged polynuclear complexes; precatalyst mixture was employed (eq 2).9 This catalyst system allowed (ii) the slow rate of reductive elimination from sterically unencumbered for the efficient monoarylation of ammonia using aryl bromides,

intermediates of the type LnPd(aryl)(NH2); and (iii) uncontrolled chlorides, iodides, and tosylates, including substrates featuring base- polyarylation because of the competitive nature of the product (hetero)- sensitive groups, without the routine need for high ammonia pressures. aryl amines relative to ammonia in the presence of many commonly In a subsequent report by Klinkenberg and Hartwig, the results of employed BHA catalysts. Despite such challenges, the judicious stoichiometric reactivity studies were disclosed, which revealed that, selection of ancillary ligand has recently enabled the development in ammonia cross-coupling reactions employing Pd–L1, the catalyst 10 of useful BHA protocols that permit the selective monoarylation resting state is an [(L1)Pd(aryl)(NH2)] complex. More recently, our of ammonia. group successfully applied a Pd–L1 catalyst system to cascade ammonia The selective monoarylation of ammonia by use of BHA protocols arylation–alkyne hydroamination processes employing functionalized was first disclosed in 2006 by Shen and Hartwig,7 who employed the 2-bromoarylacetylenes to afford NH-indoles (eq 3).11 Notably, this

palladium(II) precatalyst [(CyPF-tBu)PdCl2] featuring the JosiPhos process represents the first reported synthesis of indoles directly from ammonia through metal-catalyzed cross-coupling. The application of biaryl monophosphine ligands in cross-coupling reactions employing ammonia as a substrate has been examined by Pd (cat) H Buchwald’s group.12 Following a brief survey of biaryl monophosphine Cl H2N ligand (cat) N + ligands by employing the coupling of ammonia (5 equiv) with base Me Me chlorobenzene as the test reaction and with [Pd2(dba)3] as the palladium source (dba = dibenzylideneacetone; 2 mol % Pd and 5 mol % ligand; eq 1 (Ref. 1) 80 ºC), tBuDavePhos (L2) was found to exhibit a suitable reactivity profile in terms of substrate conversion and monoarylation selectivity.12a

More recently, this [Pd2(dba)3]–L2 precatalyst system was employed successfully by Buchwald and Tsvelikhovsky in BHA reactions of ammonia, whereby the derived aniline intermediate undergoes an Me intramolecular condensation reaction to afford dibenzodiazepines and P(t-Bu)2 Me Me related biologically active heterocyclic structures (eq 4).12b Given the Me P(t-Bu)2 PCy Fe 2 Me2N established propensity of biaryl monophosphine ligands, including Me P( -Bu) t 2 13 Ar L2, to coordinate to either Pd(0) or Pd(II) via phosphorus and one or more carbon atoms of the lower flanking arene ring,1b the specific role JosiPhos; CyPF-tBu (L1) tBuDavePhos (L2) Me4tBuXPhos (L3) Hartwig Buchwald Buchwald of the presumably uncoordinated dimethylamino group present in L2 in

(OMe) enabling such reactivity has yet to be established unequivocally. (Me) Me In a pair of publications, Beller and co-workers14 reported that MeO Me OMe PCy2 imidazole-derived monophosphine ligands are capable of supporting i-PrO Oi-Pr active complexes for the monoarylation of ammonia. Unfortunately, Me P(t-Bu)2 MeO PCy2 Ar Ar the scope of the reaction with aryl chlorides is somewhat limited, Me3(OMe)tBuXPhos (L4) BrettPhos (L5) RuPhos (L6) high reaction temperatures are often employed (≥120 ºC), and in some Buchwald Buchwald Buchwald cases high pressures of inert gases (10 bar N2) are needed to obtain Ar = 2,4,6-(i-Pr)3C6H2 satisfactory results.

PCy2 In 2010, our group developed structurally simple phenylene- P(1-Ad) 2 NMe bridged P,N ligands that have proven to be particularly useful in metal- N catalyzed C–C and C–N bond-forming reactions, including BHA.15–17 P(1-Ad)2 NMe2 O We initially disclosed the ancillary ligand Me-DalPhos (L7), which was Me-DalPhos (L7) Mor-DalPhos (L8) CM-Phos (L9) shown to be effective for the cross-coupling of a remarkably wide range Stradiotto Stradiotto Kwong of amines with structurally diverse (hetero)aryl chlorides.15 While L7 provided high conversions and good selectivities in the cross-coupling of ammonia with ortho-substituted aryl chlorides, the use of more challenging aryl chloride substrates resulted in poor monoarylation Figure 1. Sterically Demanding and Electron-Rich Phosphine Ligands selectivity.15 In the quest to circumvent such reactivity issues, further Featured in this Review. modifications of the DalPhos ligand structure were explored. These studies identified Mor-DalPhos (L8) as a highly active and selective 61 VOL. 45, NO. 3 • 2012 ligand for the cross-coupling of ammonia with aryl chlorides and tosylates (eq 5).17 Reactions employing precatalysts based on L8 proceeded with high monoarylation selectivity at generally low catalyst [Pd(P(o-Tol)3)2] (0.1–2.0 mol %) loadings and under mild reaction conditions without the need for high X L1 (0.1–2.0 mol %) NH2 pressures of ammonia. The first examples of room-temperature BHA R + NH3 R 17 E NaOt-Bu, 1,4-dioxane E chemistry involving ammonia were also included in this report. When 50–100 oC, 4–24 h X = I, Br, Cl, OTs 5 equiv or 44–99% L8 was used, the substrate scope was found to encompass a diversity E = N, CH 200 psi >35 examples of electron-rich and electron-poor (hetero)aryl chlorides and tosylates, and excellent chemoselectivity for ammonia arylation was observed in eq 2 (Ref. 9) the presence of potentially competitive aminoaryl chloride substrates.

2.2. Selective Monoarylation of Hydrazine

Aryl hydrazines are key intermediates in the synthesis of a number of [Pd(cinnamyl)Cl]2 important nitrogen heterocycles including, most notably, indoles via R' (1.25 mol %) L1 (2.5 mol %) the Fischer indole synthesis. The most commonly employed protocol R + NH3 R R' Br 3 equiv KOt-Bu (3 equiv) N for the preparation of aryl hydrazines relies on the stoichiometric H 1,4-dioxane oxidation of anilines to their corresponding diazonium salts followed 31–89% 90 oC, 3–24 h by reduction, which suffers from limited functional-group tolerance 16 examples and poor atom economy. While hydrazine can, in select cases, react eq 3 (Ref. 11) directly with electron-deficient haloarenes in nucleophilic aromatic substitution reactions, the development of effective BHA protocols for the selective monoarylation of hydrazine represents a novel entry point for the preparation of aryl hydrazines. However, the use of hydrazine O R R Cl as a cross-coupling partner in BHA chemistry presents a number of H [Pd2(dba)3] (1.5 mol %) N N L2 (5.0 mol %) potential problems, including: (i) unwanted reduction of the (hetero)- 1 2 1 2 R R + NH3 R R NaO -Bu or Cs CO N aryl (pseudo)halide substrate as well as the palladium precatalyst; (ii) t 2 3 H 5–7 equiv 1,4-dioxane, 4 Å MS metal-mediated N–N bond cleavage resulting in undesired aniline dibenzodiazepines 85–120 oC, 2–24 h byproducts; and (iii) the inherent susceptibility of the aryl hydrazine 43–93%; 9 examples product to undergo subsequent C–N cross-coupling leading to Other Structural Analogues Prepared: polyarylated products. R O O The selective monoarylation of hydrazine by use of BHA protocols N HN HN was first reported by us in 2010.18 In the course of this investigation, R1 R2 R1 R2 O N O a broad range of ancillary ligands were surveyed, the vast majority H of which afforded either poor conversion of the test substrate dibenzooxazepines dibenzodiazepinones dibenzooxazepinone 4-phenylchlorobenzene, or exclusive reduction of this substrate to 69–94%; 6 examples 73–93%; 5 examples 82% biphenyl by way of metal-catalyzed hydrodehalogenation. However, L8, as well as L1 and L7, gave rise to the desired aryl hydrazine as eq 4 (Ref. 12b) the major product. Optimization of the reaction conditions using L8 afforded a catalyst system that proved successful in transforming a range of aryl chlorides and tosylates into the corresponding aryl hydrazines with generally excellent monoarylation selectivity (eq [Pd(cinnamyl)Cl] 18 2 6). The synthesis of indazoles via BHA–condensation reactions (0.15–3.0 mol %) X L8 (0.30–6.0 mol %) NH2 employing 2-chlorobenzaldehydes and hydrazine was also disclosed R + NH3 R in this report. Notwithstanding the significance of this important E NaOt-Bu, 1,4-dioxane E 25–110 oC, 2–48 h reactivity breakthrough, some important limitations exist with X = Cl, OTs 3–4 equiv 52–99% E = N, CH 35 examples regard to this L8-based catalyst system; notably, a reduced yield (50%) of the target aryl hydrazine was obtained for the electron- poor substrate 4-trifluoromethylchlorobenzene due to competitive eq 5 (Ref. 17) hydrodehalogenation, while poor conversion (27% yield) was observed for the electron-rich substrate 4-chloroanisole.18 The successful application of the [Pd(cinnamyl)Cl]2–L1 precatalyst mixture in the BHA of hydrazine with 2-alkynylbromoarenes to afford N-aminoindoles and [Pd(cinnamyl)Cl]2 (1.5–5.0 mol %) 11 X NHNH indazoles was reported subsequently. N2H4•H2O L8 (2.3–7.5 mol %) 2 R + or R E N2H4•HCl NaOt-Bu E 2.3. Regioselective Arylation of Imidazoles and 1,4-dioxane or PhMe X = Cl, OTs 2 equiv 27–97% o 1,2,3-Triazoles E = N, CH 50–110 C, 0.33–2.0 h 32 examples The synthesis of N-arylated heterocycles is of considerable interest, given the widespread application of such compounds in diverse fields ranging from materials science to medicinal chemistry. While eq 6 (Ref. 18) a number of synthetic routes to N-arylated heterocycles have been Recent Advances in the Buchwald–Hartwig Amination Reaction Enabled by the Application 62 of Sterically Demanding Phosphine Ancillary Ligands Rylan J. Lundgren and Mark Stradiotto*

established, the ability to prepare these target compounds via arylation of imidazole,20 Buchwald and co-workers developed a completely N1-

chemistry (including SNAr or copper-mediated processes) in a highly selective protocol for the arylation of unsymmetrical 4-substituted regioselective manner and with a useful substrate scope has proven to imidazoles (eq 7).19b Activation of the catalyst derived from a mixture

be particularly challenging. However, recent reports by Buchwald and of [Pd2(dba)3] and L3 prior to introduction of the imidazole substrate co-workers19 demonstrate the utility of BHA chemistry in addressing proved crucial in circumventing the inhibitory effect of the heterocyclic such reactivity challenges through the judicious selection of ancillary substrate during in situ catalyst formation. A variety of (hetero)aryl co-ligand and the implementation of appropriate pre-catalyst activation bromides, chlorides, and triflates were employed successfully in the procedures. regioselective arylation of 4-methylimidazole, 4-phenylimidazole, The tautomeric nature of unsymmetrical 1H-imidazoles presents N-acetylhistamine, and 4-cyanomethylimidazole (0.5–2.5 mol % a challenge with respect to obtaining high levels of regioselectivity each of Pd and L3, 120 ºC). The utility of their catalytic protocol was in the arylation of such substrates. Given this, and the difficulty further demonstrated through the regiocontrolled synthesis of the associated with separating the product N1- and N3-aryl regioisomers, glycine transporter inhibitor GSK2137305 (GlaxoSmithKline®) and the the establishment of efficient and regioselective N-arylation protocols anticancer drug nilotinib (TASIGNA®).19b would be of considerable synthetic utility. Building on their earlier work Buchwald and co-workers were also successful in applying the 2 employing Me4tBuXPhos (L3) in the palladium-catalyzed N-arylation [Pd2(dba)3]–L3 catalyst system toward the highly N -selective arylation of 4,5-unsubstituted and 4-unsubstituted 1,2,3-triazoles with (hetero)- aryl bromides, chlorides, and triflates—a useful class of transformations

that to date has not been achieved by use of complementary SNAr or copper-mediated protocols (eq 8).19a In keeping with their examination [Pd2(dba)3] N of the arylation of unsymmetrical 4-substituted imidazoles (vide supra), R' (0.25–1.25 mol %) R' X N N L3 (0.5–2.5 mol %) Buchwald and co-workers noted that preactivation of the [Pd2(dba)3]– R + R L3 catalyst mixture in the absence of substrates and base was required E N K3PO4 (2.0 equiv) E H 1,4-dioxane–PhMe (1:5) in order to achieve optimal catalytic performance. Furthermore, studies X = Cl, Br, OTf 77–96% o E = N, CH 120 C, 5 h 21 examples of the influence of varying the ligand structure confirmed that the substitution pattern on both arene rings in L3 contributes importantly to the desirable performance of the catalyst system.19a eq 7 (Ref. 19b) One drawback of L3 is that it is prepared from the rather costly reagent 1,2,3,4-tetramethylbenzene. In response, Buchwald and co- workers21 have developed the synthesis of the structurally related

ligand Me3(OMe)tBuXPhos ligand (L4), which can be prepared in 59%

[Pd (dba) ] overall yield as a nearly 1:1 mixture of regioisomers from the less costly 2 3 N R' (0.25–1.0 mol %) R' 2,3,6-trimethylphenol. Comparative catalytic studies established that X L3 (0.5–1.8 mol %) N N R + R L4 can serve as a surrogate for L3 in the aforementioned regioselective N N E N K3PO4 (1.7 equiv) E arylation of imidazoles and 1,2,3-triazoles. H o PhMe, 120 C, 5 h 2 1 X = Cl, Br, OTf 46–94%, N :N ≥ 95:5 E = N, CH 22 examples 2.4. Amination of Aryl Mesylates Aryl mesylates are attractive electrophiles for palladium-catalyzed eq 8 (Ref. 19a) cross-coupling reactions owing to their ease of synthesis from readily available phenols, and the associated waste reduction in comparison to higher-molecular-weight oxygen-based leaving groups, such as tosylates or triflates.22 However, the sluggish reactivity of aryl mesylates prevented the use of such substrates in BHA until the groups of Kwong23 24 NH2 (1.0 mol %) and Buchwald independently detailed the first successful protocols in Pd L5 Cl R' 2008 (Scheme 1). It was demonstrated that catalysts featuring either the N 23 24 L5 (1.0 mol %) R'' indole-containing CM-Phos (L9) or BrettPhos (L5) could mediate R the amination of a range of aryl mesylates. Notably, Pd(OAc)2–L9 K2CO3, t-BuOH o 110 C, 16 h 82–99% mixtures promoted C–N bond-forming reactions that employ t-BuOH OMs 6 examples or water as solvent, as well as those carried out in the absence of solvent. H R + N A current limitation of such methodology is the modest substrate scope R' R'' with respect to the amine component: in the case of L5, only primary R' Pd(OAc) (0.5–4.0 mol %) anilines were coupled; and in the case of L9, reactions were generally 2 N L9 (0.5–4.0 mol %) R'' R limited to sterically demanding primary aryl amines or secondary K2CO3, t-BuOH amines. o 110 C, 4–24 h 75–98% or water or neat 19 unique examples 3. Chemoselective Arylation of Substrates Featuring Multiple N–H Functionalities Despite the significant attention that has been given to the application Scheme 1. The Amination of Aryl Mesylates Employing Catalysts Based on of BHA protocols in organic synthesis, reports documenting L5 or L9. (Ref. 23,24) chemoselective variants of such reactions involving the preferential arylation of one amine fragment in the presence of multiple, competitive, 63 VOL. 45, NO. 3 • 2012

[Pd(cinnamyl)Cl]2 (0.5–1.0 mol %) Cl L8 (2.0–4.0 mol %) NR NHR'' R + R'HN NHR'' R E NaOt-Bu, E = N, CH 1,4-dioxane or PhMe 50–97% 65–110 oC, 12–48 h 62 examples

Selected Examples

NH2

NH HN NNH HN H N

Me Me 69% 81% 79% Me

R H Me Me N H H N N N N N H H Me 90% 60% R = Me, 97%; R = OMe, 91% R = CF3, 85%; R = 2-propenyl, 94%

N N H N H Me H N N N N N N N H H H

93% 89% 94%

eq 9 (Ref. 28) and chemically distinct N–H reactive functional groups are few and reported thus far in the literature.28 Interestingly, while unhindered include contributions from the groups of Beletskaya,25 Rouden,26 and nucleophilic amine reaction partners are preferred substrates when 20,24,27 Buchwald, among others. What is apparent from these reports is employing [Pd(cinnamyl)Cl]2–L8 mixtures, this catalyst system that a number of important drawbacks exist, including: (i) a limited has also proven useful in the chemoselective arylation of a series of demonstrated substrate scope for a given Pd–L catalyst system; (ii) the alternative amine functionalities (e.g., linear and a-branched primary use of substrates where the statistical bias of amine functional groups alkylamines, imines, primary hydrazones, N,N-dialkylhydrazines, can be viewed as contributing directly to the observed product ratio; (iii) substituted anilines, and piperidine), while tolerating the presence low (<50%) isolated yields of isomeric monoarylation and diarylation of a range of potential competitor amine fragments as well as product mixtures; and/or (iv) the incompatability of synthetically useful varied substitution within the (hetero)aryl chloride reaction partner. (hetero)aryl chloride substrates. In this regard, the identification of Comparative reactivity studies involving the para isomer of L8 effective BHA catalysts for which predictive chemoselectivity models confirmed that the ortho disposition of phosphorus and nitrogen donors can be developed, and the demonstrated application of such reactivity is the key to achieving the distinct chemoselectivity behavior that is models with synthetically useful scope, remain important goals in the observed when employing L8.28 quest to expand the utility of BHA chemistry. Indeed, the establishment of reliable chemoselective transformations of this type would enable 4. Towards a Universal Catalyst for the Buchwald–Hartwig BHA protocols to be applied more broadly toward the arylation of Amination structurally complex substrates featuring multiple N–H functionalities, While advances in ligand design for use in BHA chemistry have given such as those commonly encountered in the synthesis of biologically rise to several distinct classes of catalysts that offer state-of-the-art active target compounds, thereby circumventing the need for wasteful performance for selected substrates, such “task-specific” ligands and sometimes challenging nitrogen protection and deprotection steps. often fail when alternative amine classes are employed, or require Encouraged by our finding of the distinct preference displayed by the substantially higher Pd–ligand loadings to achieve reasonable yields

[Pd(cinnamyl)Cl]2–L8 catalyst system for the selective monoarylation of the target compound. This scenario is particularly problematic for of ammonia or hydrazine when employing aryl chlorides bearing practitioners of BHA in both academia and industry, who are faced competitor primary amine or secondary amine functionalities,17,18 with the dilemma of choosing an appropriate catalyst system for our group has recently applied this catalyst system to chemoselective their particular application.29 Ideally, the identification of a single BHA reactions of the type outlined above (eq 9).28 Competition studies “universal” catalyst system that can couple the broad spectrum of confirmed the preference of the [Pd(cinnamyl)Cl]2–L8 catalyst system potential amine partners with (hetero)aryl (pseudo)halides at modest for unhindered nucleophilic amine reaction partners, which has enabled palladium loadings would serve to advance the utility of BHA protocols a range of structurally diverse di-, tri-, and tetraamines to be arylated in chemical synthesis. While such a “one-catalyst-fits-all” platform has in a chemoselective fashion based on this reactivity model. Indeed, yet to be identified, some broadly useful catalyst systems that meet this study represents the most extensive compilation of such reactivity some of the above-mentioned criteria have been reported. Recent Advances in the Buchwald–Hartwig Amination Reaction Enabled by the Application 64 of Sterically Demanding Phosphine Ancillary Ligands Rylan J. Lundgren and Mark Stradiotto*

In 2010, our group reported on the broad applicability of Me-DalPhos for primary amine monoarylation (ligated by L5) and secondary amine (L7) in BHA catalysis.15 The cross-coupling of rather challenging arylation (ligated by L6). Notwithstanding the useful scope exhibited (hetero)aryl chlorides to a diverse range of amines and related substrates by this unusual catalyst system, substrate limitations similar to those was achieved, including large and small primary alkyl- and arylamines, noted for the DalPhos system (vide supra) were observed; namely, cyclic and acyclic secondary amines, N–H imines, hydrazones, lithium imidazoles, benzimidazoles, pyrazoles, and secondary cyclic amides amide, and ammonia. In many cases, the reactions can be performed were not accommodated. Furthermore, unlike catalysts based on the at low (0.5–0.02 mol %) palladium loadings with excellent functional- DalPhos ligands L7 or L8,15,17 the Pd–L5–L6 precatalyst mixture proved group tolerance and chemoselectivity. Subsequent investigations ineffective for the monoarylation of ammonia. More recently, the broad involving Mor-DalPhos L8 confirm that this ligand variant also offers utility of BHA catalysts based either on L5 or L6 in the arylation of a similarly broad substrate scope in BHA.28 However, BHA catalysts primary or secondary amines, respectively, has been demonstrated.31 based on L7 or L8 do have limitations: weakly nucleophilic substrates including indoles, sulfonamides, and Boc-hydroxylamines each proved 5. Conclusions and Outlook resistant to coupling under standard conditions. The development and application of sterically demanding phosphine An alternative approach to the development of more comprehensive ancillary ligands continues to advance the state of the art in BHA catalysts for BHA has recently been disclosed by Fors and Buchwald catalysis. As evidenced by the selected examples featured in this (Scheme 2),30 who employed a catalyst system based on a combination review, recently developed ligands of this type have enabled the of the biaryl monophosphine ligands BrettPhos (L5) and RuPhos (L6). establishment of catalysts that accommodate previously challenging The choice of these two ligands is based on the observation that catalysts substrates in a chemoselective fashion, with broad substrate scope featuring L5 are known to function effectively for the monoarylation of and excellent functional-group tolerance. Moreover, from a practical primary amines, but perform poorly in the arylation of secondary amine perspective, most of the ancillary ligands featured in this review are substrates; the converse is true for L6. The result is a complementary both air-stable and commercially available. While it is evident that some catalyst system that not only exhibits the best reactivity properties ligand structural motifs appear to be privileged, it is noteworthy that no associated with each of L5 and L6, but that also enables high-yielding single class has proven superior in all BHA applications. Furthermore, transformations that are otherwise inefficient for either of the individual within a ligand class, even subtle structural changes have been shown catalyst systems. Using the mixed Pd–L5–L6 precatalyst mixture, to profoundly influence the performance of the derived BHA catalyst. anilines, large and small primary alkyl amines, secondary amines, and In this regard, a more fundamental appreciation of the means by which amides were coupled with (hetero)aryl chlorides, bromides, and iodides ancillary ligand structure can direct the course of BHA processes is in high yields and with Pd loadings ranging from 0.005–1.0 mol %. needed in order to guide the rational development of new catalysts to Furthermore, by exploiting the more facile reactivity of aryl bromides address the important reactivity goals that remain in the field. relative to aryl chlorides, Fors and Buchwald were able to apply the Pd–L5–L6 pre-catalyst mixture toward the high-yielding, one-pot 6. Acknowledgements synthesis of the hole-transport agent, TPD (see Scheme 2).30 On the We thank the NSERC of Canada (including Discovery, CRD, CHRP, basis of elegant stoichiometric reactivity studies, the authors proposed Engage, and I2I Grants for M. S., as well as both a Postgraduate that, under the catalytic conditions employed (≥100 ºC), reversible Scholarship and a Postdoctoral Fellowship for R. J. L.), the Canada exchange of L5 for L6 on palladium is facile, and can occur involving Foundation for Innovation, the Killam Trusts, the Province of Nova both Pd(0) and Pd(II) reactive intermediates. Such processes enable Scotia (Springboard, Innovacorp, and the Nova Scotia Research and the shuttling of palladium between two independent catalytic cycles Innovation Trust), and Dalhousie University for their generous financial support of this work.

7. References L5 complex (0.005–1.0 mol %) (1) (a) Hartwig, J. F. Acc. Chem. Res. 2008, 41, 1534. (b) Surry, D. S.; X NR'R" L6 (0.005–1.0 mol %) Buchwald, S. L. Angew. Chem., Int. Ed. 2008, 47, 6338. (c) Busacca, R + HNR'R" R E NaOt-Bu, 1,4-dioxane E C. A.; Fandrick, D. R.; Song, J. J.; Senanayake, C. H. Adv. Synth. or X = Cl, Br, I 80–99% Catal. 2011, 353, 1825. E = N, CH Cs2CO3, t-BuOH 17 examples 80–110 oC, 16–24 h (2) Guram, A. S.; Rennels, R. A.; Buchwald, S. L. Angew. Chem., Int. Ed. Engl. 1995, 34, 1348. (3) Louie, J.; Hartwig, J. F. Tetrahedron Lett. 1995, 36, 3609. (4-H2NC6H4)2 Ph -Tol L5 complex N o (4) Surry, D. S.; Buchwald, S. L. Chem. Sci. 2011, 2, 27. (0.2 mol %) Br Cl L6 (0.2 mol %) (5) (a) Fortman, G. C.; Nolan, S. P. Chem. Soc. Rev. 2011, 40, 5151. (b) + Valente, C.; Çalimsiz, S.; Hoi, K. H.; Mallik, D.; Sayah, M.; Organ, M. NaOt-Bu, 1,4-dioxane o Me 110 C, 24 h G. Angew. Chem., Int. Ed. 2012, 51, 3314. (6) (a) Aubin, Y.; Fischmeister, C.; Thomas, C. M.; Renaud, J.-L. Chem. NH2 Soc. Rev. 2010, 39, 4130. (b) Van der Vlugt, J. I. Chem. Soc. Rev. 2010, Pd N L5 Cl Ph o-Tol 39, 2302. (c) Klinkenberg, J. L.; Hartwig, J. F. Angew. Chem., Int. Ed. L5 complex TPD, 98% 2011, 50, 86. (7) Shen, Q.; Hartwig, J. F. J. Am. Chem. Soc. 2006, 128, 10028. (8) Blaser, H.-U.; Brieden, W.; Pugin, B.; Spindler, F.; Studer, M.; Togni, Scheme 2. A Multiple-Ligand BHA Catalyst System Employing L5 and L6. A. Top. Catal. 2002, 19, 3. (Ref. 30) (9) Vo, G. D.; Hartwig, J. F. J. Am. Chem. Soc. 2009, 131, 11049. (10) Klinkenberg, J. L.; Hartwig, J. F. J. Am. Chem. Soc. 2010, 132, 11830. 65 VOL. 45, NO. 3 • 2012

(11) Alsabeh, P. G.; Lundgren, R. J.; Longobardi, L. E.; Stradiotto, M. McDermott, S. M.; Buchwald, S. L. Org. Lett. 2010, 12, 4438. Chem. Commun. 2011, 47, 6936. (28) Tardiff, B. J.; McDonald, R.; Ferguson, M. J.; Stradiotto, M. J. Org. (12) (a) Surry, D. S.; Buchwald, S. L. J. Am. Chem. Soc. 2007, 129, 10354. (b) Chem. 2012, 77, 1056. Tsvelikhovsky, D.; Buchwald, S. L. J. Am. Chem. Soc. 2011, 133, 14228. (29) (a) Tasler, S.; Mies, J.; Lang, M. Adv. Synth. Catal. 2007, 349, 2286. (13) Christmann, U.; Pantazis, D. A.; Benet-Buchholz, J.; McGrady, J. E.; (b) Cooper, T. W. J.; Campbell, I. B.; Macdonald, S. J. F. Angew. Maseras, F.; Vilar, R. J. Am. Chem. Soc. 2006, 128, 6376. Chem., Int. Ed. 2010, 49, 8082. (14) (a) Schulz, T.; Torborg, C.; Enthaler, S.; Schäffner, B.; Dumrath, A.; (30) Fors, B. P.; Buchwald, S. L. J. Am. Chem. Soc. 2010, 132, 15914. Spannenberg, A.; Neumann, H.; Börner, A.; Beller, M. Chem.—Eur. J. (31) Maiti, D.; Fors, B. P.; Henderson, J. L.; Nakamura, Y.; Buchwald, S. L. 2009, 15, 4528. (b) Dumrath, A.; Lübbe, C.; Neumann, H.; Jackstell, Chem. Sci. 2011, 2, 57. R.; Beller, M. Chem.—Eur. J. 2011, 17, 9599. (15) Lundgren, R. J.; Sappong-Kumankumah, A.; Stradiotto, M. Chem.— Trademarks. GlaxoSmithKline® (Glaxo Group Limited); TASIGNA® Eur. J. 2010, 16, 1983. (Novartis AG). (16) Lundgren, R. J.; Hesp, K. D.; Stradiotto, M. Synlett 2011, 2443. (17) Lundgren, R. J.; Peters, B. D.; Alsabeh, P. G.; Stradiotto, M. Angew. About the Authors Chem., Int. Ed. 2010, 49, 4071. Rylan J. Lundgren received his B.Sc. degree (with honors) in Chemistry (18) Lundgren, R. J.; Stradiotto, M. Angew. Chem., Int. Ed. 2010, 49, 8686. in 2006 from the University of Manitoba, where he performed research (19) (a) Ueda, S.; Su, M.; Buchwald, S. L. Angew. Chem., Int. Ed. 2011, 50, with Prof. Mario Bieringer. He also conducted research in 2005 with 8944. (b) Ueda, S.; Su, M.; Buchwald, S. L. J. Am. Chem. Soc. 2012, Prof. Deryn Fogg at the University of Ottawa. In 2010, he earned his 134, 700. Ph.D. degree at Dalhousie University working under the supervision of (20) Anderson, K. W.; Tundel, R. E.; Ikawa, T.; Altman, R. A.; Buchwald, Prof. Mark Stradiotto. He is currently an NSERC Postdoctoral Fellow S. L. Angew. Chem., Int. Ed. 2006, 45, 6523. at the California Institute of Technology working with Prof. Gregory (21) Ueda, S.; Ali, S.; Fors, B. P.; Buchwald, S. L. J. Org. Chem. 2012, 77, C. Fu. 2543. Mark Stradiotto received his B.Sc. degree (with honors) in Applied (22) So, C. M.; Kwong, F. Y. Chem. Soc. Rev. 2011, 40, 4963. Chemistry (1995) and his Ph.D. degree in Organometallic Chemistry (23) So, C. M.; Zhou, Z.; Lau, C. P.; Kwong, F. Y. Angew. Chem., Int. Ed. (1999) from McMaster University, the latter under the supervision 2008, 47, 6402. of Professors Michael A. Brook and Michael J. McGlinchey. After (24) Fors, B. P.; Watson, D. A.; Biscoe, M. R.; Buchwald, S. L. J. Am. conducting research as an NSERC Postdoctoral Fellow at the Chem. Soc. 2008, 130, 13552. University of California, Berkeley, in the research group of Prof. T. (25) Beletskaya, I. P.; Bessmertnykh, A. G.; Averin, A. D.; Denat, F.; Don Tilley (1999–2001), Mark moved to the Department of Chemistry Guilard, R. Eur. J. Org. Chem. 2005, 261. at Dalhousie University, where he now holds the rank of Professor with (26) Cabello-Sanchez, N.; Jean, L.; Maddaluno, J.; Lasne, M.-C.; Rouden, tenure. Prof. Stradiotto is a member of the editorial advisory board of J. J. Org. Chem. 2007, 72, 2030. Organometallics, and has been named a Synlett/Synthesis Promising (27) (a) Huang, X.; Anderson, K. W.; Zim, D.; Jiang, L.; Klapars, A.; Young Professor Journal Awardee. Most recently, he was awarded the Buchwald, S. L. J. Am. Chem. Soc. 2003, 125, 6653. (b) Henderson, Canadian Society for Chemistry 2012 Strem Chemicals Award for Pure J. L.; Buchwald, S. L. Org. Lett. 2010, 12, 4442. (c) Henderson, J. L.; or Applied Inorganic Chemistry.

For High Versatility and Selectivity in Asymmetric Catalysis

Ruthenium-PNNP Catalyst

NN N Cl N Promotes several enantioselective Ru PP P Cl P • CH2Cl2 Ph Ph transformations such as fluorination 2 2 Ph Ph Ph Ph and [2+2] and [4+2] cycloadditions. 779040 742104 PNNP [RuCl (PNNP)] Add Aldrich™ to • Chiral tetradentate ligand 2 your research Air-, moisture- and heat-stable program. • References: (1) Althaus, M.; Becker, C.; Togni, A.; Mezzetti, A. • Highly efficient Organometallics 2007, 26, 5902. (2) Schotes, C.; Mezzetti, Aldrich.com/ A. Angew. Chem. Int. Ed. 2011, 50, 3072. (3) Schotes, C.; catalysisacta • Excellent selectivity Mezzetti, A. J. Am. Chem. Soc. 2010, 132, 3652.

79652 Looking for a Fluorinating Reagent?

439479 716219 719439

Aldrich® Chemistry is proud to offer customers a growing portfolio of fluorination, difluoromethylation and 719447 734039 738115; 378216 trifluoromethylation reagents to suit their every fluorination need.

Aldrich fluorination reagents: • Are compatible with many functional groups • Afford high product yields under mild reaction conditions 738123 730416 494119; 94327; 94324 • Allow access to several fluorinated motifs in a highly chemoselective manner From milligram to bulk quantities, your research will move forward faster when you add trustworthy products, services and information from Aldrich Chemistry. L510084 L510106 752924

754218; 696641 L510009 483869

483877 SFL00001

Add Aldrich™ to your research program. Aldrich.com/fluoroacta

79581

79581_Acta 45.3 Ad - Fluorination.indd 1 11/15/2012 9:16:26 AM 67 VOL. 45, NO. 3 • 2012 Transition-Metal-Mediated Fluorination, Difluoromethylation, and Trifluoromethylation

Zhuang Jin, Gerald B. Hammond,* and Bo Xu* Department of Chemistry University of Louisville Louisville, Kentucky 40292, USA Email: [email protected]

Dr. Zhuang Jin Prof. Gerald B. Hammond Dr. Bo Xu

Keywords. transition metal; catalysis; fluorination; difluoro- 3.1.2. Trifluoromethylation of Nucleophiles through methylation; trifluoromethylation. Oxidative Coupling 3.1.3. Other Copper-Mediated Trifluoromethylations Abstract. Introducing fluorine into organic molecules is a proven 3.2. Palladium-Mediated Trifluoromethylation successful strategy in designing pharmaceuticals with improved 3.2.1. Pd-Catalyzed Trifluoromethylation of Aryl and Alkenyl therapeutic effects, but has remained a long-standing challenge for Halides via a Pd(0)–Pd(II) Cycle synthetic organic chemists. Fortunately, newly developed transition- 3.2.2. Pd-Catalyzed Trifluoromethylation via a Pd(II)–Pd(IV) metal-catalyzed or mediated fluorination, difluoromethylation, and Cycle trifluoromethylation reactions are addressing this challenge and 3.3. Trifluoromethylation Mediated by Other Metals facilitating the synthesis of fluorinated compounds. In this review, we 3.4. Trifluoromethylation Involving Free Radicals have classified and organized the recent developments in fluorination, 4. Transition-Metal-Promoted Difluoromethylation difluoromethylation, and trifluoromethylation reactions based on the 4.1. Difluoromethylation through an RCF2Cu Intermediate metal catalyst and the underlying mechanism of the C–F, C–CF2, and 4.2. Difluoromethylation via Radical Species

C–CF3 bond formation. We also comment on the limitations of these 5. Summary and Outlook methods. 6. Acknowledgements 7. References Outline 1. Introduction 1. Introduction 2. Transition-Metal-Promoted Fluorination Extensive research has established that introducing fluorine into a 2.1. Pd-Mediated Fluorination target molecule can effectively modify its physicochemical properties.1 2.1.1. Fluorinations via Reductive Elimination from R–Pd–F As a result, up to 20% of pharmaceuticals2 and approximately 30% 2.1.1.1. Reductive Elimination from R–Pd(II)–F of agrochemicals2b contain fluorine and/or the trifluoromethyl group. 2.1.1.2. Reductive Elimination from R–Pd(IV)–F Additionally, fluorine-18 18( F) is regarded as one of the most clinically 2.1.2. Pd-Catalyzed Fluorination of Allyl Halides and Allyl important radioisotopes for Positron Emission Tomography (PET).3 Esters Nevertheless, over a century after its discovery, the introduction 2.1.3. Pd-Catalyzed Fluorination via Oxidative Addition to of fluorine into organic molecules continues to be a long-standing C–C Unsaturated Bonds challenge for synthetic organic chemists. 2.2. Gold-Catalyzed Fluorination In general, fluorinated compounds can be prepared through the 2.2.1. Using a Nucleophilic Fluorine Source use of molecular-building-block methods or of fluorination reagents.1g 2.2.2. Using an Electrophilic Fluorine Source The former utilizes commercial fluorine-containing building blocks1f

2.3. Silver-Mediated Fluorination (functionalized compounds containing F, CF2, CF3, and/or OCF3; e.g., 2.4. Fluorination Mediated by Other Transition Metals 4-fluorobenzaldehyde). The latter method utilizes fluorinating reagents, 3. Transition-Metal-Promoted Trifluoromethylation most of which are commercially available (Figure 1). Their mode of

3.1. Copper-Mediated Trifluoromethylation action is to introduce F or CF3 into target molecules through functional 3.1.1. Trifluoromethylation of Electrophiles (MainlyArI) group transformations. If suitable fluorination methods are available,

3.1.1.1. Trifluoromethylation via a CF3Cu medicinal chemists usually prefer these because they can introduce Intermediate Generated in Situ fluorine where and when they want it.

3.1.1.2. Trifluoromethylation via a Preformed CF3Cu Fluorination of an aromatic system has been effected most commonly Complex through the Balz–Schiemann reaction (transformation of anilines into Transition-Metal-Mediated Fluorination, Difluoromethylation, and Trifluoromethylation 68 Zhuang Jin, Gerald B. Hammond,* and Bo Xu*

aryl fluorides via diazonium fluoroborates)4 and the Halex reaction emerged as powerful methods for synthesizing fluorinated agents8 as (halogen-exchange reaction),5 whereas the introduction of fluorine demonstrated by the number of excellent reviews that have appeared on into an aliphatic system has been accomplished through nucleophilic the topic in the past three years.9 (e.g., the conversion of alcohol into fluoride via DAST) or electrophilic In this review, we survey ground-breaking research on fluorination (e.g., fluorination of the a carbon of a carbonyl group by transition-metal-mediated fluorination, difluoromethylation, and Selectfluor®).1g Moreover, trifluoromethylation has relied heavily on trifluoromethylation reactions that have been reported from 2007 to the Swarts reaction6 and the reaction of carboxylic acid derivatives early 2012. Some transition-metal-catalyzed fluorinations, such as 7 10 with SF4. Although all of these traditional methods employ relatively the electrophilic fluorination of metal enolates, are not included, but cheap starting materials or reagents—without the need for expensive can be found in other reviews.9a,b Some reactions that are not typical metal catalysts—they often require harsh conditions, leading to poor for transition metals are also not covered.11 Moreover, in the case of functional-group tolerance. difluoromethylation and trifluoromethylation methods, we limit the

Transition-metal catalysis plays a major role in organic synthesis discussion to the use of C-1 fluorine sources, such as TMSCF3. (e.g., in coupling and metathesis reactions). In recent years, many transition-metal-catalyzed fluorinations and trifluoromethylations have 2. Transition-Metal-Promoted Fluorination 2.1. Pd-Mediated Fluorination 2.1.1. Fluorinations via Reductive Elimination from R–Pd–F 2.1.1.1. Reductive Elimination from R–Pd(II)–F (a) Nucleophilic Fluorine Source Transition metals, such as palladium, have been widely used KF, CsF, AgF, TBAF, HF, Py•(HF)x (Olah's Reagent), Et3N•3HF for carbon–carbon and carbon–heteroatom bond formations via reductive elimination from a R–Pd(II)–X intermediate (X = carbon or (b) Electrophilic Fluorine Source heteroatom). However, reductive elimination from R–Pd(II)–F is not Cl Cl Me + F + F F an easy task, as demonstrated by the pioneering work of Grushin and N N N N + F N N N N 12 PhO2S SO2Ph co-workers on the reductive elimination from isolated [ArPd(II)F] F F Me Me – – – – – complexes. It wasn’t until 2009 that a successful catalytic C–F bond 2BF4 2PF6 BF4 TfO TfO 13 ® formation via R–Pd(II)–F was achieved. Buchwald and co-workers Selectfluor F-TEDA-PF6 NFSI 1a 1b 2a 2b 2c 3 found that bulky tBuBrettPhos is a suitable ligand for palladium, which enables the fluorination of aryl triflates with CsF (eq 1).13 Increasing (c) Nucleophilic CF3 Source OMe the amount of CsF to 6 equivalents and adding 30 mol % of the – TMSCF3 TESCF3 F3C B OMe solubilizing agent poly(ethylene glycol) dimethyl ether significantly OMe K+ Ruppert–Prakash's shortens the reaction time (one example showed a full conversion in Reagent 4 less than 30 min). Although many functional groups can be tolerated in the fluorination protocol, no reaction took place with aryl triflates (d) Electrophilic or Radical CF3 Precursor containing amines or carbonyls in the ortho position. It was proposed Me O Me that the C–F bond formation proceeds through reductive elimination I–CF O O + + + 3 S S S I I from an arylpalladium(II) fluoride complex, and DFT calculations CF CF CF 14 CF3 CF3 3 3 3 by Yandulov and Tran showed that the C–F bond is formed via a – – – TfO TfO BF4 concerted reductive elimination from an arylpalladium(II) fluoride Togni's Togni's Umemoto's complex. The fluorination needed high temperatures and long reaction Reagent (5a) Reagent II (5b) Reagent (6a) 6b 6c times, rendering the methodology inapplicable for the introduction of 18F, which is used for Positron Emission Tomography (PET) studies. To solve this problem, Buchwald’s group later developed a microflow Figure 1. Common Fluorine Sources Covered in this Review. CsF packed-bed reactor, in which aryl triflates can be fluorinated in a short time (20 min).15 More specifically, a toluene solution containing aryl triflate, palladium, and ligand was injected into a CsF packed- bed reactor which was heated to 120 oC or 130 oC, and the fluorinated CsF (2 equiv) products were collected from the outlet of the reactor. BuBrettPhos (6 mol %) OTf t F [(cinnamyl)PdCl]2 (2 mol %) A more recent mechanistic investigation conducted by Buchwald R R PhMe or CyH and co-workers revealed that C–F reductive elimination from an o 7880–130 C, 12 h LPd(II)(Ar)F complex (L = tBuBrettPhos or RockPhos) does not occur 57–84% when the aryl group is electron-rich, but rather, a modified phosphine, Ar reductive II generated in situ, serves as the actual supporting ligand during catalysis LnPd elimination OMe 16 F with such substrates.

MeO P(t-Bu)2 R = alkyl, alkenyl, Ph, CO, NO2, amino i-Pr i-Pr 2.1.1.2. Reductive Elimination from R–Pd(IV)–F According to the previously mentioned work of Grushin and co- workers,12 reductive elimination from R–Pd(II)–F is not an easy path i-Pr tBuBrettPhos for aryl fluoride synthesis. Sanford and others found that reductive elimination from R–Pd(IV)–F could be a more favored process; this may eq 1 (Ref. 13) be due to the fact that a Pd(IV) center is possibly more electron-deficient than its Pd(II) counterpart, thus favoring reductive elimination. The 69 VOL. 45, NO. 3 • 2012 first Pd(II)-catalyzed C–H fluorination through reductive elimination from R–Pd(IV)–F was reported by Sanford in 2006 (Scheme 1, Part R Pd(OAc) R R + F 2 L (a)).17 Likewise, Yu and co-workers have reported an efficient, Pd(II)- (10 mol %) n H N – IV F (a) + BF4 Pd catalyzed synthesis of aryl fluorides using a nitrogen-containing N MeCN, F3CC6H4 N F N functional group as directing group (Scheme 1, Parts (b) and (c)).18 µw (300 W) 150 oC, 1.5–2 h More applications of Pd(IV) reductive eliminations have recently 92a 10, 33–75% been published. Furuya and Ritter reported direct evidence of carbon– (2.5–4.5 equiv) fluorine formation via reductive elimination from a high-valence Me F Pd(OTf) •2H O palladium fluoride complex.19 Accordingly, palladium(IV) difluoride + F 2 2 NHTf N (10 mol %) NHTf 15 was carefully prepared and characterized by X-ray crystallography, (b) R + TfO– R Me Me NMP (0.5 equiv) and led upon heating to the fluorinated product, 16, through reductive DCE, 120 oC 11 2c (1.5–3.0 equiv) 12, 41–88% elimination (Scheme 2, Part (a)).19 Further systematic mechanistic 0.5–8 h analysis showed that the ancillary pyridylsulfonamide ligand played O Me F O Pd(OTf) (MeCN) 3 + F 2 4 a crucial role in the reductive elimination, probably due to a k - NHAr N (10 mol %) NHAr (c) R + TfO– R coordination mode, in which the oxygen atom of the sulfonyl group NMP (20 or 50 mol %) 20 Me Me coordinates to palladium. Similarly, another palladium(IV) difluoride DCE, 100 or 120 oC 13 2c (1.5–3.0 equiv) 14, 36–78% complex, 17, isolated by Ball and Sanford, furnished the fluorinated 2–24 h aromatic compound 18 upon heating with electrophilic fluorinating NMP = N-methyl-2-pyrrolidinone Ar = 2,3,5,6-tetrafluoro-4-(trifluoromethyl)phenyl agents (XeF2, NFSI (3)) in nitrobenzene, hinting that C–F reductive elimination is a reasonable pathway (Scheme 2, Part (b)).21 Scheme 1. Synthesis of Aryl Fluorides through Reductive Elimination from Ritter’s group later reported a palladium complex, 19, that can be a R–Pd(IV)–F Intermediate. (Ref. 17,18) readily transmetallated to yield complexes 21.22 Both 19 and 21 contain a bidentate ligand possessing a neutral and an anionic nitrogen donor. Complex 21 subsequently reacted with Selectfluor® to yield fluorinated F N aromatic compounds 8 under mild conditions in a short reaction time F 22 DMSO (Scheme 3). Two possible reaction mechanisms were proposed, (a) N Pd N namely electrophilic palladium–carbon bond cleavage and carbon– F 150 oC, 10 min N fluorine reductive elimination from a high-valency palladium complex. o-Ns 15 16, 97% Compared to the harsh reaction conditions and limited substrate scope of conventional fluorination methodologies, this method seems to be o-Ns = ortho-nitrobenzenesulfonyl F very appropriate for late-stage fluorination, including introducing 18F 18 into organic compounds. F is one of the most relevant radioisotopes in t-Bu Positron Emission Tomography (PET),3,23 and its incorporation requires N F F+ (b) Pd F F not only a fast reaction because of its 110-minute half-life, but also o N F O2NC6H5, 80 C that the reaction be carried out at the end of any synthetic sequence. F 17 18 Ritter’s group has recently developed a fluoride-derived, electrophilic t-Bu H + late-stage fluorination reagent for PET studies.24 The authors prepared F F Yield palladium(IV) complex 22, which reacted with a nucleophilic fluoride XeF2 92% source (KF) to form monofluorinated palladium(IV) complex 23 3 83% – (confirmed by X-ray crystallography) (Scheme 4).24 Palladium(II) 2c (BF4 ) 50% complexes 24 were prepared from arylboronic acids 20 and complex 19´, and reacted with 23 in acetonitrile at room temperature to produce Scheme 2. Evidence of C–F Bond Formation via Reductive Elimination from Pd(IV). (Ref. 19,21) 25. The monofluorinated benzene products, 8, were formed through reductive elimination from complex 25 upon heating at 80 oC for 10 min. The entire procedure, including HPLC purification, took less than 18 B(OH) 60 min, a time length that is appropriate for F PET imaging. Using this Ns 2 Ns 18 N N protocol, the authors prepared three F-containing drug analogues with K2CO3 N Pd Py + R N Pd Py radiochemical yields of 10–33%. OAc MeOH–C6H6 (1:1) o 23 C, 2–18 h R 2.1.2. Pd-Catalyzed Fluorination of Allyl Halides and Allyl Esters 19 20 21, 65–91% The use of a nucleophilic fluorine source in the Tsuji–Trost reaction, Selectfluor® (1) instead of carbon, nitrogen, or oxygen nucleophiles can be a challenge.25 MeCN or acetone 50 oC, 0.5 h Recently, though, a palladium-catalyzed asymmetric synthesis of 26 allylic fluoride was reported by Katcher and Doyle. Cyclic allylic F Ns = 4-nitrobenzenesulfonyl chlorides 26 were asymmetrically fluorinated by AgF, in the presence of R = H, alkyl, alkoxy, Ph, OH, C(O), halogen R palladium(0) and Trost’s bisphosphine ligand 27, to form the fluorinated 8, 31–81% products in moderate-to-high yields and excellent enantioselectivities (Scheme 5, Part (a)). The reaction also produced a small amount of the Scheme 3. Palladium-Mediated Fluorination of Boronic Acids. (Ref. 22) diene elimination products (less than 10 %). The authors proposed that Transition-Metal-Mediated Fluorination, Difluoromethylation, and Trifluoromethylation 70 Zhuang Jin, Gerald B. Hammond,* and Bo Xu*

Me 2+

N F + B(OH)2 KF (1 equiv) -Ans -Ans p R (20) p N 18-cr-6 (3 equiv) N N N Pd N Pd N N Pd Py N Pd Py N N MeCN N N K2CO3 OAc B o N N 50 C, 5 min N B N R PhH–MeOH (1:1) N N o N N N N 23 C, 10 h 2TfO– TfO–

22 23 (90%) 24 (47–94%) 19'

18 19 F = F or F; p-Ans = para-anisylsulfonyl MeO MeCN, 23 oC +

O S N TfO– F acetone O Py N N N N R Pd + Pd B o N N N 85 C, 10 min R N N N 19F: 67–93% F 18F: 10–33% 8 25

Scheme 4. Fluoride-Derived Electrophilic Late-Stage Fluorination Protocol for PET Studies. (Ref. 24)

Pd(0) oxidatively adds to 26 with inversion of configuration by an NS 2- allyl ester must be attached to an aromatic ring, and (ii) only primary type reaction, generating a AgCl precipitate that becomes the driving fluorides can be prepared by this protocol. force for the fluorination. The attack of fluoride on the allylpalladium intermediate gives an overall retention of configuration. Although this 2.1.3. Pd-Catalyzed Fluorination via Oxidative Addition to fluorination can tolerate functional groups including ethers, amines, C–C Unsaturated Bonds esters, and amides, it is not without limitations: (i) the leaving group Another category of Pd-catalyzed fluorinations is the oxidative can only be chloride, and (ii) the synthesis of acyclic allyl fluorides nucleophilic addition to C–C unsaturated bonds. Palladium-catalyzed is not effective. Later, Doyle’s group succeeded in developing an intra- and intermolecular aminofluorinations of alkenes were developed 29 enantioselective fluorination of acyclic allylic halides by employing by Guosheng Liu and co-workers. AgF and PhI(OPiv)2 were palladium(0) and Trost’s naphthyl ligand 30 in a similar way.27 Thus, employed as the fluorinating agent and oxidant, respectively, in the acyclic allylic chlorides (or bromides) 29 were fluorinated with excess intramolecular aminofluorination of unactivated terminal alkenes 34 to AgF to yield branched allylic fluorides 31 in moderate-to-good yields vicinal fluoroamines 35 in the presence of palladium catalyst (Scheme 29a (Scheme 5, Part (b)). While linear allylic chlorides (or bromides) 6, Part (a)). The authors found that addition of MgSO4 improves 29 underwent the reaction with only moderate enantioselectivity, the yield of 35, and that the C–F bond is probably formed by direct

a-branched or heteroatom-substituted allylic halides were fluorinated reductive elimination from a Pd(IV) intermediate or through an SN2- with excellent 90–97% ee’s. Substituents and functional groups such type nucleophilic substitution of fluoride. as benzyl and silyl ethers, aldehydes, and alkyl bromides are tolerated Liu also reported on the intermolecular variant, whereby a variety in this protocol. of styrenes 36 were regioselectively aminofluorinated with the Gouverneur’s group also developed a palladium-catalyzed allylic electrophilic fluorinating agent N-fluorobenzenesulfonimide (NFSI, 3) fluorination using para-nitrobenzoate as leaving group and TBAF•(t- to give fluorinated amines 37 in up to 80% yields (Scheme 6, Part 28 29b BuOH)4 as fluoride source. Phenyl-substituted allylic ester 32 was (b)). Both electron-donating and electron-withdrawing groups fluorinated to give allylic fluoride 33 in high yield (Scheme 5, Part at different positions of the phenyl ring had no significant effect on (c)). The mild conditions and fast reaction times allowed the authors to this fluoroamination, although the reaction gave small amounts of prepare an allylic “hot” fluoride. 18[ F]-Labeled cinnamyl fluoride was difluorinated amine as side product. The authors found that the reaction obtained in radiochemical yields of 9–42% (n = 12; n being the number is restricted to activated alkenes, such as styrene, and proposed that of carried-out experiments) for a reaction quenched after 5 min, and no Pd(0) is oxidized by NFSI to a Pd(II) intermediate, with the ensuing significant improvement was observed after 30 min (10–51%, n = 5). fluoropalladation of styrene being a key step in the aminofluorination There are some limitations to this method: (i) the double bond in the process. 71 VOL. 45, NO. 3 • 2012

Peng and Liu employed a palladium-catalyzed tandem fluorination– rearrangement and fluorination of propargyl acetates, leading to cyclization of enyne 38 to fluorinated lactam39 in good yield (Scheme a-fluoroenones in good yields and regioselectivities. Later, De Haro and 6, Part (c)).30 Enynes containing branched chains or internal olefins were Nevado disclosed the synthesis of a-fluoroketones ora -fluoroacetals,50 , not suitable, yielding poor-to-trace amounts of the lactam. The authors using similar conditions (Scheme 9, Part (c)).41 Furthermore, Liu, Xu, proposed that cis-fluoropalladation (predominant) of the alkyne occurs and co-workers reported the gold-catalyzed tandem aminofluorination first, followed by alkene insertion into the vinylpalladium intermediate to form a Csp3–Pd bond, which is then reduced by isopropanol to give fluorinated lactam 39. Interestingly, the additive 4-nitrophenol was Pd2(dba)3 (5 mol %) found to slightly increase the yield of the lactam. Cl 27 (10 mol %) F (a) AgF (1.1 equiv) X X 2.2. Gold-Catalyzed Fluorination THF, rt, 24 h 2.2.1. Using a Nucleophilic Fluorine Source 26, (+/–) 28 X = C, N, O 56–85% The discovery by Sadighi’s group that an (NHC)gold(I) fluoride 85–96% ee complex reacts with excess alkyne to give a b-(fluorovinyl)gold Pd (dba) (5 mol %) 2 3 F complex prompted them to develop this transformation into a gold- 30 (10 mol %) 31 (b) R X catalyzed hydrofluorination of alkynes to fluoroalkenes. Several AgF (3 equiv) R types of alkyne, 40, were effectively trans-hydrofluorinated to yield 29 PhMe, rt, 48 h 31 X = Cl, Br up to 88% fluoroalkenes41 in good yields and moderate regioselectivities (Scheme b:l = 1:4 to >20:1 7, Part (a)). The authors reported that (i) good regioselectivities can b = branched; l = linear up to 97% ee be achieved only with aryl–alkyl alkynes, (ii) Et3N•3HF is a suitable TBAF•(t-BuOH)4 (2.5 equiv) Ph nucleophilic fluorinating agent, and (iii) powdered KHSO4 and a co- Ph F (c) OCO2(C6H4-4-NO2) catalyst, PhNMe2•HOTf, greatly increase the yield of the fluoroalkenes. [Pd(dba)2] (5 mol %) 32 PPh3 (15 mol %) 33 The regioselectivity limitations can be overcome by employing THF, rt, 1 h 66 to >95% directing groups. Miller and co-workers chose a 2,2,2-trichloroethoxy- carbamate moiety as directing group to successfully hydrofluorinate alkynes 42 stereo- (trans) and regioselectively (rr 49:1 to >50:1) to give O O O O predominantly Z fluoroalkenes 43 (Scheme 7, Part (b)).32 NH HN NH HN

PPh2 Ph2P PPh2 Ph2P 2.2.2. Using an Electrophilic Fluorine Source 27 ( ) 30 ( ) Gouverneur’s group developed the first gold-catalyzed oxidative R,R R,R fluorination reaction, by mixing a difluorinated ynone (PhCH(OH)– ® CF2C(O)CCPh), AuCl, and Selectfluor in acetonitrile at room Scheme 5. Fluorination of Allylic Halides and Allylic Acetates. (Ref. 26–28) temperature for a few days.33 A trifluorodihydropyranone (20%) was obtained together with a difluorinated pyranone (33%), formed from the competitive protodeauration of the organogold intermediate. A fluorodeauration mechanism was proposed to explain the formation of Pd(OAc) (10 mol %) R R R 2 F PhI(OPiv)2 (2 equiv) R the fluorinated products observed. (a) TsHN + AgF 34 MgSO (2 equiv) Gold-catalyzed multicomponent transformations are very rare. Our 4 N MeCN, rt Ts group recently accomplished a truly gold-catalyzed, multicomponent 34, R = H, Me, Ph 5 equiv 35, 80–84% transformation—the functionalized hydration of alkynes (Scheme 8).35 N(SO2Ph)2 A major shortcoming of traditional metal-catalyzed hydration is that Pd(OAc)2 (5 mol %) F BC (7.5 mol %) F it only adds the elements of H2O to an alkyne, whereas functionalized (b) R + N R PhO2S SO2Ph dioxane hydration combines four intermolecular components (alkyne, o 36 NFSI 40–50 C, 10 h 37, 37–80% nucleophile, electrophile, and coupling reagent) to form a single product, R = H, Me, NO2, CN, F, Cl, 2.5 equiv a highly functionalized a-fluoroketone, in good yield and moderate Br, MeO2C, AcO, Ms, Ts regioselectivity. This one-pot, multicomponent transformation—using Ph F Pd(O2CCF3)2 (5 mol %) readily available starting materials (alkyne, water, organoboronic acid, F BC (7.5 mol %) Ph H (c) + N ® PhO2S SO2Ph and Selectfluor )—has obvious advantages over literature methods 4-NO2C6H4OH (20 mol %) O O N o N that need multiple steps and are not library-friendly.36 We also detected Ts DMA, 50 C, Me2CHOH Ts experimentally (by 19F NMR and X-ray photoelectron spectroscopy) 38 NFSI 39, 70% E:Z = 5:1 that Au(I) is oxidized by Selectfluor® to give a Au(III) species, which DMA = N,N-dimethylacetamide Ph Ph is much more active than Au(I) in hydration and cyclization. In the proposed mechanism, the nucleophile (water) first attacks the gold- N N Me Me 37 activated alkyne to form a vinylgold complex, which can further bathocuproine (BC) 3 2,9-dimethyl-4,7-diphenyl-1,10-phenanthroline react with a metal reagent (e.g., R B(OH)2) through a transmetallation process.38 The final C–F formation mechanism is not clear yet; one possibility is that gold actually enhances the nucleophilicity of the vinylgold intermediate, allowing it to react with Selectfluor®. Scheme 6. Palladium-Catalyzed Fluorinations and Cyclizations as Reported by Guosheng Liu and Co-workers. (Ref. 29,30) Nevado’s (Scheme 9, Part (a))39 and Gouverneur’s groups (Scheme 9, Part (b))40 have independently developed an efficient gold-catalyzed Transition-Metal-Mediated Fluorination, Difluoromethylation, and Trifluoromethylation 72 Zhuang Jin, Gerald B. Hammond,* and Bo Xu*

of propargyl hydrazines 51 to give, after cyclization and fluorination, gold might not play a significant role in the formation of the C–F bond.

fluorinated pyrazoles 52 in moderate-to-excellent yields (Scheme 9, Direct evidence for the Csp3–F reductive elimination was reported Part (d)).42 However, the reaction also produced significant amounts of by Mankad and Toste,43 who prepared an alkyl(IPr)gold complex and

nonfluorinated pyrazoles. readily oxidized it by XeF2 to give the corresponding difluorinated Reductive elimination or fluorodeauration has been proposed as a tetracoordinate gold(III) complex. Reductive elimination from this in possible mechanism for the formation of the C–F bond in the preceding situ generated Au(III) complex led to the monofluorinated products in reactions.42 There is, however, no direct experimental evidence so low-to-moderate yields, with the yield being dependent on the nature far to support these hypotheses, and in all of these cases, the normal of the alkyl group. If a b hydrogen (relative to gold) is present in the protodeauration product (the compound that is obtained in the absence alkyl group, b-hydride elimination becomes competitive with C–F of a fluorination reagent) can react with Selectfluor® to give the reductive elimination; while a carbocation-like rearrangement occurs corresponding fluorinated product in good yield without the need for prior to C–F reductive elimination, when the alkyl group bears a strained a gold catalyst (Scheme 9, Part (e)).40–42 Therefore, it is most likely moiety and lacks a b hydrogen. Density Functional Theory (DFT) that these reactions go through a tandem two-step mechanism. The calculations supported a C–F reductive elimination occurring through a fluorination step may occur outside of the gold-catalyzed cycle, and transient cationic intermediate. Toste’s work demonstrates that reductive elimination from R–Au(III)–F may be a feasible process.

2.3. Silver-Mediated Fluorination LAuX (2.5 mol %) The silver-mediated fluorination of arylstannanes and boronic acids PhNMe2•HOTf (10 mol %) 1 Et3N•3HF, KHSO4 R F was pioneered by Ritter and co-workers, who reported the fluorination (a) R1 R2 CH Cl , rt, 18–30 h ® 44 2 2 H R2 of aryl- and alkenylboronic acids with Selectfluor (1a). Arylboronic 40 41, 63–86% acids 20 were mixed with stoichiometric amounts of AgOTf and NaOH (a 1 crucial additive) in methanol to give an isolable arylsilver intermediate, R = n-Pent, Ph, 4-MeOC6H4, 4-AcC6H4, thien-2-yl 2 ® R = n-Pent, n-Hex, Ph which reacted with Selectfluor to furnish the fluorinated products 8 in good yields (Scheme 10, Part (a)). In addition to arylboronic acids, CCl3 LAuX (2.5 mol %) CCl3 PhNMe2•HOTf (2.5 mol %) alkenylboronic acids can be fluorinated to give fluoroalkenes (65–85%), O O O O R2 Et3N•3HF, KHSO4 H in which the alkene stereochemistry is preserved. This methodology (b) 2 HN CH2Cl2, rt, 15 h HN R can tolerate electron-poor, electron-rich, di-ortho-substituted arenes, R1 R1 F and heteroaromatics. The authors also prepared a few fluorinated 42 43, 57–74% pharmaceutically active agents, demonstrating the broad functional-

1 2 R = i-Pr, Ph; R = Me, n-Bu, Ph group tolerance of their methodology. Preliminary mechanistic investigations hinted that bimetallic oxidation–reductive elimination X = BF4, Ot-Bu is a possible pathway for C–F bond formation. There are, however, a i-Pr i-Pr i-Pr i-Pr couple of limitations to this method: (i) the use of 2 equivalents of a

L = N N N N silver salt, and (ii) the formation of 10–20% of hydrodestannylated side 45 -Pr products in the case of arylstannane substrates. i-Pr i-Pr i-Pr i Cl Cl Ritter’s group also developed a silver-mediated fluorination of SIPr ClIPr arylstannanes 53 using the fluorination agent 1b (an anion-exchange product of Selectfluor®), obtaining fluorinated arenes in moderate-to- good yields (Scheme 10, Part (b)).45 1b was found to give higher yields Scheme 7. Gold-Catalyzed Hydrofluorination of Alkynes.(Ref. 31,32) of the fluorination products than Selectfluor®. Ritter and co-workers also developed a silver-catalyzed fluorination of arylstannanes 54 using 1b.46 The key to using only catalytic amounts of silver salt was

to employ the additives NaHCO3 and NaOTf (Scheme 10, Part (c)).

2 O R 3 Ph3PAuCl (5 mol %) Carbohydrates, peptides, polyketides, and alkaloids were tolerated + R B(OH)2 R1 (a) R2 1 + H2O + 1a MeCN–H O (20:1) by this method, although certain amines, thiol ethers, and carboxyl R 2 3 rt, 18–24 h R F groups were not tolerated. The proposed fluorination mechanism of 1 44, 47–90% R = Me, Et, n-Pent, AcO, BzO, BnCO2, BzO(CH2)2 the catalytic variant is similar to that proposed for the silver-mediated 2 rr = 2.1:1 to 6.7:1 R = Me, n-Pr, n-Bu 3 13 examples stoichiometric fluorination and is thought to proceed via intermediate R = Ph, 4-XC6H4 (X = Me, Ph, F, Cl) 55 (Scheme 10, Part (d)). O A silver-catalyzed intramolecular oxidative aminofluorination of -Bu n OBz allenes with NFSI (3) has been developed by Liu and co-workers,47 Ph F OBz + PhB(OH) (2 equiv) Ph3PAuCl (5 mol %) 46 (b) 2 + and represents an efficient method for the synthesis of various 4-fluoro- + H2O + 1a (2.5 equiv) MeCN–H O (20:1) O n-Bu 2 2,5-dihydropyrroles, which can be elaborated into the corresponding rt, 18 h BzO 45 n-Bu fluorinated pyrroles in good yields. Ph F 47 46 + 47, 88% 2.4. Fluorination Mediated by Other Transition Metals 46:47 = 6.7:1 The first platinum-mediated fluorination was reported by Vigalok’s group (Scheme 11, Part (a)).48 Platinum(II) complex 56 was prepared Scheme 8. Functionalized Hydration of Alkynes. (Ref. 35) and oxidized by XeF2 to yield the benzylic-fluorinated product 57 in quantitative yield. The authors proposed that platinum(II) is oxidized to 73 VOL. 45, NO. 3 • 2012 platinum(IV) complex 58, and fluoride assists in the metallation of the benzyl group to give complex 59 before rendering the product via C–F 1a (2 equiv), NaHCO3 R2 O 1 OAc reductive elimination. Gagné’s group has also developed a platinum- R (IPr)AuNTf2 (5 mol %) (a) 49 R R1 R mediated fluorination protocol (Scheme 11, Part (b)). For example, 2 MeCN–H O (20:1) R 2 F 80 oC, 20 min complex 60 was reacted with various electrophilic fluorinating agents, 48 49, 57–87% such as XeF2 and N–F salts, to deliver the C–F reductive elimination R, R1, R2 = alkyl, cycloalkyl, Ph product 61 in good yield. The fluorination became more efficient as the 1a (2 equiv) 1 steric congestion of the platinum complex grew larger. A benzyl group, R1 OAc (SIPr)AuCl (5 mol %) R O (b) R R2 R lacking a b hydrogen, gave high yield of the fluorination product, but R2 AgOTf (12.5 mol %) other simple acyclic alkyl groups were not fluorinated efficiently owing MeCN, 40 oC, 1.5–72 h F 48 49, 29–85% to competitive b elimination. R, R1, R2 = H, alkyl, cycloalkyl, aryl A copper-catalyzed, halide-exchange reaction under very mild 1a (2 equiv) reaction conditions using a family of model aryl halides has been MeO OMe 50 Ph3PAuNTf2 (5 mol %) reported by Ribas and co-workers (Scheme 11, Part (c)). Their work (c) R1 R2 R2 R1 MeOH, 70 oC suggests that reductive elimination from R–Cu–F is also a feasible F process, although it is still too early to determine the impact of its R1, R2 = H, alkyl, aryl 50, 71–92% application in synthesis. 1a (2 equiv) 2 R2 R NH Ph3PAuNTf2 (2.5 mol %) N 1 N (d) R N 3. Transition-Metal-Promoted Trifluoromethylation NaHCO3 (2 equiv) R3 MeCN, rt, 2 h F R3 3.1. Copper-Mediated Trifluoromethylation R1 3.1.1. Trifluoromethylation of Electrophiles (Mainly Arl) 51 52, 30–90% Since the trifluoromethylation of aromatic compounds with metal R1, R2, R3 = alkyl, aryl, heteroaryl complexes has been extensively reviewed by Tomashenko and Grushin,9f we will limit our discussion to new developments in this (e) Possible Reaction Intermediate: R2 R1 rapidly expanding area. N 1 N R2 • R R R OMe R3 3.1.1.1. Trifluoromethylation via a CF3Cu Intermediate OAc H Generated in Situ in (a) & (b) in (c) in (d) The generation (from CF2Br2), spectroscopic detection, and chemical reactivity of CF3Cu (trifluoromethylcopper) has been revealed through the pioneering work of Burton and co-workers.51 Another source of Scheme 9. Gold-Catalyzed Fluorination of Alkynes. (Ref. 39–42) 9f 52 CF3Cu is the CuI–Cu or CF3Hg–Cu system. An improved protocol, whereby the transient trifluoromethylcopper species is generated in situ 1. NaOH (1.0 equiv) from TMSCF3 (Ruppert–Prakash’s reagent, a more convenient source AgOTf (2.0 equiv) of CF3) in the presence of cuprous iodide and potassium fluoride, has MeOH, 0 oC 53 been advanced. All of the most recent work on trifluoromethylation (a) Ar B(OH)2 Ar F 54 2. 1a (1.05 equiv) has been based on Me3SiCF3 or reagents derived from it. 20 3 Å MS, acetone, 23 oC 8, 70–85% Amii and co-workers reported an improved copper-catalyzed Ar = mono- and disubstituted benzene, 1-Np, HetAr trifluoromethylation of aromatic iodides by (trifluoromethyl)- triethylsilane.55 They designed their catalytic cycle based on the 1b (1.2 equiv) (b) ArSn(n-Bu)3 Ar F knowledge that the reaction between the copper catalyst and the AgOTf (2.0 equiv) 53 acetone, 23 oC, 20 min 8, 63–83% trifluoromethylating agent to form the active intermediate CuCF3 is very fast, but that the second step, after delivery of the trifluoromethylated Ar = HetAr, 2,4,6-Me3C6H3, 4-RC H (R = H, Ph, MeO, OH, CN, CHO, F, Me N(O)CH ) arene, is much slower and cannot resupply sufficient amounts of the 6 4 2 2 copper source for the first reaction. The authors found that adding a Ag2O (5 mol %) NaHCO3 (2.0 equiv) diamine ligand such as 1,10-phenanthroline solved the problem. NaOTf (1.0 equiv) (c) ArSn(n-Bu)3 Ar F Both electron-rich and electron-poor aromatic iodides can be 1b (1.5 equiv) 54 8, 76–89% efficiently trifluoromethylated by3 Et SiCF3 (Scheme 12, Part (a)). MeOH (5 equiv) Furthermore, heteroaromatic iodides (three examples) were smoothly acetone, 65 oC, 4 h trifluoromethylated by this protocol. Ar = 2,4,6-Me3C6H3, 4-RC6H4 (R = Ph, MeO, Br, CN) A collaborative team led by Weng developed a bimetal-mediated [a combination of CuI (10 mol %) and AgF (1.3 equiv)] trifluorometh- (d) Presumed Pathway for C–F Bond Formation in (a) and (b) 56 ylation of aryl iodides with Me3SiCF3, which is much cheaper than F II NaHCO3 or NaOTf Et SiCF , to the give trifluoromethylated products in moderate-to- Ar Ag Ln Ar–F + 2 AgI 3 3 reductive elimination excellent yields (Scheme 12, Part (b)). A mechanistic study revealed [AgII] 55 that AgCF3 is most probably generated first, and then transfers the CF3 group to copper to form CuCF3, which is the active catalytic species. Hafner and Bräse also reported an efficient method for the Scheme 10. Ritter’s Silver-Mediated Fluorination of Arylstannanes and Arylboronic Acids. (Ref. 44–46) trifluoromethylation of halogenated double bonds using in situ generated “trifluoromethyl copper”. TMSCF3 was converted selectively Transition-Metal-Mediated Fluorination, Difluoromethylation, and Trifluoromethylation 74 Zhuang Jin, Gerald B. Hammond,* and Bo Xu*

into “trifluoromethyl copper” using CuI as the copper source and KF as Me3SiCF3, and introduced it as a stable, easy-to-handle, and efficient promoter; the trifluoromethylation of activated and unactivated alkenyl trifluoromethylating agent for aryl iodides in the presence of a copper halides occurred mostly in good-to-excellent yields in up to a gram catalyst, giving rise to the trifluoromethylated products in moderate-to- scale.57 excellent yields (Scheme 13, Part (a)).58 A wide variety of substituents

In addition to using R3SiCF3 directly as the CF3 source, other and functional groups, such as cyano, sulfide, halide, ester, amide, and

reagents have been employed to generate CF3Cu, although many of acetal are tolerated in the reaction. More importantly, while Gooßen’s

these are derived from R3SiCF3. Gooßen and co-workers prepared protocol allows the trifluoromethylation of heterocyclic iodides, one of

potassium (trifluoromethyl)trimethoxyborate (F3CB(OMe)3K, 4) from its limitations is that the trifluoromethyl moiety can react with ketones and aldehydes. Xiao’s group has reported a copper-mediated trifluoromethylation AdAd AdAd P Me P Me of heteroaromatic iodides using trifluoromethylsulfonium salt 6b and 59 XeF2, CH2Cl2 Cu (Scheme 13, Part (b)). A large variety of trifluoromethylated (a) Pt Me Pt Me –30 oC to rt heteroaromatics including pyridine, pyridazine, pyrazole, imidazole, Py Py Me F indole, and other oxygen- or sulfur-containing heteroaromatics, were 56 57, 100% prepared in excellent yields. In some cases, the reaction was very XeF2 sensitive to steric hindrance and did not yield the desired products. As C–F elimination for a possible mechanism, the authors proposed that the likely active Me Me Me Me intermediate is CuCF3, which is probably generated via a single- PAd2 PAd2 –HF electron transfer between Cu and 6b. Pt Pt Amii’s group also reported a copper-catalyzed trifluoromethylation F F H F Py Py of aryl iodides using O-silylated trifluoroacetaldehyde hemiaminal 69,

58 59 readily prepared from CF3CH(OH)(OEt) and morpholine. Both electron- rich and electron-poor aryl iodides can be trifluoromethylated in the Ts Ts Me Me presence of a complex of copper and ligand, to give trifluoromethylated N N 60 – XeF2 (1.2–1.5 equiv) aromatics in moderate-to-excellent yields (Scheme 13, Part (c)). (b) BF4 (PPP)Pt CD2Cl2 (anhyd) F However, this protocol failed when a ketone or aldehyde group was H 0 oC, 3 min H present in the aryl iodide, due to the nucleophilic trifluoromethylation 60 61, 91% conditions. PPP = bis(2-diphenylphosphinoethyl)phenylphosphine (triphos) Kremlev, Tyrra, and co-workers reported that Zn(CF3)Br•2DMF and CuBr act as an alternative trifluoromethylating agent, capable Cl of trifluoromethylating aryl iodides (Scheme 13, Part (d)).61 For

AgF (8 equiv) this purpose, Zn(CF3)Br•2DMF and CuBr were mixed in DMF at (c) N CuIII N N F HN + N HN H H H H MeCN, rt, 8 h room temperature for 3 h, and the in situ generated CuCF3 readily N N N+ trifluoromethylated electron-poor aryl iodides at 85–90 oC to give Me Me Me moderate yields of trifluoromethylated products. Unfortunately, the 62 63, 91% 64, 8% reaction with electron-rich aryl iodides was accompanied by formation of a pentafluoroethylation product. Vicic’s group disclosed the Scheme 11. Fluorinations Mediated by Pt and Cu. (Ref. 48–50) decarboxylative trifluoromethylation of aryl iodides or bromides by using a copper trifluoroacetate intermediate.62 Their protocol employed

a mixture of CuI and CF3CO2Na to trifluoromethylate aryl iodides (and bromides) via a decarboxylative pathway to give trifuoromethylated aromatic compounds in moderate yields after heating at 160 oC for 24 CuI (1–10 mol %) 1,10-phenanthroline h. Li, Duan, and collaborators discovered that a ligand-free Cu–Ag2O (5–10 mol %) system facilitated the trifluoromethylation of aryl iodides by CF3CO2Na Et3SiCF3 (1.2–2 equiv) Ar I Ar 63 (a) CF3 (Scheme 13, Part (e)). CF3CO2Na, Cu, and Ag2O were mixed with a KF (1.2–2 equiv) 65 66, 44–99% o NMP, DMF, 60 oC, 24 h variety of aryl iodides in DMF and heated at 130 C for 15 h to give the trifluoromethylated arenes in moderate-to-excellent yields. Several Ar = aryl, heteroaryl functional groups such as nitro, ester, sulfido, amino, and cyano, CuI (10 mol %) were tolerated in this protocol. Shibata’s group reported using shelf- DMEDA (20 mol %) NaOt-Bu (20 mol %) stable, electrophilic trifluoromethylating reagent 6b to effect a copper- Me3SiCF3 (2 equiv) mediated, chemoselective trifluoromethylation at the benzylic position (b) I CF3 R AgF (1.33 equiv) R in good-to-high yields under mild conditions (Scheme 13, Part (f)).64 o 65 NMP, 90 C, 6 h 66, 47–98% Grushin and co-workers discovered the first direct cupration R = alkyl, aryl, alkoxy, NO2, CN, CF3; RC6H4 = 1-Np, 2-MeO-1-Np of fluoroform (HCF3), the most attractive CF3 source for the DMEDA = N,N'-dimethylethylenediamine introduction of the trifluoromethyl group into organic molecules.

The fluoroform-derived CuCF3 solution can be effectively stabilized with TREAT-HF (triethylamine trihydrofluoride, Franz’s Reagent) to Scheme 12. Copper-Catalyzed Trifluoromethylation of Aromatic Iodides by produce a CuCF3 reagent that readily trifluoromethylates organic and R SiCF . (Ref. 55,56) 3 3 inorganic electrophiles in the absence of additional ligands such as phenanthroline.65 75 VOL. 45, NO. 3 • 2012

3.1.1.2. Trifluoromethylation via a Preformed CF3Cu Complex

Vicic’s group isolated (NHC)CuCF3 (72), characterized it by X-ray 4 (3 equiv) crystallography, and reacted it with aromatic and heteroaromatic CuI (20 mol %) (a) Ar I Ar CF3 (Ref. 58) iodides to give excellent yields of the corresponding trifluoromethylated 1,10-phenanthroline 65 66, 52–97% products, albeit five equivalents of the starting iodide was required (20 mol %) (Scheme 14, Part (a)).66 Complex 72 can also trifluoromethylate DMSO (anhyd, deoxygenated) 60 oC, 16 h benzyl bromide in moderate yield (58%). This work provided direct Ar = mono- and disubstituted benzene, heteroaryl evidence for CF3Cu’s role as the intermediate in the copper-mediated trifluoromethylation of organic iodides. 6b (2 equiv), Cu (3 equiv) (b) (Het)Ar I (Het)Ar CF3 (Ref. 59) Hartwig and co-workers prepared and spectroscopically DMF, 60 oC, 9–11 h 67 68, 85–98% characterized (1,10-phenanthroline)CuCF3 (73), and found that it 19 examples Morph-CH(OTMS)CF3 exists in the ionic form [(1,10-phenanthroline)2Cu][Cu(CF3)2] in DMF (69, 2 equiv) (Scheme 14, Part (b)).67 Complex 73 efficiently trifluoromethylates CuI (10 mol %) (c) Ar I Ar CF3 (Ref. 60) 1,10-phenanthroline aromatic iodides containing different functional groups to give the 65 66, 40–97% trifluoromethylated products in excellent yields. Additionally, 73 can (10 mol %) CsF (2 equiv) readily trifluoromethylate heterocyclic iodides; such as quinoline, diglyme, 80 oC, 24 h purine, and indole; and alkenyl iodides (1 example: trans-1-iodooctene, Ar = mono- and disubstituted benzene, 1-Np, 2-Np, 2-Me-thien-5-yl 99%); producing the trifluoromethylated derivatives in excellent yields. More importantly, 73 also trifluoromethylates some aromatic bromides Zn(CF3)Br•2DMF (1 equiv) to deliver moderate-to-good yields of the corresponding products. (d) X Y (Ref. 61) Grushin’s group prepared and isolated [(Ph P) Cu(CF )] and R CuBr (2 equiv), DMF R 3 3 3 o 65 85–90 C, 8 h 66, 51–69% (1,10-phenanthroline)Cu(PPh3)CF3 (74), and demonstrated that 74 could trifluoromethylate aromatic iodides upon heating at 80 oC, R = 4-EtO2C, 4-NO2, 2,4-(NO2)2; X = I, CH2I; Y = CF3, CH2CF3 leading to the trifluoromethyl-substituted products in moderate-to-good Cu (30–40 mol %) 68 yields (Scheme 14, Part (c)). Ag2O (30–40 mol %) (e) Ar I Ar CF3 (Ref. 63) CF CO Na (4 equiv) 65 3 2 66, 56–95% 3.1.2. Trifluoromethylation of Nucleophiles through DMF, 130 oC, 15 h Oxidative Coupling Ar = substituted benzene, 1-Np, quinolin-3-yl, quinolin-8-yl

The CF3Cu intermediate not only can react with electrophiles such as aryl iodides, but it can also participate in oxidative coupling with 6b (2 equiv), Cu (3 equiv) (f) ArCH2 Br ArCH2 CF3 (Ref. 64) selected nucleophiles in the presence of a suitable oxidizing reagent. NMP, 60 oC, 11 h 70 71, 45–83% The copper-mediated aerobic oxidative trifluoromethylation of terminal Ar = substituted benzene, 2-Np, aryl, heteroaryl 69 alkynes with Me3SiCF3 was first reported by Chu and Qing. Both alkyl- and aryl-substituted terminal alkynes 75 were trifluoromethylated Scheme 13. Copper-Mediated Trifluoromethylation of Aromatic Iodides with Me SiCF through the mediation of a ligated CuI (Scheme 15, Part 3 3 Employing a Variety of CF3 Sources. (a)). A number of functional groups and substituents including alkoxy, amino, ester, nitro, chloro, and bromo were tolerated. Very recently, Fu’s group also reported a copper-catalyzed trifluoromethylation of terminal alkynes using Umemoto’s reagent,70 while Weng, Huang, and i-Pr o 71 N DMF, 25 C co-workers reported similar findings by employing Togni’s reagent. (a) X I + CuCF3 X CF3 The first copper-mediated oxidative cross-coupling of aryl- and R N R i-Pr alkenylboronic acids 20 with (trifluoromethyl)trimethylsilane was also 65 (5 equiv) 72 66, 91–99% 72 reported by Qing’s group, who employed a copper(I)–ligand complex X = CH, N; R = H, 4-MeO, 4-Ph as catalyst and a silver salt as oxidant. Both electron-rich and electron- DMF poor aromatic rings were efficiently trifluoromethylated with 3Me SiCF3 (b) Ar I + (Phen)CuCF3 Ar CF3 rt or 50 oC 65 73 66 (Scheme 15, Part (b)). The same research group also showed that their 18 h method is effective for the preparation of trifluoromethylalkenes. 1 equiv 1.2–3.0 equiv 68–100% Independently, Buchwald’s group disclosed the trifluoromethylation Ar = aryl, heteroaryl; Phen = 1,10-phenanthroline of arylboronic acids (Scheme 15, Part (c))73 by employing a process I CF3 that is amenable to typical bench-top setups with the reaction typically DMF (c) + (Phen)Cu(PPh3)CF3 requiring only 1–4 h under mild conditions. Buchwald’s protocol 80 oC, 16 h tolerates a range of functional groups, allowing access to a variety of R R trifluoromethylarenes, and avoids the use of stoichiometric amounts of 65 74 66 silver salt. 1.1 equiv 1.0 equiv 60–85% Qing and co-workers reported the copper-catalyzed oxidative R = H, Me, MeO, F, Cl, NO2, CN, F3C; ArI = 2-iodopyridine trifluoromethylthiolation of arylboronic acids 20 with a combination of Me3SiCF3 and elemental sulfur at room temperature. This reaction provides a concise and efficient method for the synthesis of aryl Scheme 14. Trifluoromethylation of Aryl Iodides with Preformed, Stable CF Cu Complexes. (Ref. 66–68) trifluoromethyl thioethers 77 under mild conditions (Scheme 15, 3 Part (d)).74 Transition-Metal-Mediated Fluorination, Difluoromethylation, and Trifluoromethylation 76 Zhuang Jin, Gerald B. Hammond,* and Bo Xu*

Gooßen and co-workers reported that arylboronic acid pinacol

Me3SiCF3 (5 equiv) esters can be converted into the corresponding benzotrifluorides CuI (1 equiv) (a) with an easy-to-use, one-component trifluoromethylating reagent, R R CF3 (Ref. 69) 1,10-phenanthroline (1 equiv) potassium (trifluoromethyl)trimethoxyborate (4), mediated by copper 75 76, 47–91% KF, DMF, air, 100 oC acetate under an oxygen atmosphere (Scheme 15, Part (e)).75 Qi, Shen, R = alkyl, Ph, aryl, heteroaryl and Lu reported a Cu-mediated ligandless aerobic fluoroalkylation 76 of arylboronic acids using an RfI–Cu combination. Although no Me3SiCF3 (5 equiv) (CuOTf)2•C6H6 (0.6 equiv) examples of trifluoromethylation were provided, this method can, at 1,10-phenanthroline (1.2 equiv) least in theory, be used for trifluoromethylation when ICF3 is employed. (b) R B(OH)2 R CF3 (Ref. 72a) 20 Ag2CO3 (1.0 equiv) 66, 68–90% A copper–catalyzed, direct C–H oxidative trifluoromethylation o KF, K3PO4, DMF, 45 C of heteroarenes has been achieved using Me3SiCF3 and a variety of R = alkenyl, aryl, heteroaryl oxidants such as air or t-BuOOt-Bu (Scheme 16).77 1,3,4-Oxadiazoles, 1,3-azoles, and indoles were trifluoromethylated in yields ranging from Me3SiCF3 (2 equiv) Cu(OAc)2 (1 equiv) moderate to excellent, and the reaction was compatible with a variety (c) R B(OH)2 R CF3 (Ref. 73) 1,10-phenanthroline (1.1 equiv) of substituents and functional groups. 20 66, 34–68% CsF, O2 (1 atm), 4 Å MS The copper-catalyzed oxidative trifluoromethylation approach can DCE or i-PrCN, rt also use suitable electrophilic trifluoromethylation reagents. Sodeoka R = aryl, heteroaryl and co-workers have developed a copper-catalyzed C2-selective R R 78 S8, Me3SiCF3 trifluoromethylation of indoles using 5b. Substituted indoles were CuSCN (10 mol %) trifluoromethylated by 5b in the presence of copper catalyst to give (d) B(OH)2 SCF3 (Ref. 74) 1,10-phenanthroline (20 mol %) low-to-excellent yields of C2-trifluoromethylated indoles (Scheme 17, Ag CO , K PO , 4 Å MS 20 2 3 3 4 77, 58–91% DMF, rt, 24 h Part (a)). Indoles having electron-withdrawing groups, such as an ester or ketone at R1 or R2, resulted in low product yields using this protocol. R = H, Ph, PhO, BnO, t-Bu, Br, (MeO)3, H2C=CH, MeO2C, CN, MeSO2, PhC(O), H2NC(O); ArB(OH)2 (Ar = 1-Np, 6-MeO-2-Np) The proposed reaction mechanism assumes that the copper catalyst increases the electrophilicity of the hypervalent iodine reagent. Me O Me K[(OMe)3BCF3] (4, 2 equiv) Xiao and co-workers have reported a copper(0)-mediated, ligand-free (e) Ar B Ar CF3 (Ref. 75) Me trifluoromethylation of arylboronic acids using 6b to give moderate-to- O O2 (1 atm), Cu(OAc)2 (1 equiv) Me o 79 78 DMSO, 60 C, 16 h 66, 36–99% satisfactory yields of products (Scheme 17, Part (b)). This methodology Ar = aryl, heteroaryl tolerates a variety of substituents and functional groups including halo, aldehydo, and cyano. The authors proposed that the reaction might Scheme 15. Oxidative Trifluoromethylation of Terminal Alkynes and Aryl- involve a Cu(II) or Cu(III) oxidative pathway. On the other hand, Lei and Alkenylboronic Acids. Liu’s group discovered that copper(I) triflate (20 mol %), supported by 2 equiv of 2,4,6-trimethylpyridine, catalyzed the trifluoromethylation of aryl-, heteroaryl-, and vinylboronic acids with 6a at room temperature.80 Independently, Tianfei Liu and Qilong Shen reported the copper-catalyzed

Me3SiCF3 (4 equiv) trifluoromethylation of a broad range of aryl- and alkenylboronic acids Cu(OAc)2 (40 mol %) N N 1,10-phenanthroline (40 mol %) N N with Togni’s reagent (5a), to give trifluoromethylated systems in good- (a) CF 81 Ar O Ar O 3 to-excellent yields (Scheme 17, Part (c)). t-BuONa (1.1 equiv) NaOAc (3 equiv), air, DCE 43–91% Sodeoka and co-workers reported the trifluoromethylation of 4 Å MS, sealed tube, 80 oC, 6 h allylsilanes through the use of CuI and 5b under mild conditions.

Ar = Ph, 1-Np, 4-XC6H4 (X = Me, t-Bu, MeO, CF3, NO2, Cl, MeO2C) The reaction of allylsilanes unsubstituted at the 2 position furnished vinylsilanes, while C2-substituted allylsilanes afforded desilylated Me3SiCF3 (4 equiv) 82 Cu(OAc)2 (40 mol %) products (Scheme 17, Part (d)). Hu’s group reported that CuF2•2 H2O Y 1,10-phenanthroline (40 mol %) Y catalyzed the reaction between electrophilic fluoroalkylating agents (b) CF3 R N t-BuONa (1.1 equiv) R N such as 5b and α,β-unsaturated carboxylic acids by activating both NaOAc (3 equiv) 32–88% reactants, affording di- and trifluoromethylalkenes in high yields and t-BuOOt-Bu (3 equiv), N2, DCE 83 4 Å MS, sealed tube, 80 oC, 6 h excellent E/Z selectivity (Scheme 17, Part (e)). Furthermore, Shen and collaborators reported a sequential Y = O, S, NMe, N(CH2)2CH=CH2; R = H, Me, Ph, Cl, Br iridium-catalyzed C–H activation borylation and copper-catalyzed

Me3SiCF3 (3 equiv) 84 R1 R1 trifluoromethylation of arenes. The reaction is conducted under mild 3 Cu(OH)2 (10 mol %) 3 R 1,10-phenanthroline (10 mol %) R reaction conditions and tolerates a variety of functional groups. The (c) CF3 N KF (3 equiv) N advantage of this tandem procedure was further demonstrated by the 2 2 R Ag2CO3 (2 equiv), N2, DCE R late-stage trifluoromethylation of a number of biologically active sealed tube, 80 oC, 12 h 64–89% molecules. 1 2 3 R = Me, i-Pr, Cy, c-Pent; R = Me, Et, Bn, Ph; R = H, Cl, Br, MeO2C 3.1.3. Other Copper-Mediated Trifluoromethylations The copper-catalyzed trifluoromethylation of unactivated terminal Scheme 16. Copper-Catalyzed, Direct, C–H Oxidative Trifluoromethylation of alkenes with 5b was reported by Parsons and Buchwald to afford linear Heteroarenes Employing Me SiCF and Oxidizing Reagents. (Ref. 77) 3 3 allylic trifluoromethylated products in good yields and with high E/Z selectivity (Scheme 18, Part (a)).85 This mild trifluoromethylation 77 VOL. 45, NO. 3 • 2012 tolerates a wide variety of functional groups including alcohols, to trifluoromethylate internal and branched alkenes. Chu and Qing protected amines, esters, amides, and alkyl bromides. Epoxide- reported a Cu-catalyzed oxidative trifluoromethylation of terminal containing alkenes can be trifluoromethylated using a copper salt alkenes that proceeds under mild conditions, employs the readily of lower Lewis acidity, namely, copper(I) thiophene-2-carboxylate available and less expensive Me3SiCF3 as the CF3 source, and leads to 88 (CuTC). Since the authors were not confident about the actual reaction efficient formation of sp3C –CF3 bonds (Scheme 18, Part (c)). pathway, they proposed two possibilities, one involving a free radical Szabó and co-workers have very recently reported a regio- and intermediate and the other an alkylcopper species. Independently, stereoselective Cu-catalyzed addition of a hypervalent iodine reagent to Wang and co-workers developed a similar preparation of allylic alkynes and alkenes. In the presence of CuI, the reaction is suitable for trifluoromethylated compounds under mild conditions. Their reaction the trifluoromethylbenzoyloxylation and trifluoromethylhalogenation employs an inexpensive copper chloride as catalyst and a hypervalent of alkenes and alkynes. Electron-donating substituents accelerate 86 iodine(III) reagent as both the oxidant and the CF3 source. the process, and alkenes react faster than alkynes, emphasizing the Alternatively, Fu, Liu, and co-workers used Umemoto’s reagent electrophilic character of the addition reaction.89 (6a) to effect a copper-catalyzed trifluoromethylation of terminal alkenes, leading to linear allylic trifluoromethylated products in low- 3.2. Palladium-Mediated Trifluoromethylation to-good yields with exclusive E selectivity (Scheme 18, Part (b)).87 3.2.1. Pd-Catalyzed Trifluoromethylation of Aryl and As in Buchwald’s method, this protocol tolerates many functional Alkenyl Halides via a Pd(0)–Pd(II) Cycle groups; such as sulfone, ester, amine, ketal, nitro, ether, amide, Buchwald’s group has developed the first palladium-catalyzed ketone, hydroxyl, and epoxide; making this approach attractive to trifluoromethylation of aryl halides using Et3SiCF3 as the trifluoro- process chemists. Both experimental and theoretical calculations have methylating agent.90 Both electron-rich and electron-poor aromatic suggested that a copper(III) intermediate containing CF3 probably chlorides were trifluoromethylated to furnish the desired products adds onto the double bond, followed by formation of a Heck-like four- in good-to-excellent yields (Scheme 19, Part (a)). The reaction is membered-ring transition state. The above two methodologies failed compatible with heteroaromatics; including indoles, carbazoles, quinolines, and benzofurans; and numerous functional groups such as ester, acetal, amide, nitrile, ether, and dialkylamino. Ortho-substituted aryl chlorides gave the corresponding trifluoromethylated products in 1 1 R (1.2 equiv) R 3 5b 3 low yields when the bulky ligand BrettPhos was employed, but RuPhos R CuOAc (10 mol %) R (a) CF3 provided good yields of the products. Unfortunately, this protocol does N MeOH, rt, 6 h N not tolerate ketones, aldehydes, and unprotected OH and NH groups. A R2 R2 (Ref. 78) mechanistic study by the authors confirmed that the reaction proceeds 16–95% through a classical Pd(0)–Pd(II) catalytic cycle. R1 = H, alkyl, ester; R2 = H, Me, Bn, Ac, Boc; R3 = H, MeO, Br The palladium-catalyzed trifluoromethylation of cyclohexenyl 6b (2 equiv) sulfonates has also been implemented by Cho and Buchwald (Scheme Cu (2 equiv) 19, Part (b)).91 Various cyclohexenyl triflates and nonaflates underwent (b) B(OH)2 CF3 R R NaHCO3 (1 equiv) trifluoromethylation under mild reaction conditions using a catalyst 20 DMF, 50 oC, 11 h 66, 46–74% system composed of Pd(dba)2 or [(allyl)PdCl]2 and the monodentate (Ref. 79)

R = alkyl, Ph, PhO, HOCH2, NO2, CN, CHO, halide

5b (1 equiv) 5a (1.2 equiv) R R CF3 CuI (5 mol %) (a) (Ref. 85) 1,10-phenanthroline (CH3CN)4CuPF6 54–80% 1.25 equiv (10 mol %) (15 mol %) E/Z = 89:11 to 97:3 (c) R B(OH)2 R CF3 MeOH, 0 oC to rt, 24 h K2CO3 (2 equiv) 66, 50–95% R = Ph, functionalized alkyl diglyme, 35 oC, 14–24 h

R = vinyl, aryl, heteroaryl (Ref. 81) 6a (1.2 equiv) CuTc (20 mol %) R CF (b) R 3 5b (1.2 equiv) 1 (Ref. 87) 1 R 2,4,6-trimethylpyridine R CuI (10 mol %) 32–78% 2 2 1 equiv (d) [Si] R [Si] CF3 or R (2 equiv) MeOH, rt, 24 h DMAc, 40 oC, 24 h CF3 (Ref. 82) AB R = functionalized alkyl A: ≤93% (R1 = R2 = H); B: ≤84% (R1 = Ar, alkynyl, einyl, alkyl; R2 = H, alkyl) Me3SiCF3 (4 equiv) CuTc (30 mol %) R CF 5b (1 equiv) (c) R 3 R1 R1 (Ref. 88) CuF2•2H2O (20 mol %) K2CO3 (4 equiv) 31–87% (e) 1 equiv 2 2 PhI(OAc)2 (2 equiv) R CO2H 1,4-dioxane, H O R CF3 2 NMP, 80 oC, 24 h 4 equiv 80 oC, 12 h 42–74% = 92:8 to >99:1 R = functionalized alkyl or aryl (Ref. 83) E:Z Tc = thiophene-2-carboxylate; DMAc = N,N-dimethylacetamide

Scheme 17. Copper-Catalyzed Trifluoromethylation Using Electrophilic Scheme 18. Trifluoromethylation of Terminal Alkenes by Togni’s, Umemoto’s, Trifluoro­methylation Reagents. or Ruppert–Prakash’s Reagent. Transition-Metal-Mediated Fluorination, Difluoromethylation, and Trifluoromethylation 78 Zhuang Jin, Gerald B. Hammond,* and Bo Xu*

– biaryl phosphine ligand tBuXPhos. The trifluoromethyl anion (CF3 ) 2-arylpyridines were successfully trifluoromethylated to give 94 or its equivalent for the process was generated in situ from TMSCF3 fluorinated products in good yields (Scheme 20, Part (a)). In many

in combination with KF or from TESCF3 with the addition of RbF. cases, pyrimidine, imidazole, and thioazole can also be suitable Samant and Kabalka developed a highly efficient copper-catalyzed directing groups in this reaction; however, the detailed roles of copper

trifluoromethylation of aryl bromides with Me3SiCF3 in micellar media acetate and TFA are still unknown. (Scheme 19, Part (c)).92 Owing to the use of the surfactant sodium Guosheng Liu and co-workers have reported the oxidative trifluoro- dodecyl sulfate (SDS), this protocol tolerated ketones, aldehydes, and methylation of a wide variety of N,3-disubstituted indoles at C2

unprotected alcohols and amines, which were not compatible with by Me3SiCF3 in the presence of a ligated palladium catalyst to give Buchwald’s process.90 These authors reasoned that micelles prevent trifluoromethylated indoles in moderate-to-good yields (Scheme 20, 95 the decomposition of Me3SiCF3, and that side products can be avoided Part (b)). The authors found that: (a) TEMPO can increase the yield

in this manner. They also proposed that the spatial orientation of the of the final product by inhibiting radical side reactions, (b) PhI(OAc)2

ArLPdCF3 intermediate, resulting from a hydrophilic CF3 and a is a suitable oxidant in this work, (c) indoles having a tosyl group hydrophobic ligand, had a positive effect on the reductive elimination nitrogen or a free amine group are not suitable substrates for this step. Buchwald’s team also developed an efficient synthesis of aryl trifluoromethylation, and (d) when the C2 position of the indole is trifluoromethyl sulfides in excellent yields. Aryl trifluoromethyl blocked, trifluoromethylation occurs at the C3 position (3 examples). sulfides are an important class of compounds in both the pharmaceutical A preliminary mechanistic study showed that a possible catalytic cycle

and agrochemical areas; their synthesis can be achieved under mild proceeds through electrophilic palladation, oxidation by PhI(OAc)2– IV conditions by the Pd-catalyzed reaction of aryl bromides with a Me3SiCF3 to form ArPd CF3, followed by reductive elimination to yield trifluoromethylthiolate nucleophile (Scheme 19, Part (d)).93 the final product. The same research group also reported a palladium- catalyzed intramolecular oxidative aryltrifluoromethylation reaction of 3.2.2. Pd-Catalyzed Trifluoromethylation via a Pd(II)–Pd(IV) activated alkenes (Scheme 20, Part (c)).96 This reaction allows for the

Cycle efficient synthesis of a variety of CF3-containing oxindoles. An initial

Yu and co-workers developed a palladium-catalyzed, nitrogen- mechanistic study suggested that the reaction might involve a Csp3– IV directing-group-assisted, aromatic C–H trifluoromethylation using Pd CF3 intermediate, which undergoes reductive elimination to afford

fluorinating agent 6c and trifluoroacetic acid as promoter. Numerous a Csp3–CF3 bond.

Sanford and her group uncovered direct evidence for aryl–CF3 bond formation via reductive elimination from a palladium(IV) complex. II TESCF3 (2 equiv) ArPd CF3 complexes were prepared, oxidized by the electrophilic [(allyl)PdCl]2 or Pd(dba)2 (3–6 mol %) (a) Ar Cl Ar CF3 (Ref. 90) BrettPhos or RuPhos 70–94% (Pd:ligand=1:1.5) 6c (1.5 equiv) KF (2 equiv), dioxane Pd(OAc)2 (10 mol %) o (a) N N 130–140 C, 6–20 h R R Cu(OAc)2 (1 equiv) Ar = aryl, heteroaryl TFA (10 equiv) CF3 9 DCE, 110 oC, 48 h 54–88% TMSCF (2 equiv), or 3 R = Me, MeO, Cl, other TESCF3 (1.5 equiv) [(allyl)PdCl]2 or Pd(dba)2 X CF3 (4–6 mol %) Me3SiCF3 (4 equiv) (b) R1 R1 (Ref. 91) 3 Pd(OAc)2 (10 mol %) 3 tBuXPhos (8–12 mol %) R Ligand (15 mol %) R R R KF (2 equiv), or (b) CF3 51–84% N PhI(OAc)2 (2 equiv) N RbF (1.5–2 equiv), dioxane CsF (4 equiv) R2 R2 90–100 oC, 3–10 h TEMPO (0.5 equiv) MeCN, rt 33–83% X = OTf or ONf; R = H, Ph, n-Pent, heteroaryl, other 1 2 R = alkyl, MeO2C; R = Me, Et, n-Bu, Bn, Ph, SEM, Ts TMSCF3 (2 equiv) 3 R = H, Me, MeO, Cl, Br, MeO2C [cinnamylPdCl]2 (10 mol %) (c) Ar Br Ar CF3 (Ref. 92) BrettPhos (10 mol %) Me3SiCF3 (4 equiv) CF3 R 68–80% R Pd(OAc)2 (10 mol %) SDS, CsF (2 equiv) H o Ligand (15 mol %) PhMe, 110 C, 12 h (c) Ar Ar O PhI(OAc) (2 equiv) Ar = 1-Np, anthracen-9-yl, substituted benzene N O 2 N Yb(OTf)3 (20 mol %) R' SDS = sodium dodecyl sulfate (a surfactant) R' CsF (4 equiv), EtOAc, rt 43–89%

AgSCF3 (1.3 equiv) R = Me, Ph, HOCH2, AcOCH2, MeOCH2, PhthNCH2, Ph(MeO)CH [(cod)Pd(CH2TMS)2] (1.5–3.0 mol %) R' = Me, Ph, Ts, SEM, Bn (d) Ar Br Ar SCF3 (Ref. 93) O O BrettPhos (1.75–3.30 mol %) 91 to >99% Ph(Et) NI (1.3 equiv) N N 3 SEM = (CH3)3SiCH2CH2OCH2 PhMe, 80 oC, 2 h Ligand Ar = aryl, heteroaryl, substituted benzene

Scheme 19. Palladium-Catalyzed Trifluoromethylation of Aryl Halides and Scheme 20. Pd-Catalyzed, Directing-Group-Assisted C–H Trifluoromethyla- Vinyl Pseudohalides. tions. (Ref. 94–96) 79 VOL. 45, NO. 3 • 2012 fluorinating agent 2c, and the Pd(IV) intermediate reductively eliminated to give trifluoromethylated arenes in low-to-excellent yields R (Scheme 21, Part (a)).97 A more detailed mechanistic study showed that 2c, PhNO2 reductive elimination from the Pd(IV) intermediate probably proceeds (a) L R CF Pd 3 – L CF 80 oC, 3 h through dissociation of TfO , followed by aryl–CF3 coupling (Scheme 3 98 66, 29–89% 21, Part (b)). DFT calculations indicated that, in this context, CF3 R = H, Me, MeO, Ph, F, CF3 L2 = tBu-bpy, tmeda, dppe acts as an electrophile and the aryl ligand as a nucleophile. Sanford C6D5NO2 80 oC, 3 h and co-workers also reported the preparation and isolation of a related F Pd(IV) complex and its characterization by X-ray crystallography. This R = F, 77% F complex was reductively eliminated in different solvents to give the t-Bu 99 2c trifluoromethylated product in moderate yield (Scheme 21, Part (c)). o N CF L DCE, 23 C 3 (b) Pd PdIV L CF3 L = tBu-bpy F 3.3. Trifluoromethylation Mediated by Other Metals N O A silver-mediated cross-coupling of trifluoromethoxide with aryl- -Bu t F3C S O stannanes and arylboronic acids to give aryl trifluoromethyl ethers was 53% O advanced by Ritter and co-workers (Scheme 22, Parts (a) and (b)).100 F3C This is the first report of a transition-metal-mediated arylC –OCF3 bond OH N 2 solvent N formation. (c) PdIV o CF An iron(II)-catalyzed trifluoromethylation of potassium vinyl- OAc 60 C, 12 h 3 trifluoroborates was developed by Buchwald and co-workers (Scheme OAc 22, Part (c)).101 2-Arylvinyl substrates in particular provided flluorinated 54–62% Solvent = AcOH, DCE, CHCl3, PhNO2 major product products in good yields and excellent E/Z ratios. The reaction is amenable to a bench-top setup, and proceeds under exceedingly mild reaction conditions. A preliminary mechanistic analysis suggested that Scheme 21. Direct Evidence for Aryl–CF3 Bond Formation via Reductive the reaction involves a carbocationic intermediate that is promoted by Elimination from a Palladium(IV) Complex. (Ref. 97,99) Lewis acid catalysis, but a radical-type mechanism could not be ruled out. The silver-mediated trifluoromethylation of unreactive arenes with

Me3SiCF3 was reported by Sanford’s group to give trifluoromethylated 102 arenes in moderate-to-high yields (Scheme 22, Part (d)). Additionally, TAS•OCF3 (2 equiv) heteroaromatics, such as N-methylpyrrole, thiophene, and caffeine, 1b (1.2 equiv) (a) Ar Sn(n-Bu)3 Ar OCF3 were also trifluoromethylated to afford the corresponding products AgPF6 (2 equiv) 59–88% with moderate-to-excellent site selectivity. Initial mechanistic studies NaHCO3 (2 equiv) THF–acetone (1:3) showed that a CF3 radical intermediate is not a likely species in this –30 oC, 2–4 h reaction. Ar = substituted benzene, aryl, heteroaryl Hafner and Bräse reported the trifluoromethylation of substituted aromatic triazenes ortho to the triazene functionality in the presence 1. NaOH (1 equiv) MeOH, rt, 15 min I I of in situ generated AgCF3; selectivity for ortho C–H substitution (b) Ar B(OH)2 [ArAg •Ag ] was high and yields were good.103 Owing to the possibility of further 2. AgPF6 (2 equiv) 20 0 oC, 0.5 h elaboration of triazenes, a variety of CF3-substituted building blocks TAS•OCF3 (2 equiv) are now accessible. Our group has disclosed an efficient method for 1b (1.2 equiv) Ar = substituted benzene, NaHCO (2 equiv) preparing a-trifluoromethylated N-heterocycles with medicinally aryl, heteroaryl 3 THF–acetone (1:3) important ring sizes (e.g., 5–7) in a one-pot reaction involving two o + –30 C, 2–4 h tandem nucleophilic additions starting from readily available secondary TAS = (Me2N)3S aminoalkynes (eq 2).104 Ar OCF3 63–76%

3.4. Trifluoromethylation Involving Free Radicals 5b (1 equiv) FeCl2 (10–15 mol %) Yamakawa’s group disclosed an iron(II)-catalyzed trifluoromethylation BF3K CF3 (c) R R of aromatics by ICF in the presence of H O in DMSO. Electron-rich MeCN, rt, 24 h 3 2 2 1.1 equiv 34–79% arenes and heteroaromatics including pyridines, pyrimidines, pyrroles, E:Z = 67:33 to >95:5 indoles, thiophenes, furans, pyrazoles, imidazoles, triazoles, and R = alkyl, styryl, aryl, heteroaryl thiazoles were trifluoromethylated to furnish the desired products in 105 Me3SiCF3 (1 equiv) low-to-high yields (Scheme 23, Part (a)). The authors proposed that AgOTf (4 equiv) (d) HAr Ar CF3 the hydroxyl radical that is formed from the reduction of H2O2 by Fe(II) KF (4 equiv), DCE 5–20 equiv o 66, 42–88% is trapped by DMSO to form a methyl radical, which then reacts with N2, 85 C, 24 h ICF3 to release a CF3 radical that is in turn trapped by the arene. Mejía Ar = Ph, 4-Tol, 1-Np, substituted benzene, heteroaryl and Togni reported that methyltrioxorhenium acts as a catalyst (5–10 mol %) for the direct electrophilic trifluoromethylation of various aromatic and heteroaromatic compounds with the hypervalent iodine Scheme 22. Silver- and Iron-Mediated Trifluoromethoxylations and reagent 5b (Scheme 23, Part (b)). Monitoring of this reaction by EPR Trifluoromethylations.(Ref. 100–102) demonstrated the involvement of radical species.106 Transition-Metal-Mediated Fluorination, Difluoromethylation, and Trifluoromethylation 80 Zhuang Jin, Gerald B. Hammond,* and Bo Xu*

The first enantioselective, organocatalytic a-trifluoromethylation and a-perfluoroalkylation of aldehydes have been accomplished using TMSCF3 R1 AgF (1.5 equiv), H2O (1 equiv) R1 n' a readily available iridium photocatalyst and a chiral imidazolidinone 2 N 107 NHR dioxane, µw, 100 oC, 40 min F C catalyst by MacMillan and co-workers (Scheme 24, Part (a)). n 3 2 R A range of a-trifluoromethyl and a-perfluoroalkyl aldehydes were 87–95% obtained from commercially available perfluoroalkyl halides in high 1 2 n = 1–3; n' = 1–2; R = H, n-Bu; R = Ph, Bn, allyl, Ph(Me)CH efficiency and enantioselectivity. The resulting a-trifluoromethyl aldehydes were subsequently shown to be versatile precursors for eq 2 (Ref. 104) the construction of a variety of enantioenriched trifluoromethylated building blocks. MacMillan’s group also reported an efficient method for the α-trifluoromethylation of carbonyl compounds and enolsilanes through this photoredox catalysis strategy.108 Ando and co-workers

ICF3 (3 equiv) similarly published a rhodium-catalyzed α-trifluoromethylation of FeSO4 or Cp2Fe 109 (0.3–0.5 equiv) ketones via silyl enol ethers. (a) HAr Ar CF 3 Nagib and MacMillan were able to generate CF3 radicals from H2O2 (2–6 equiv) 1 equiv o 66, 6–96% H2SO4, DMSO, 40–50 C a light-induced reaction involving a ruthenium photocatalyst and Ar = Ph, substituted benzene, heteroaryl trifluoromethanesulfonyl chloride (ClSO2CF3), a reagent that can be a useful CF3 radical precursor (Scheme 24, Part (b)). They demonstrated

that their approach can add CF3 groups to already functionalized aryl 5b (1 equiv) 110 MeReO3 (5–10 mol %) groups in molecules such as ibuprofen. (b) HAr Ar CF3 o Very recently, Ye and Sanford reported a mild and general approach CDCl3, 70 C, overnight 1.5 equiv 66, 11–77% for the merged Cu-catalyzed–Ru-photocatalyzed trifluoromethylation Ar = substituted benzene, heteroaryl, ferrocenyl and perfluoroalkylation of arylboronic acids (Scheme 24, Part (c)). This method also takes advantage of visible-light photoredox

catalysis to generate CF3• under mild conditions, and merges it with 111 Scheme 23. Trifluoromethylation of Aromatics Presumed to Proceed copper-catalyzed arylboronic acid functionalization. Baran’s team through a Free Radical Mechanism. (Ref. 105,106) prepared CF3 radicals using tert-butyl hydroperoxide as an oxidant to controllably decompose sodium trifluoromethylsulfinate, NaSO2CF3. This method can be employed for the trifluoromethylation of pyridines 112 and other nitrogen-based heteroaromatics using CF3 radicals; but, since a transition metal is not involved in the reaction, it is not covered

O ICF3 (~ 8 equiv) O in this review. A (20 mol %) + B (0.5 mol %) R R (a) H H 2,6-lutidine (1.1 equiv), DMF CF3 4. Transition-Metal-Promoted Difluoromethylation hν, –20 oC, 7.5–8 h 61–86% Difluoromethylation has received less attention than trifluoro- R = functionalized alkyl, cycloalkyl, aryl 90–99% ee methylation, but in theory, many of the methods employed for trifluoromethylation could also be applied to difluoromethylation. ClSO2CF3 (1–4 equiv) Ru(phen)3Cl2 (1–2 mol %) (b) Ar H Ar CF3 K2HPO4 (anhyd, 3 equiv) 4.1. Difluoromethylation through an RCF2Cu Intermediate 20 70–94% MeCN, hν, 23 oC, 24 h Amii and co-workers have prepared difluoromethylated arenes via Ar = Ph, substituted benzene, aryl, heteroaryl copper-mediated cross-coupling followed by decarboxylation.113 Both electron-rich and electron-deficient aromatic iodides were cross- ICF3 (5 equiv) CuOAc (20 mol %) coupled with a-silyldifluoroacetates to give low-to-high yields of (c) Ar B(OH)2 Ar CF3 a-aryldifluoroacetates, which were subjected to base hydrolysis and Ru(bpy)3Cl2•6H2O (1 mol %) 20 39–93% K2CO3 (1 equiv) decarboxylation to produce the corresponding difluoromethylated DMF, hν, 60 oC, 12 h arenes in moderate-to-high yields (Scheme 25, Part (a)). However, the Ar = Ph, substituted benzene, heteroaryl intermediate a-aryldifluoroacetic acid that possess an electron-donating + group failed to deliver the corresponding difluoromethyated arenes. Zhang and co-workers reported a copper-catalyzed cross-coupling O Me t-Bu N N N of 2-iodobenzoates with bromozinc difluorophosphonate (Scheme 25, + Ir Part (b)), generated from BrCF P(O)(OEt) and zinc in dioxane. Notable Me N t-Bu N 2 2 N H2 -Bu – t features of this reaction are its high efficiency, excellent functional- CF3CO2 – PF6 group compatibility, and operational simplicity. This protocol provides a useful and facile access to aryldifluorophosphonates, of interest in A B 114 115 (organocatalyst) (photocatalyst) life sciences. Qing and co-workers reported a copper-mediated oxidative cross-coupling reaction of terminal alkynes with readily available a-silyldifluoromethylphosphonates under mild conditions (Scheme 25, Part (c)). This method allows for the efficient synthesis Scheme 24. Trifluoromethylation through Photocatalysis.(Ref. 107,110,111) of a series of synthetically useful a,a-difluoropropargylphosphonates

with excellent functional-group compatibility. 81 VOL. 45, NO. 3 • 2012

Fier and Hartwig have disclosed a copper-mediated difluoromethyl- 5. Summary and Outlook ation of electron-neutral or electron-rich, and sterically hindered, aryl Newly developed transition-metal-catalyzed or mediated fluorination, and vinyl iodides using Me3SiCF2H to give the difluoromethylated difluoromethylation, and trifluoromethylation reactions have contributed products in 30–90% yields.116 While aryl iodides possessing an to making the synthesis of fluorinated compounds easier than what electron-withdrawing moiety were not suitable for this methodology, was possible just a few years ago. Despite the great strides, there many functional groups including tertiary amines, amides, ethers, are still important limitations to these new approaches. Compared esters, bromides, ketals, and protected alcohols were tolerated. to traditional methodologies, many of the new methods presented More importantly, this methodology can difluoromethylate vinyl in this review use expensive metal and/or ligands, stoichiometric iodides to give allylic difluoromethylated alkenes with retention of amounts of metal mediators and/or air/water-free conditions. These stereochemistry. limitations make many of them impractical in large-scale synthesis or medicinal chemistry venues. We expect that forthcoming research will 4.2. Difluoromethylation via Radical Species address these shortcomings. So far, most of the fluorination methods Baran and co-workers have developed the new difluoromethylation presented are Pd-based, whereas most of the trifluoromethylation and reagent Zn(SO2CF2H)2 (DFMS) from ClSO2CF2H, and demonstrated its difluoromethylation approaches are Cu- or Ag-based. Recent advances ability to difluoromethylate organic substrates in a mild, chemoselective, using new metal catalysts such as Fe and Au lift our expectations of easy-to-carry-out, and scalable process. Heteroaromatics, such possible breakthroughs in the near future. Likewise, it is our hope that as pyridines, pyrroles, pyrimidines, quinolines, thiadiazoles, and there will be further developments in new radical-based approaches. pyridinones, were readily difluoromethylated by Zn(SO2CF2H)2 in the presence of t-BuOOH as oxidant to give the difluoromethylated 6. Acknowledgements 117 products in low-to-excellent yields (eq 3). Zn(SO2CF2H)2 can We are grateful to the National Science Foundation for financial also difluoromethylate thiols and enones. The authors proposed that support (CHE-1111316). the reaction occurs via a radical process in which the CF2H radical generated from Zn(SO2CF2H)2 possesses a nucleophilic character. 7. References (1) (a) Chambers, R. D. Fluorine in Organic Chemistry; Blackwell Publishing–CRC Press: Boca Raton, FL, 2004. (b) Hiyama, T.

R3SiCF2CO2Et Organofluorine Compounds: Chemistry and Applications; Springer- (1.2 equiv) Verlag: Berlin, 2000. (c) Kirsch, P. Modern Fluoroorganic Chemistry: CuI (0.2–1.0 equiv) (a) Ar I Ar CF2CO2Et Synthesis, Reactivity, Applications; Wiley-VCH: Weinheim, 2004. KF (1.2 equiv) 29–87% DMSO, 60 oC, 15 h (d) Müller, K.; Faeh, C.; Diederich, F. Science 2007, 317, 1881. (e) K2CO3, MeOH Schlosser, M. Angew. Chem., Int. Ed. 1998, 37, 1496. (f) Soloshonok, H2O, rt, 1–2 h KF or CsF (5 equiv) V. A. In Fluorine-Containing Synthons; Soloshonok, V. A., Ed.; ACS Ar CF2H Ar CF2CO2H Symposium Series 911; American Chemical Society: Washington, DMF or NMP 57–89% 170–200 oC, 2–48 h DC, 2005. (g) Uneyama, K. ; Blackwell Ar = Ph, mono- and disubstituted benzene, quinolin-2-yl Publishing: Oxford, 2006. (h) Gerebtzoff, G.; Li-Blatter, X.; Fischer, H.; Frentzel, A.; Seelig, A. ChemBioChem 2004, 5, 676. BrCF P(O)(EtO) CO2Me 2 2 CO2Me (2 equiv) (2) (a) Bégué, J.-P.; Bonnet-Delpon, D. J. Fluorine Chem. 2006, 127, 992. I CuI (0.1 equiv) CF2PO(OEt)2 (b) R R (b) Isanbor, C.; O’Hagan, D. J. Fluorine Chem. 2006, 127, 303. phen (0.2 equiv) (3) (a) Ametamey, S. M.; Honer, M.; Schubiger, P. A. Chem. Rev. 2008, Zn (2.0 equiv) 53–95% 108, 1501. (b) Miller, P. W.; Long, N. J.; Vilar, R.; Gee, A. D. Angew. dixoane, 60 oC 24 or 48 h Chem., Int. Ed. 2008, 47, 8998.

R = H, Me, (MeO)2, NO2, CF3, MeO2C, F, Cl, Br (4) Balz, G.; Schiemann, G. Ber. Dtsch. Chem. Ges. (presently Eur. J. phen = 1,10-phenanthroline Inorg. Chem.) 1927, 60, 1186.

TMSCF2P(O)(R'O)2 (5) Finger, G. C.; Kruse, C. W. J. Am. Chem. Soc. 1956, 78, 6034. CuI, phen (6) Grund, S. C.; Hanusch, K.; Breunig, H. J.; Wolf, H. U. Antimony (c) R R CF2P(O)(OR')2 t-BuOK, DDQ and Antimony Compounds. In Ullmann’s Encyclopedia of Industrial o 40–69% DMF, –15 C Chemistry; Wiley-VCH: Weinheim, Germany, 2000; Vol. 4, pp 11- R = n-Pent, Ph, monosubstituted benzene, 1-Np, 6-MeO-2-Np, thien-3-yl; R' = Et, i-Pr, n-Bu 42. Published online 15 Jul 2006: http://onlinelibrary.wiley.com/ doi/10.1002/14356007.a03_055.pub2/pdf (accessed Oct 16, 2012). (7) Boswell, E. G. A., Jr.; Ripka, W. C.; Scribner, R. M.; Tullock, C. W. Scheme 25. Copper-Mediated Difluoromethylation of Iodoarenes and Org. React. 1974, 21, 1. Terminal Alkynes. (Ref. 113–115) (8) Gouverneur, V. Science 2009, 325, 1630. (9) (a) Furuya, T.; Kamlet, A. S.; Ritter, T. Nature 2011, 473, 470. (b) Hopkinson, M. N.; Gee, A. D.; Gouverneur, V. Isr. J. Chem. 2010,

Zn(SO2CF2H)2 (2–4 equiv) 50, 675. (c) Engle, K. M.; Mei, T.-S.; Wang, X.; Yu, J.-Q. Angew. t-BuOOH (3–6 equiv) Het H Het CF2H Chem., Int. Ed. 2011, 50, 1478. (d) Besset, T.; Schneider, C.; TFA (1 equiv) Cahard, D. Angew. Chem., Int. Ed. 2012, 51, 5048. (e) Gouverneur, CH Cl –H O (2.5:1) 2 2 2 30–90% V.; Lozano, O. Asymmetric Fluorination, Monofluoromethylation, 23 oC 22 examples Difluoromethylation, and Trifluoromethylation Reactions. In eq 3 (Ref. 117) Stereoselective Synthesis 3: Stereoselective Pericyclic Reactions, Cross Coupling, and C–H and C–X Activation; Science of Synthesis Transition-Metal-Mediated Fluorination, Difluoromethylation, and Trifluoromethylation 82 Zhuang Jin, Gerald B. Hammond,* and Bo Xu*

Series, Volume 3; De Vries, J. G., Molander, G. A., Evans, P. A., Series Ed. 2010, 49, 7247. Eds.; Georg Thieme Verlag: Stuttgart, Germany, 2011; Chapter 3.20, (36) (a) Nakamura, M.; Hajra, A.; Endo, K.; Nakamura, E. Angew. Chem., pp 851–930. (f) Tomashenko, O. A.; Grushin, V. V. Chem. Rev. 2011, Int. Ed. 2005, 44, 7248. (b) Bravo, P.; Piovosi, E.; Resnati, G. J. Chem. 111, 4475. (g) Lundgren, R. J.; Stradiotto, M. Angew. Chem., Int. Ed. Res., Synop. 1989, 134. (c) Bravo, P.; Resnati, G. J. Chem. Soc., Chem. 2010, 49, 9322. (h) Hollingworth, C.; Gouverneur, V. Chem. Commun. Commun. 1988, 218. (d) Dolbier, W. R., Jr.; Lee, S. K.; Phanstiel, O., 2012, 48, 2929. IV Tetrahedron 1991, 47, 2065. (10) Herrmann, A. T.; Smith, L. L.; Zakarian, A. J. Am. Chem. Soc. 2012, (37) Liu, L.-P.; Xu, B.; Mashuta, M. S.; Hammond, G. B. J. Am. Chem. Soc. 134, 6976. 2008, 130, 17642. (11) (a) Koller, R.; Stanek, K.; Stolz, D.; Aardoom, R.; Niedermann, K.; (38) (a) Hashmi, A. S. K.; Ramamurthi, T. D.; Rominger, F. J. Organomet. Togni, A. Angew. Chem., Int. Ed. 2009, 48, 4332. (b) Tomita, Y.; Chem. 2009, 694, 592. (b) Zhang, G.; Peng, Y.; Cui, L.; Zhang, L. Ichikawa, Y.; Itoh, Y.; Kawada, K.; Mikami, K. Tetrahedron Lett. 2007, Angew. Chem., Int. Ed. 2009, 48, 3112. (c) Mankad, N. P.; Toste, F. D. 48, 8922. (c) Sato, K.; Yuki, T.; Tarui, A.; Omote, M.; Kumadaki, I.; J. Am. Chem. Soc. 2010, 132, 12859. Ando, A. Tetrahedron Lett. 2008, 49, 3558. (d) Tomita, Y.; Itoh, Y.; (39) De Haro, T.; Nevado, C. Chem. Commun. 2011, 47, 248. Mikami, K. Chem. Lett. 2008, 37, 1080. (40) Hopkinson, M. N.; Giuffredi, G. T.; Gee, A. D.; Gouverneur, V. Synlett (12) (a) Grushin, V. V. Chem.—Eur. J. 2002, 8, 1006. (b) Grushin, V. V.; 2010, 2737. Marshall, W. J. Organometallics 2007, 26, 4997. (c) Grushin, V. V. (41) De Haro, T.; Nevado, C. Adv. Synth. Catal. 2010, 352, 2767. Acc. Chem. Res. 2010, 43, 160. (42) Qian, J.; Liu, Y.; Zhu, J.; Jiang, B.; Xu, Z. Org. Lett. 2011, 13, 4220. (13) Watson, D. A.; Su, M.; Teverovskiy, G.; Zhang, Y.; García-Fortanet, J.; (43) Mankad, N. P.; Toste, F. D. Chem. Sci. 2012, 3, 72. Kinzel, T.; Buchwald, S. L. Science 2009, 325, 1661. (44) Furuya, T.; Ritter, T. Org. Lett. 2009, 11, 2860. (14) Yandulov, D. V.; Tran, N. T. J. Am. Chem. Soc. 2007, 129, 1342. (45) Furuya, T.; Strom, A. E.; Ritter, T. J. Am. Chem. Soc. 2009, 131, 1662. (15) Noël, T.; Maimone, T. J.; Buchwald, S. L. Angew. Chem., Int. Ed. (46) Tang, P.; Furuya, T.; Ritter, T. J. Am. Chem. Soc. 2010, 132, 12150. 2011, 50, 8900. (47) Xu, T.; Mu, X.; Peng, H.; Liu, G. Angew. Chem., Int. Ed. 2011, 50, (16) Maimone, T. J.; Milner, P. J.; Kinzel, T.; Zhang, Y.; Takase, M. K.; 8176. Buchwald, S. L. J. Am. Chem. Soc. 2011, 133, 18106. (48) Kaspi, A. W.; Goldberg, I.; Vigalok, A. J. Am. Chem. Soc. 2010, 132, (17) Hull, K. L.; Anani, W. Q.; Sanford, M. S. J. Am. Chem. Soc. 2006, 128, 10626. 7134. (49) Zhao, S.-B.; Becker, J. J.; Gagné, M. R. Organometallics 2011, 30, (18) (a) Wang, X.; Mei, T.-S.; Yu, J.-Q. J. Am. Chem. Soc. 2009, 131, 7520. 3926. (b) Chan, K. S. L.; Wasa, M.; Wang, X.; Yu, J.-Q. Angew. Chem., Int. (50) Casitas, A.; Canta, M.; Solà, M.; Costas, M.; Ribas, X. J. Am. Chem. Ed. 2011, 50, 9081. Soc. 2011, 133, 19386. (19) Furuya, T.; Ritter, T. J. Am. Chem. Soc. 2008, 130, 10060. (51) Wiemers, D. M.; Burton, D. J. J. Am. Chem. Soc. 1986, 108, 832. (20) Furuya, T.; Benitez, D.; Tkatchouk, E.; Strom, A. E.; Tang, P.; (52) Nowak, I.; Robins, M. J. J. Org. Chem. 2007, 72, 2678. Goddard, W. A., III; Ritter, T. J. Am. Chem. Soc. 2010, 132, 3793. (53) Mitsudera, H.; Li, C.-J. Tetrahedron Lett. 2011, 52, 1898. (21) Ball, N. D.; Sanford, M. S. J. Am. Chem. Soc. 2009, 131, 3796. (54) Singh, R. P.; Shreeve, J. M. Tetrahedron 2000, 56, 7613 and references (22) Furuya, T.; Kaiser, H. M.; Ritter, T. Angew. Chem., Int. Ed. 2008, 47, therein. 5993. (55) Oishi, M.; Kondo, H.; Amii, H. Chem. Commun. 2009, 1909. (23) (a) Young, S. J. Chem. Soc., Trans. 1881, 39, 489. (b) Sun, H.; (56) Weng, Z.; Lee, R.; Jia, W.; Yuan, Y.; Wang, W.; Feng, X.; Huang, K.- DiMagno, S. G. Angew. Chem., Int. Ed. 2006, 45, 2720. (c) Pike, V. W. Organometallics 2011, 30, 3229. W.; Aigbirhio, F. I. J. Chem. Soc., Chem. Commun. 1995, 2215. (57) Hafner, A.; Bräse, S. Adv. Synth. Catal. 2011, 353, 3044. (24) Lee, E.; Kamlet, A. S.; Powers, D. C.; Neumann, C. N.; Boursalian, G. (58) Knauber, T.; Arikan, F.; Röschenthaler, G.-V.; Gooßen, L. J. Chem.— B.; Furuya, T.; Choi, D. C.; Hooker, J. M.; Ritter, T. Science 2011, 334, Eur. J. 2011, 17, 2689. 639. (59) Zhang, C.-P.; Wang, Z.-L.; Chen, Q.-Y.; Zhang, C.-T.; Gu, Y.-C.; Xiao, (25) Trost, B. M.; Van Vranken, D. L. Chem. Rev. 1996, 96, 395. J.-C. Angew. Chem., Int. Ed. 2011, 50, 1896. (26) Katcher, M. H.; Doyle, A. G. J. Am. Chem. Soc. 2010, 132, 17402. (60) Kondo, H.; Oishi, M.; Fujikawa, K.; Amii, H. Adv. Synth. Catal. 2011, (27) Katcher, M. H.; Sha, A.; Doyle, A. G. J. Am. Chem. Soc. 2011, 133, 353, 1247. 15902. (61) Kremlev, M. M.; Tyrra, W.; Mushta, A. I.; Naumann, D.; Yagupolskii, (28) Hollingworth, C.; Hazari, A.; Hopkinson, M. N.; Tredwell, M.; Y. L. J. Fluorine Chem. 2010, 131, 212. Benedetto, E.; Huiban, M.; Gee, A. D.; Brown, J. M.; Gouverneur, V. (62) McReynolds, K. A.; Lewis, R. S.; Ackerman, L. K. G.; Dubinina, G. Angew. Chem., Int. Ed. 2011, 50, 2613. G.; Brennessel, W. W.; Vicic, D. A. J. Fluorine Chem. 2010, 131, 1108. (29) (a) Wu, T.; Yin, G.; Liu, G. J. Am. Chem. Soc. 2009, 131, 16354. (b) (63) Li, Y.; Chen, T.; Wang, H.; Zhang, R.; Jin, K.; Wang, X.; Duan, C. Qiu, S.; Xu, T.; Zhou, J.; Guo, Y.; Liu, G. J. Am. Chem. Soc. 2010, 132, Synlett 2011, 1713. 2856. (64) Kawai, H.; Furukawa, T.; Nomura, Y.; Tokunaga, E.; Shibata, N. Org. (30) Peng, H.; Liu, G. Org. Lett. 2011, 13, 772. Lett. 2011, 13, 3596. (31) Akana, J. A.; Bhattacharyya, K. X.; Mueller, P.; Sadighi, J. P. J. Am. (65) Zanardi, A.; Novikov, M. A.; Martin, E.; Benet-Buchholz, J.; Grushin, Chem. Soc. 2007, 129, 7736. V. V. J. Am. Chem. Soc. 2011, 133, 20901. (32) Gorske, B. C.; Mbofana, C. T.; Miller, S. J. Org. Lett. 2009, 11, 4318. (66) Dubinina, G. G.; Furutachi, H.; Vicic, D. A. J. Am. Chem. Soc. 2008, (33) Schuler, M.; Silva, F.; Bobbio, C.; Tessier, A.; Gouverneur, V. Angew. 130, 8600. Chem., Int. Ed. 2008, 47, 7927. (67) Morimoto, H.; Tsubogo, T.; Litvinas, N. D.; Hartwig, J. F. Angew. (34) (a) Hashmi, A. S. K. Chem. Rev. 2007, 107, 3180. (b) Gorin, D. J.; Chem., Int. Ed. 2011, 50, 3793. Sherry, B. D.; Toste, F. D. Chem. Rev. 2008, 108, 3351. (68) Tomashenko, O. A.; Escudero-Adán, E. C.; Belmonte, M. M.; Grushin, (35) Wang, W.; Jasinski, J.; Hammond, G. B.; Xu, B. Angew. Chem., Int. V. V. Angew. Chem., Int. Ed. 2011, 50, 7655. 83 VOL. 45, NO. 3 • 2012

(69) Chu, L.; Qing, F.-L. J. Am. Chem. Soc. 2010, 132, 7262. Tokuhisa, K.; Yamakawa, T. J. Fluorine Chem. 2010, 131, 98. (70) Luo, D.-F.; Xu, J.; Fu, Y.; Guo, Q.-X. Tetrahedron Lett. 2012, 53, 2769. (106) Mejía, E.; Togni, A. ACS Catal. 2012, 2, 521. (71) Weng, Z.; Li, H.; He, W.; Yao, L.-F.; Tan, J.; Chen, J.; Yuan, Y.; Huang, (107) Nagib, D. A.; Scott, M. E.; MacMillan, D. W. C. J. Am. Chem. Soc. K.-W. Tetrahedron 2012, 68, 2527. 2009, 131, 10875. (72) (a) Chu, L.; Qing, F.-L. Org. Lett. 2010, 12, 5060. (b) Jiang, X.; Chu, (108) Pham, P. V.; Nagib, D. A.; MacMillan, D. W. C. Angew. Chem., Int. Ed. L.; Qing, F.-L. J. Org. Chem. 2012, 77, 1251. 2011, 50, 6119. (73) Senecal, T. D.; Parsons, A. T.; Buchwald, S. L. J. Org. Chem. 2011, 76, 1174. (109) Sato, K.; Yuki, T.; Yamaguchi, R.; Hamano, T.; Tarui, A.; Omote, M.; (74) Chen, C.; Xie, Y.; Chu, L.; Wang, R.-W.; Zhang, X.; Qing, F.-L. Kumadaki, I.; Ando, A. J. Org. Chem. 2009, 74, 3815. Angew. Chem., Int. Ed. 2012, 51, 2492. (110) Nagib, D. A.; MacMillan, D. W. C. Nature 2011, 480, 224. (75) Khan, B. A.; Buba, A. E.; Gooßen, L. J. Chem.—Eur. J. 2012, 18, 1577. (111) Ye, Y.; Sanford, M. S. J. Am. Chem. Soc. 2012, 134, 9034. (76) Qi, Q.; Shen, Q.; Lu, L. J. Am. Chem. Soc. 2012, 134, 6548. (112) Ji, Y.; Brueckl, T.; Baxter, R. D.; Fujiwara, Y.; Seiple, I. B.; Su, S.; (77) Chu, L.; Qing, F.-L. J. Am. Chem. Soc. 2012, 134, 1298. Blackmond, D. G.; Baran, P. S. Proc. Natl. Acad. Sci. U.S.A. 2011, (78) Shimizu, R.; Egami, H.; Nagi, T.; Chae, J.; Hamashima, Y.; Sodeoka, 108, 14411. M. Tetrahedron Lett. 2010, 51, 5947. (113) Fujikawa, K.; Fujioka, Y.; Kobayashi, A.; Amii, H. Org. Lett. 2011, 13, (79) Zhang, C.-P.; Cai, J.; Zhou, C.-B.; Wang, X.-P.; Zheng, X.; Gu, Y.-C.; 5560. Xiao, J.-C. Chem. Commun. 2011, 47, 9516. (114) Feng, Z.; Chen, F.; Zhang, X. Org. Lett. 2012, 14, 1938. (80) Xu, J.; Luo, D.-F.; Xiao, B.; Liu, Z.-J.; Gong, T.-J.; Fu, Y.; Liu, L. (115) Jiang, X.; Chu, L.; Qing, F.-L. Org. Lett. 2012, 14, 2870. Chem. Commun. 2011, 47, 4300. (116) Fier, P. S.; Hartwig, J. F. J. Am. Chem. Soc. 2012, 134, 5524. (81) Liu, T.; Shen, Q. Org. Lett. 2011, 13, 2342. (117) Fujiwara, Y.; Dixon, J. A.; Rodriguez, R. A.; Baxter, R. D.; Dixon, D. (82) Shimizu, R.; Egami, H.; Hamashima, Y.; Sodeoka, M. Angew. Chem., D.; Collins, M. R.; Blackmond, D. G.; Baran, P. S. J. Am. Chem. Soc. Int. Ed. 2012, 51, 4577. 2012, 134, 1494. (83) He, Z.; Luo, T.; Hu, M.; Cao, Y.; Hu, J. Angew. Chem., Int. Ed. 2012, 51, 3944. Trademarks. Selectfluor® (Air Products and Chemicals, Inc.). (84) Liu, T.; Shao, X.; Wu, Y.; Shen, Q. Angew. Chem., Int. Ed. 2012, 51, 540. (85) Parsons, A. T.; Buchwald, S. L. Angew. Chem., Int. Ed. 2011, 50, 9120. About the Authors (86) Wang, X.; Ye, Y.; Zhang, S.; Feng, J.; Xu, Y.; Zhang, Y.; Wang, J. J. Zhuang Jin was born in 1978 in Inner Mongolia, China. He received Am. Chem. Soc. 2011, 133, 16410. his undergraduate (2000) and master’s (2004) degrees from Inner (87) Xu, J.; Fu, Y.; Luo, D.-F.; Jiang, Y.-Y.; Xiao, B.; Liu, Z.-J.; Gong, T.-J.; Mongolia University, where he carried out medicinal chemistry research Liu, L. J. Am. Chem. Soc. 2011, 133, 15300. under the guidance of Professor Gereltu Borjihan. In 2007, Zhuang (88) Chu, L.; Qing, F.-L. Org. Lett. 2012, 14, 2106. Jin enrolled in the doctoral program in chemistry at the University of (89) Janson, P. G.; Ghoneim, I.; Ilchenko, N. O.; Szabó, K. J. Org. Lett. Louisville, where he joined Professor Hammond’s research group. His 2012, 14, 2882. graduate research involved organofluorine chemistry, gold catalysis, (90) Cho, E. J.; Senecal, T. D.; Kinzel, T.; Zhang, Y.; Watson, D. A.; green chemistry, and natural product synthesis. He received his Ph.D. Buchwald, S. L. Science 2010, 328, 1679. degree in 2012, and was awarded The John Richard Binford Memorial (91) Cho, E. J.; Buchwald, S. L. Org. Lett. 2011, 13, 6552. Award. Currently, he is a postdoctoral fellow at The Scripps Research (92) Samant, B. S.; Kabalka, G. W. Chem. Commun. 2011, 47, 7236. Institute in Jupiter, Florida. (93) Teverovskiy, G.; Surry, D. S.; Buchwald, S. L. Angew. Chem., Int. Ed. Gerald B. Hammond was born in Lima, Perú. He received his 2011, 50, 7312. undergraduate degree from the Pontificia Universidad Católica del (94) Wang, X.; Truesdale, L.; Yu, J.-Q. J. Am. Chem. Soc. 2010, 132, 3648. Perú, and obtained his Ph.D. degree in 1985 from the University of (95) Mu, X.; Chen, S.; Zhen, X.; Liu, G. Chem.—Eur. J. 2011, 17, 6039. Birmingham, England. Following an academic career at the University (96) Mu, X.; Wu, T.; Wang, H.; Guo, Y.; Liu, G. J. Am. Chem. Soc. 2012, of Massachusetts Dartmouth, Professor Hammond moved to the 134, 878. University of Louisville in 2004, where he is currently Endowed Chair (97) Ball, N. D.; Kampf, J. W.; Sanford, M. S. J. Am. Chem. Soc. 2010, 132, in Organic Chemistry. His main research interest focuses on the 2878. construction of organic molecules, especially fluorine-containing, and (98) Ball, N. D.; Gary, J. B.; Ye, Y.; Sanford, M. S. J. Am. Chem. Soc. 2011, natural-product-based drug discovery studies on Peruvian plants. 133, 7577. Bo Xu was born in Hubei province, China. He received his (99) Ye, Y.; Ball, N. D.; Kampf, J. W.; Sanford, M. S. J. Am. Chem. Soc. undergraduate degree and master’s degree from East China University 2010, 132, 14682. of Science and Technology, Shanghai, China. He then worked as a (100) Huang, C.; Liang, T.; Harada, S.; Lee, E.; Ritter, T. J. Am. Chem. Soc. research associate at the Shanghai Institute of Organic Chemistry from 2011, 133, 13308. 2000 to 2003. Bo Xu obtained his Ph.D. degree from the University of (101) Parsons, A. T.; Senecal, T. D.; Buchwald, S. L. Angew. Chem., Int. Ed. Louisville in 2008 (advisor Prof. G. B. Hammond), receiving The Guy 2012, 51, 2947. Stevenson Award for Excellence. Dr. Xu is currently a research assistant (102) Ye, Y.; Lee, S. H.; Sanford, M. S. Org. Lett. 2011, 13, 5464. professor at the University of Louisville. His research interests include (103) Hafner, A.; Bräse, S. Angew. Chem., Int. Ed. 2012, 51, 3713. the development of novel and environmentally friendly synthetic (104) Han, J.; Xu, B.; Hammond, G. B. Org. Lett. 2011, 13, 3450. methodologies, especially the synthesis of novel fluorinated building (105) Kino, T.; Nagase, Y.; Ohtsuka, Y.; Yamamoto, K.; Uraguchi, D.; blocks for drugs and advanced materials. Fluolead™– A Novel and Versatile Fluorinating Agent

CH3 SF3

t-Bu CH3

4-tert-Butyl-2, 6-dimethylphenylsulfur trifluoride (Fluolead, 734039)

In recognition of their importance in chemical synthesis, Examples of fluorination reactions mediated by Fluolead. Aldrich® Chemistry is continuously expanding the portfolio of fluorination technologies. (1) Fluorenone O 734039 FF Tolerance of many functional groups HF-py • heptane 70 oC • Highly selective functionalization 92% (2) Polyketone Highly efficient and broad reaction scope O • 734039 F F O HF-py F F Mild reaction conditions O F F • CH2Cl2 o O 50 C F F 80% (3) Dicarboxylic Acid 734039 CO2H CF3 CO2H neat CF3 o 100 C 95%

References: (1) Kirk, K. L. Org. Process Res. Dev. 2008, 12, 305-321. (2) Umemoto, T.; Singh, R. P.; Xu, Y.; Saito, N. J. Am. Chem. Soc. 2010, 132, 18199-18205.

Add Aldrich™ for innovation in your research. Aldrich.com/fluorinationacta

Fluolead is a trademark of UBE America, Inc. 79633

79633_Acta 45.3 Ad - Fluolead.indd 1 11/9/2012 10:21:17 AM New Equipment for Chemical Synthesis

Aldrich® Kugelrohr Bulb-to-Bulb Vacuum Aldrich® Chemical Dryer Distillation Apparatus Dries Samples Using Heat, Vacuum and Desiccant Quickly Distills the Most Difficult Materials with Minimal Loss • Digital thermocouple control • Sample viewing port • Block heater enclosed for safety • Removable drying tube assembly • Distills up to 230 °C • Safe, front loading design • 25 mL to 2 L flask capacity • Precise digital controls • High vacuum capability • Oscillating drive motor

Cat. No. Product Description Cat. No. Product Description Aldrich Chemical Dryer (Order Glass Drying Tube Assembly Separately) Kugelrohr Distillation Apparatus (Order Glassware Separately) Z683507 120 V US plug Z683477 120 V US plug Z683515 230 V Euro plug Z683485 230 V Euro plug Z683523 230 V UK plug Z683493 230 V UK plug Glass Drying Tube Assembly Kugelrohr Glassware Sets Z222712 Complete, ST/NS 34/35 joints Z748293 25 and 100 mL flasks, ST/NS 14/20 joints Z748307 1 L flasks, ST/NS 24/40 joints For product and ordering information, visit Z748315 2 L flasks, ST/NS 24/40 joints Aldrich.com/chemicaldryer For product and ordering information, visit Aldrich.com/kugelrohr

79667

79667_Acta 45.3 Ad - Kugelrohr_Add Aldrich.indd 1 11/14/2012 9:10:26 AM P.O. Box 14508 St. Louis, MO 63178 USA

Speed, Convenience, Service

SEARCH

Aldrich® Market Select is a comprehensive solution for identification and procurement of readily available screening compounds and building blocks. The structure database and purchasing website include regularly updated catalog data from more than 60 of the most reliable suppliers around the world. SELECT

We make compound and building block procurement easy! • Quote & Order Management • Custom Formatting & Packaging • International Shipping and Compliance • Consolidate Invoices and Supplier Payment PURCHASE • Post-procurement Follow-up Support

Purchase your chemistry with ease at www.AldrichMarketSelect.com

OVL 79228 79236 1112

79758_Acta 45.3 Ad - Market Select-Back cover_gray.indd 1 11/13/2012 1:51:26 PM